You are on page 1of 328

Mandatory Quiz - Fundamental of Nursing (FN1) - Attempt 2

The Nurses' Code of Ethics is a formal statement of a group's ideals and values. It is a set of ethical principles that is shared by members of the group, reflects their moral judgments over time, and serves as a standard for their professional actions. Which of the following is NOT included in the nurses' code of ethics? Choose one answer. a. Nurses are the advocates of the client. b. Values, customs, and spiritual beliefs held by individuals are to be respected. c. Personal information acquired in the process of giving nursing care shall be held in strict confidence. d. The hallmark of accurate documentation of actions and outcomes of delivered care is a nursing responsibility. Question 2 Marks: 1 To guide wound care, the nurse can use the RYB color code of wounds, which is based on the concept on the color of an open wound - red, yellow, or black (RYB) - rather than the depth or size of the wound. Which of these is the correct scheme for the wound care using the RYB color code? Choose one answer. a. Debride red, cleanse yellow, protect black b. Protect red, debride yellow, cleanse black

c. Cleanse red, protect yellow, debride black d. Protect red, cleanse yellow, debride black Question 3 Marks: 1 Nursing informatics is the specialty that integrates nursing science, computer science, and information science to manage and communicate data, information and knowledge in nursing practice to support patients, nurses, and other providers in their decision-making in all roles and settings (ANA, 2001). The two most common types of computer systems used by nurses are management information systems (MIS) and hospital information systems (HIS). Which of these is the main difference between MIS and HIS? Choose one answer. a. HIS is more general as MIS is more specific in managing data. b. MIS focuses on the types of data needed to manage client care activities and health care organizations. HIS is designed to facilitate the organization and application of data used to manage an organization or management. c. MIS and HIS do not have any differences. d. MIS is designed to facilitate the organization and application of data used to manage an organization or management. HIS focuses on the types of data needed to manage client care activities and health care organizations. Question 4 Marks: 1 There is a need for recording of the accomplished nursing interventions and the client's responses for effective communication among the members of the health care team. Documentation of nursing activities is included in which phase of the nursing process? Choose one answer. a. Diagnosis b. Evaluation c. Implementation d. Assessment

Question 5 Marks: 1 Assessment is a continuous process carried out during all phases of the nursing process. Which of these is NOT part of the assessment phase? Choose one answer. a. Distinguishing relevant from irrelevant data b. Comparing patterns with norms c. Organizing data d. Validating data

6 Marks: 1 Nurses are almost often involved in the decision-making pertaining to some ethical issues. Which of these situations LEAST require the use of complex ethical decision-making skills in nurses? Choose one answer. a. Your pregnant client is considering abortion due to financial constraints. b. Your client is terminally-ill and his family is saying that he has lived a good life and are considering ending his life. c. Your client has just been recently diagnosed to be HIV-positive. d. Your patient is depressed and does not respond to any of your questions. Question 7 Marks: 1 Positioning a client in good body alignment and changing the position regularly (every 2 hours) and systematically are essential aspects of nursing practice. Your client is experiencing difficulty of breathing, therefore you place him in a semi-Fowler's position. A possible problem of this position is the posterior flexion of the lumbar curvature which may be brought about by the unsupported portion of the upper part of the body elevated at 30-45 degrees commencing at the hips. How will you make sure this problem is prevented?

Choose one answer. a. Place a pillow under the forearms to eliminate pull on shoulder. b. Place a pillow at the lower back. c. Place a pillow under lower legs. d. Place a pillow to support the head, neck and upper back. Question 8 Marks: 1 The management theory developed by McGregor is the Theory X and Theory Y. This theory is about the two different ways to motivate or influence others based on underlying attitudes about human nature. Which of the following describe a Theory X nurse manager's perception of his/her nursing staff? Choose one answer. a. They cannot offer creative solutions to help organizations advance. b. They do not like the work that they are doing. c. They like to be supervised and avoid added responsibilities. d. All of the choices. Question 9 Marks: 1 Bioethical principles are statements about broad, general, philosophical concepts that guide one's particular actions. You are the nurse of a chronically-ill patient. Once, you have promised him that you will come back to talk to him and answer his questions after you have made your rounds to your other patients. However, you have forgotten to keep your word because of the more critical conditions of your other patients. What bioethical principle have you ignored? Choose one answer. a. non-maleficence b. autonomy c. justice d. fidelity

Question 10 Marks: 1 The research design is the overall plan for obtaining answers to the questions being studied and for handling some of the difficulties encountered during the research process. The purpose of the design is to maximize control over factors that can interfere in the validity of the findings. What type of research design is used wherein there is total control of the study? Choose one answer. a. pretest-posttest design b. nonexperimental design c. experimental design d. quasi-experimental design

11 Marks: 1 Nursing practice involves all aspects of the health-illness continuum. You are a nurse in the community taking care of residents with hypertension. When you refer to other health professionals those who have severe hypertension that are already at high-risk for stroke, you are doing which area of nursing practice? Choose one answer. a. Restoring health b. Caring for the dying c. Preventing illness d. Promoting health and wellness Question 12 Marks: 1 A physician orders Ceftriaxone (Rocephin) 2.5 g given via IV piggyback every 8 hours for a client with severe infection. The pharmacy sends a vial labeled 5g/10ml. When preparing the medication, the nurse should use:

Choose one answer. a. 2.5 ml b. 10 ml c. 7.5 ml d. 5 ml Question 13 Marks: 1 The age and developmental stage of a client is an important variable that will influence both the reaction to and expression of pain. You are a nurse in the pediatric ward and your client, Jenny, 5 years old, is often crying because of pain. How will you best divert her attention from the pain experience? Choose one answer. a. Appeal to the child's belief in magic by using a "magic" blanket to take away pain. b. Play music or tapes of a heartbeat. c. Clarify misconceptions on pain. d. Provide a behavioral rehearsal of what to expect and how it will look and feel. Question 14 Marks: 1 Nurses should deliver holistic care to all clients across the lifespan. In doing this, you are putting into practice the theory of: Choose one answer. a. Dorothea Orem b. Florence Nightingale c. Martha Rogers d. Imogene King Question 15 Marks: 1

Leadership is commonly defined as a process of influence in which the leader influences others toward goal achievement. Three leadership styles are still widely recognized today: the autocratic, democratic, and laissez-faire leadership. A laissez-faire leader is differentiated from other types of leadership through which of the following? Choose one answer. a. The leader decides for the whole unit by himself. b. The leader leaves the decision-making up to the group. c. There is less control applied in terms of handling the subordinates. d. The leader acts primarily as a facilitator and a resource person.

Marks: 1 Republic Act No. 9173, otherwise known as the Philippine Nursing Act of 2002 was implemented to define the scope of nursing practice, licensing requirements, and standards of nursing care. The features of this Law include the following, EXCEPT: Choose one answer. a. A member of the Board of Nursing should have at least ten (10) years of continuous practice of the nursing profession provided that the last five (5) years of which shall be in the Philippines. b. The Board of Nursing shall designate the places and dates of the Nursing Licensure Examination. c. The Certificate of Registration may be revoked or suspended if the nurse demonstrated unprofessional or unethical conduct. d. A faculty in a college of nursing should have at least one (1) year of clinical practice in a field of specialization. Question 17 Marks: 1 In health care delivery, basic ethical principles assist the health professionals to determine the right or wrong in regard to value issues involving the pursuit of health, alleviation of suffering, and assisting patients toward peaceful death. Explanation by the attending physician of the important findings, management, prognosis and evaluations to a newly-diagnosed breast cancer patient is an

application of: Choose one answer. a. non-maleficence b. veracity c. autonomy d. beneficence Question 18 Marks: 1 The nursing care of older adults poses special challenges because of great variation in their physiological, cognitive, and psychosocial health. Which of the following describes GERONTICS? Choose one answer. a. It is the study of all aspects of aging process and its consequences. b. It considers the nursing care of older adults to be the art and practice of nurturing, caring, and comforting rather than merely the treatment of disease. c. It is concerned with the assessment of health and functional status of older adults; diagnosis, planning and implementing health care and services, and; evaluating the effectiveness of such care. d. It is the branch of medicine that deals with the physiological and psychological aspects of aging and with diagnosis and treatment of diseases affecting older adults. Question 19 Marks: 1 Vital signs, which should be looked at in total, are checked to monitor the functions of the body. A client with end-stage renal disease is undergoing hemodialysis. He has an AV fistula on his right arm and the client's chart reads "save both arms" for a possible creation of another fistula on his left arm. When taking his blood pressure, you know that the only possible way is to take it on his thighs. Where should you place the BP cuff? Choose one answer. a. Around the knee of the client

b. Around his mid-thigh with the bladder over the posterior aspect of the thigh and the bottom edge around the knee c. Around the upper portion of his mid-thigh with the bladder over the posterior aspect the thigh and the bottom edge in line with the knee. d. Around his lower thigh with the bladder over the posterior aspect of the thigh and the upper edge around the knee Question 20 Marks: 1 Liver biopsy is the removal of a small amount of liver tissue, usually needle aspiration. During the procedure, you instruct the client to inhale and exhale deeply several times and to hold his breath after the final exhalation for up to 10 seconds as the needle is inserted. What is the main purpose why the client needs to holds his breath during needle insertion? Choose one answer. a. Holding the breath minimizes the pain experienced by the client as the needle is inserted. b. This immobilizes the chest wall and liver and keeps the diaphragm in its highest position, avoiding injury to the lung and liver. c. For comfort purposes only d. Holding the breath contracts the abdominal muscles so that the contour of the liver will be well-defined for easier insertion of the needle.

21 Marks: 1 Clients who have been immobilized for even a few days may require assistance with ambulation. You are assisting your client while she tries to ambulate after her surgery 2 days ago. Suddenly, she feels weak and seems to be fainting, however, there is no nearby chair or wheelchair wherein you can lower the patient. You can assist the client to a horizontal position on the floor before fainting occurs. You can do the following, EXCEPT: Choose one answer. a. Assume a broad stance with both feet parallel to each other.

b. Allow the client to slide down your leg, and lower the person gently to the floor. c. Bring the client backward so that your body supports the person. d. All of the above. Question 22 Marks: 1 Nurses document evidence of the nursing process in a variety of forms throughout the clinical record. You are a nurse in the medical ward and you are assigned to a new patient for the shift. You want to know about the case of the patient and the kind of nursing care and therapeutic management already done to help her in her condition throughout her stay at the hospital. You therefore read the: Choose one answer. a. Nursing discharge summary b. Progress notes c. Kardex d. Flow sheet Question 23 Marks: 1 Arterial blood gases (ABGs) are performed to evaluate the client's acid-base balance and oxygenation. Your client is experiencing a prolonged, severe, asthma attack. Which of the following ABG results would you anticipate? Choose one answer. a. Decreased PaCO2, decreased PaO2, and increased pH. b. Decreased PaCO2, increased PaO2, and decreased pH. c. Increased PaCO2, decreased PaO2, and decreased pH. d. Increased PaCO2, increased PaO2, and increased pH. Question 24 Marks: 1

Nursing practice is governed by many legal concepts and it is important for nurses to know the basics of legal concepts, because nurses are accountable for their professional judgments and actions. You inserted a Foley catheter to a patient with urinary retention even though the patient refused to. You informed the patient that this will benefit her. Even if you have a good intention, you are liable for: Choose one answer. a. battery b. invasion of privacy c. assault d. unintentional tort Question 25 Marks: 1 Negligence is defined as a misconduct or practice that is below the standard expected of an ordinary, reasonable, and prudent person. Which of the following situations will most likely cause the nurse to be sued for negligence? Choose one answer. a. Nurse Pat gave the wrong antibiotic and after 30 minutes the patient experienced an anaphylactic reaction. b. Nurse Anne noticed that she aspirated 1 ml of the medication instead of the prescribed 0.5 ml in a syringe. c. Nurse Cecille is about to administer an oral medication when the patient complained that it is not the one usually given to her. d. Nurse Abby mixed a medication in the wrong kind of IV fluid during preparation in the medication room.

26 Marks: 1 Electrolytes, charged ions capable of conducting electricity, are present in all body fluids and fluid compartments. When you are assessing your patient, you noticed the following signs and symptoms: numbness, tingling of the extremities and around the mouth, muscle tremors, cramps and hyperactive deep tendon reflexes. Which of the

following electrolytes is most likely to be imbalanced in your patient? Choose one answer. a. Magnesium b. Potassium c. Sodium d. Calc ium Question 27 Marks: 1 The level of consciousness, which is a component of the neurologic examination, can be measured using the Glasgow Coma Scale (GCS). Marie, a 45-year-old woman, was brought in at the Emergency Department after a vehicular accident. She opens her eyes to verbal command, moves to localized pain and makes sounds that are incomprehensible. What will be her score on the GCS that you, as her nurse, will document on her chart? Choose one answer. a. 11 b. 10 c. 8 d. 9 Question 28 Marks: 1 Total parenteral nutrition (TPN) affords the provision of energy and nutrients intravenously and it is an important responsibility of the nurse to monitor the patient regularly for possible complications related to this therapeutic management. You are regularly assessing your client who is receiving TPN for signs of hyperglycemia. You will note which of the following if your client is already hyperglycemic? Choose one answer. a. Fever, nausea, vomiting b. Nausea, vomiting and chills

c. Cold clammy skin, sweating and weakness d. Thirst, increased urine output and warm flushed skin Question 29 Marks: 1 When using an instrument or tool in quantitative research, it is important to ensure its reliability and validity. Reliability is the consistency with which it measures the target attribute, while validity refers to the degree to which an instrument measures what it is supposed to measure. Harry, a nurse researcher, has found an instrument to measure the level of self-esteem of school-age children. He wants to test its validity so he consults a panel of experts to evaluate the validity of the instrument. This is called: Choose one answer. a. None of the above b. content validity c. face validity d. Both content and face validity Question 30 Marks: 1 Rhonchi (gurgles) are continuous, low-pitched, coarse, gurgling, harsh, louder sounds with a moaning or snoring quality. A nurse understands that the cause of this adventitious breath sounds is: Choose one answer. a. Air passing through narrowed air passages as a result of secretions, swelling, and tumors. b. Rubbing together of inflamed pleural surfaces. c. Air passing through fluid or mucus in any air passage. d. Air passing through a constricted bronchus as a result of secretions, swelling, and tumors.

31

Marks: 1 Because nursing research usually focuses on humans, a major nursing responsibility is to be aware of and to advocate on behalf of client's rights. Which of the following statements best describe the right to self-determination in research? Choose one answer. a. assurance of the anonymity of a study participant b. avoidance of any exposure to the possibility of injury beyond everyday situations c. freedom from constraints, or any undue influences to participate in a study d. provision of complete information about the research/study Question 32 Marks: 1 The nurse-manager decides what task should be done, when, where and by whom. This can be done through delegation. Which of the following is NOT a reason for delegating? Choose one answer. a. Delegation reduces managerial costs. b. Delegation maximizes the use of talents of staff associates. c. Delegation is an opportunity to transfer one's accountability to a task. d. Delegation saves time and can help develop others. Question 33 Marks: 1 Cardiopulmonary resuscitation (CPR) is a combination of oral resuscitation (mouth-to-mouth breathing), which supplies oxygen to the lungs, and external cardiac massage (chest compressions), which is intended to reestablish cardiac function and blood circulation. Based on the latest guidelines in CPR, what is the compression-to-ventilation ratio for all clients except that of newborns? Choose one answer. a. 10 compressions:2 breaths

b. 15 compressions:2 breaths c. 5 compressions:1 breath d. 30 compressions:2 breaths Question 34 Marks: 1 Nursing care oriented to health promotion, wellness, and illness prevention can be understood in terms of health activities on the different levels of preventive care. A client is a known hypertensive for 5 years. She has been taught at the health center about the possible complications of the disease if it is not managed properly and this includes stroke. She has modified her lifestyle since then by quitting smoking, exercising regularly and by eating a balanced diet. These activities demonstrate: Choose one answer. a. Tertiary prevention b. Secondary prevention c. Primary prevention d. None of the choices Question 35 Marks: 1 An enema is a solution introduced into the rectum and the large intestine to distend the intestine and sometimes to irritate the intestinal mucosa, thereby increasing peristalsis and the excretion of feces and flatus. Which of the following clients is more likely to receive a retention enema? Choose one answer. a. A 45-year-old man who will undergo colonoscopy on the following day b. A 5-year-old child with ascariasis c. A client experiencing excessive flatus. d. A 28-year-old female client to undergo Caesarian section the next day

36 Marks: 1 Several positions are frequently required during the physical assessment, however it is important to consider the client's ability to assume a position as well as their physical condition, energy level, and age. Mrs. AF, 65 years old, is diagnosed to have cervical cancer for 5 years already, however, there are no co-morbidities present. She is in the clinic for a follow-up and the nurse is to assess her genital area. What position should the nurse ask Mrs. AF to assume? Choose one answer. a. Dorsal recumbent b. Lithotomy c. Sims d. Prone Question 37 Marks: 1 Because of their unique position in the health care system, nurses experience conflicts among their loyalties and obligations to clients, families, primary care providers, employing institutions, and licensing bodies. The attending physician of your patient placed an order in the chart to administer a pain medication STAT. However, when you arrived in your patient's room, he seems to not be in pain. What will be your action? Choose one answer. a. Wait until the patient experiences pain again then administer the medication. b. Withhold the medication, further assess the patient and inform the attending physician of your findings. c. Administer the medication anyway because you might get reprimanded by the physician. d. Inform the charge nurse of the condition. Question 38 Marks: 1

Informed consent is an agreement by a client to accept a course of treatment or a procedure after being provided complete information, including the benefits and risks of treatment, alternatives to the treatment, and prognosis if not treated by a health care provider. An informed consent needs to be acquired for the performance of endoscopy for a 17-year-old male adolescent, however, there are no other significant others present during that time. A nurse should remember that the patient: Choose one answer. a. Is able to give voluntary consent when his parents are not available b. Will most likely be unable to choose between alternatives when asked to consent c. Is not able to make an acceptable or intelligent choice d. Does not have the legal capacity to give consent Question 39 Marks: 1 Thoracentesis is used to remove the excess fluid or air to ease breathing. Your client is to undergo thoracentesis and you assist him to assume a position that is indicated for the procedure. Which of these can be his position during the procedure? 1 Sitting position with the arms above the head 2 Sitting position with the arm elevated and stretched forward 3 Sitting position in which the client leans over a pillow or overbed table 4 Sitting position with both arms crossed in front of the chest Choose one answer. a. All of the above b. 3 and 4 c. 2, 3 and 4 d. 1, 2 and 3 Question 40 Marks: 1 Determining the different variables in research is an important task of the researcher. In the research question, "What is the effect of

progressive muscle relaxation on the blood pressure of hypertensive residents in an urban community in Metro Manila?", which is the independent variable? Choose one answer. a. hypertensive residents b. urban community in Metro Manila c. blood pressure d. progressive muscle relaxation

41 Marks: 1 Urinary incontinence, or involuntary urination is a symptom, not a disease. It can have a significant impact on the client's life, creating physical problems such as skin breakdown and possibly leading to psychosocial problems such as embarrassment, isolation, and social withdrawal. Your client, Melanie, is 8 months pregnant and she reports dribbling of urine whenever she laughs, coughs, or sneezes. What will be your appropriate nursing diagnosis and intervention for her? Choose one answer. a. Urge urinary incontinence - increase bladder's ability to hold urine and the client's ability to suppress urination. b. Reflex urinary incontinence - regular periodic use of a catheter to empty the bladder c. Stress urinary incontinence - encourage pelvic muscle exercises d. Functional urinary incontinence - encourage timed verbal toileting reminders and positive social feedback for successful toileting Question 42 Marks: 1 Peplau's theory involves the use of a therapeutic relationship between the nurse and the client. When the client assumes the dependent client role, the nurse and client are on which phase of the therapeutic relationship? Choose one answer.

a. Resolution b. Orientation c. Identification d. Exploitation Question 43 Marks: 1 One of the most potentially hazardous procedures that health care personnel face is using and disposing of needles and sharps. Needlestick injuries can be prevented if these guidelines are followed, EXCEPT: Choose one answer. a. Never bend or break needles before disposal. b. When recapping a needle, use a one-handed "scoop" method. c. Recap used needles. d. Use appropriate puncture-proof disposal containers to dispose of uncapped needles and sharps. Question 44 Marks: 1 R.A. 9173 (The Philippine Nursing Act of 2002)defines the scope of nursing practice in the Philippines. Which of the following statements is NOT included in the scope of nursing? Choose one answer. a. A community health nurse teaching a group of hypertensive clients about regular monitoring of blood pressure. b. A nurse performing internal examination of a woman during labor who experienced minimal vaginal bleeding during the second trimester. c. A nurse administering an IV antibiotic after being ordered by the physician. d. A nurse providing oral hygiene to a bed-ridden patient. Question 45

Marks: 1 Preoperative teaching is a vital part of nursing care to reduce clients' anxiety and postoperative complications and increases their satisfaction with the surgical experience. Which of the following statements by the client indicates that preoperative teaching regarding gallbladder surgery has been effective? Choose one answer. a. "After surgery, I can immediately resume the diet I had before I was operated." b. "I am not going to cough after surgery because if I do, my wound will tear apart." c. "I cannot eat or drink anything at least 8 hours before my surgery." d. "I will only do deep breathing exercises when I am experiencing pain."

46 Marks: 1 Confidentiality is the protection of the participants in a study such as that individual identities are not linked to information provided and are never publicly divulged. Which of the following statements best manifest confidentiality? Choose one answer. a. The research study was done in the pediatric wards of a tertiary hospital in Manila. b. The respondents are employees of the only private hospital in the municipality of San Juan in Batangas. c. The patient is L.C.R., 34, female of 5 Jupiter Street, Makati. d. none of the above Question 47 Marks: 1 Evidence-based practice is the conscientious, explicit, and judicious use of current best evidence in making decisions about the care of individual patients. You are a nurse-manager in the pay ward who uses evidence-based practice in planning care for your clients. Which

of the following is your priority in planning for these clients? Choose one answer. a. Care plans are individualized according to the client's needs. b. Care plans follow the standards of care established by the institution's nursing service. c. Care plans that are standardized are used on all patients. d. Care plans are based on the nurse's clinical experience and from the latest research findings. Question 48 Marks: 1 It is important that nurses make nursing diagnoses with a high level of accuracy. Which of these guidelines should be followed when writing a nursing diagnostic statement? Choose one answer. a. Use medical terminology rather than nursing terminology to describe the probable cause of the client's response b. Use statements based on nurse's perception of the client's response c. Word the statement so that it is legally advisable d. Word the diagnosis as generally as possible Question 49 Marks: 1 Planning is a deliberative, systematic phase of the nursing process that involves decision making and problem solving. This phase includes establishing client goals and objectives. Which of the following objectives for the nursing diagnosis of Ineffective Airway Clearance related to poor cough effort is NOT properly stated? Choose one answer. a. The client will not experience aspiration. b. The client will show no signs of pallor or cyanosis by 12 hours postsurgery. c. The client will demonstrate effective coughing and increased air

exchange within 24 hours after surgery. d. The client will have lungs clear upon auscultation during entire postoperative period. Question 50 Marks: 1 During physical assessment of the skin, the nurse may palpate it to locate skin lesions, which are any pathological skin changes. During palpation of the skin, the nurse assesses skin lesions, which she documented in the client's chart as a nodule. Which of the following describes a nodule? Choose one answer. a. Irregularly shaped, elevated area or superficial localized edema that varies in size. b. Solid mass that may extend deep through the subcutaneous tissue, larger than 1-2 cm. c. A palpable, circumscribed, solid elevation in the skin, smaller than 0.5 cm. d. An elevated solid mass, deeper and firmer than a papule, that is 0.5-2 cm.

ANSWERS 1 Marks: 1 The Nurses' Code of Ethics is a formal statement of a group's ideals and values. It is a set of ethical principles that is shared by members of the group, reflects their moral judgments over time, and serves as a standard for their professional actions. Which of the following is NOT included in the nurses' code of ethics? Choose one answer. a. Nurses are the advocates of

the client.

Incorrect

Marks for this submission: 0/1. Question 2 Marks: 1

To guide wound care, the nurse can use the RYB color code of wounds, which is based on the concept on the color of an open wound

I
b. Values, customs, and spiritual beliefs held by individuals are to be respected. c. Personal information acquired in the process of giving nursing care shall be held in strict confidence. d. The hallmark of accurate documentation of actions and - the hallmark of accurate documentation of outcomes of delivered care is actions and outcomes of delivered care is a a nursing responsibility. nursing responsibility. Instead of responsibility, accurate documentation is a nursing ACCOUNTABILITY. The other choices are included in the nurses' code of ethics (Kozier and Balita).

- red, yellow, or black (RYB) - rather than the depth or size of the wound. Which of these is the correct scheme for the wound care using the RYB color code? Choose one answer. a. Debride red, cleanse yellow, protect black

b. Protect red, debride yellow, cleanse black

c. Cleanse red, protect yellow, debride black

d. Protect red, cleanse yellow, debride black

- protect red, cleanse yellow, debride black. Red wounds are usually in the late regeneration phase of tissue repair. They need to be protected to avoid disturbance to regenerating tissue. Yellow wounds are characterized primarily by liquid to semiliquid "slough" that is often accompanied by purulent drainage or previous infection. Yellow wounds are cleansed to remove nonviable tissue. Black wounds are covered with thick necrotic tissue, or eschar. They require debridement or removal of the necrotic material. Removal of nonviable tissue from a wound must occur before the wound can heal (Kozier).

Incorrect

Marks for this submission: 0/1. Question 3 Marks: 1 Nursing informatics is the specialty that integrates nursing science, computer science, and information science to manage and communicate data, information and knowledge in nursing practice to support patients, nurses, and other providers in their decision-making in all roles and settings (ANA, 2001). The two most common types of computer systems used by nurses are management information systems (MIS) and hospital information systems (HIS). Which of these is the main difference between MIS and HIS? Choose one answer. a. HIS is more general as MIS is more specific in managing data.

b. MIS focuses on the types of data needed to manage client care activities and health care organizations. HIS is designed to facilitate the organization and application of data used to manage an organization or management.

c. MIS and HIS do not have any differences.

d. MIS is designed to facilitate the organization and application of data

- MIS is designed to facilitate the organization and application of data

used to manage an organization or management. HIS focuses on the types of data needed to manage client care activities and health care organizations.

used to manage an organization or management. HIS focuses on the types of data needed to manage client care activities and health care organizations. HIS is a type of MIS but is more specific to health care. In MIS, all levels of management benefit from the ability to access data. Typically, an HIS will have subsystem in the areas of admission, medical records, clinical laboratory, pharmacy, order entry and finance. (Kozier).

Incorrect Marks for this submission: 0/1. Question 4 Marks: 1 There is a need for recording of the accomplished nursing interventions and the client's responses for effective communication among the members of the health care team. Documentation of nursing activities is included in which phase of the nursing process? Choose one answer. a. Diagno sis

b. Evaluati on

c. Implem - implementation. Implementation is the phase in the nursing entatio process wherein the nurse performs the nursing interventions. It n also consists of doing and documenting the activities that are the specific nursing actions needed to carry out the interventions. Completion of the implementation phase is done by recording the interventions and the client's responses in the nursing progress notes (Kozier). d. Assess ment

Incorrect Marks for this submission: 0/1. Question 5 Marks: 1 Assessment is a continuous process carried out during all phases of the nursing process. Which of these is NOT part of the assessment phase? Choose one answer. a. Distinguishing relevant from irrelevant data

b. Comparing patterns with norms

- comparing patterns with norms. This activity is included in the diagnosis phase of the nursing process. Meanwhile, assessment is the systematic and continuous collection, organization, validation, and documentation of data. The data gathered in the assessment phase are used in the next phase, diagnosing wherein they are analyzed and

synthesized into nursing problems (Kozier)

c. Organizing data

d. Validating data

6 Marks: 1 Nurses are almost often involved in the decision-making pertaining to some ethical issues. Which of these situations LEAST require the use of complex ethical decision-making skills in nurses? Choose one answer. a. Your pregnant client is considering abortion due to financial constraints.

b. Your client is terminally-ill and his family is saying that he has lived a good life and are considering ending his life.

Incorrect

Marks for this submission: 0/1. Question 7 Marks: 1

Positioning a client in good body alignment and changing the position regularly (every 2 hours) and systematically are essential aspects of nursing practice. Your client is experiencing difficulty of breathing, therefore you place him in a semi-Fowler's position. A possible problem of this position is the posterior flexion of the lumbar

I
c. Your client has just been recently diagnosed to be HIVpositive. - your patient is depressed and does not respond to any of your questions. This is the least priority among all the choices. Abortion is a highly publicized issue about which many people feel very strongly. d. Your patient is depressed and However, nurses have no right to impose does not respond to any of your their values on a client but support clients' questions. right to information and counseling in making decisions. Concerning HIV and AIDS, there has been a strong social stigma and the moral obligation to care for this kind of client cannot be set aside unless the risk exceeds responsibility. In end-of-life issues, it is of utmost importance during this time is to provide them with information and professional assistance, as well as the highest quality of care and caring (Kozier).

curvature which may be brought about by the unsupported portion of the upper part of the body elevated at 30-45 degrees commencing at the hips. How will you make sure this problem is prevented? Choose one answer. a. Place a pillow under the forearms to eliminate pull on shoulder.

b. Place a pillow at the lower back.

- place a pillow at the lower back. Placing a supportive device such as a pillow on the lumbar region will support this part of the body when placed in a Fowler's position. This will prevent the posterior flexion of the lumbar curvature. Placing a pillow under the lower legs will prevent pressure on the heels. Placing a pillow under the forearms will prevent shoulder muscle strain and possible dislocation of the shoulders. Placing a pillow to support the head, neck and upper back will prevent hyperextension of the neck (Kozier).

c. Place a pillow under lower legs.

d. Place a pillow to support the head, neck and upper back.

Incorrect

Marks for this submission: 0/1. Question 8 Marks: 1 The management theory developed by McGregor is the Theory X and Theory Y. This theory is about the two different ways to motivate or influence others based on underlying attitudes about human nature. Which of the following describe a Theory X nurse manager's perception of his/her nursing staff? Choose one answer. a. They cannot offer creative solutions to help organizations advance.

b. They do not like the work that they are doing.

c. They like to be supervised and avoid added responsibilities.

d. All of the choices.

The answer is D - all of the choices. The Theory X view is that in bureaucratic organizations, employees prefer security, direction, and minimal responsibility. Coercion, threats, or punishment are necessary because people do not like the kind of work that they are doing. On the other hand, Theory Y's

assumptions are that in the right conditions, people enjoy their work; can show self-control and discipline; are able to contribute creatively and are motivated by ties to the group, the organization, and the work itself (Kelly-Heidenthal). Incorrect Marks for this submission: 0/1. Question 9 Marks: 1 Bioethical principles are statements about broad, general, philosophical concepts that guide one's particular actions. You are the nurse of a chronically-ill patient. Once, you have promised him that you will come back to talk to him and answer his questions after you have made your rounds to your other patients. However, you have forgotten to keep your word because of the more critical conditions of your other patients. What bioethical principle have you ignored? Choose one answer. a. nonmalefic ence

b. autono my

c. justice

Incorrect

Marks for this submission: 0/1. Question 10 Marks: 1

The research design is the overall plan for obtaining answers to the questions being studied and for handling some of the difficulties encountered during the research process. The purpose of the design is to maximize control over factors that can interfere in the validity of the findings. What type of research design is used wherein there is total control of the study? Choose one answer. a. pretestposttest design

In
- fidelity. The bioethical principle of fidelity means to be faithful to agreements and promises. Clients take such problems seriously and nurses should, too. Autonomy refers to the right to make one's d. fidelity own decisions. Nurses who follow this principle should recognize that each client is unique. Justice is fair, equitable, and appropriate treatment according to what is due or owed to persons. Nonmaleficence is the duty to "do no harm". This requires nurses to act in such a way as to avoid causing harm to clients. Meanwhile, the bioethical principle of beneficence means "doing good" and it requires nurses to act in ways that benefit clients. Lastly, the principle of veracity refers to the practice of telling the truth (Balita and Kozier). b. nonexperi mental

design

11

Marks: 1

Nursing practice involves all aspects of the health-illness continuum. You are a nurse in the community taking care of residents with hypertension. When you refer to other health professionals those who have severe hypertension that are already at high-risk for stroke, you are doing which area of nursing practice? Choose one answer.

In
- experimental design. An experimental design is used by a c. researcher wherein the researcher controls (manipulates) the experiment independent variable and randomly assigns subjects to different al design conditions. A pretest-posttest design is an example of an experimental study. A quasi-experimental design is an intervention study in which subjects are not randomly assigned to treatment conditions, but the researcher exercises certain controls to enhance the study's validity by manipulating the independent variable. A nonexperimental design is wherein the researcher does no manipulation of the independent variable. d. quasiexperiment al design a. Restoring - restoring health. This area of nursing practice focuses on the health ill client, and it covers early detection of the disease to helping the client during the recovery period. Promoting health and wellness involves enhancing the individual's or community's lifestyle. In illness prevention, maintenance of health through prevention of disease is the goal. This can be achieved through

Incorrect

Marks for this submission: 0/1. Question 12 Marks: 1

A physician orders Ceftriaxone (Rocephin) 2.5 g given via IV piggyback every 8 hours for a client with severe infection. The pharmacy sends a vial labeled 5g/10ml. When preparing the medication, the nurse should use: Choose one answer.

Co
immunizations, prenatal care, etc. Caring for the dying includes helping clients live as comfortably as possible until death and helping significant others cope with death (Kozier). b. Caring for the dying c. Preventing illness d. Promoting health and wellness

a. 2.5 ml

Incorrect

Marks for this submission: 0/1. Question 13 Marks: 1

The age and developmental stage of a client is an important variable that will influence both the reaction to and expression of pain. You are a nurse in the pediatric ward and your client, Jenny, 5 years old, is often crying because of pain. How will you best divert her attention

In
b. 10 ml c. 7.5 ml

d. 5 ml - 5 ml. The amount of drug to be given is computed using the formula: amount to administer (x)=desired dose or dose ordered/dose on hand multiplied by the quantity on hand. For this item, we compute the amount to administer (x) by using 2.5 g/5 g x 10 ml. The answer will be therefore, 5 ml (Lippincott NCLEX Reviewer).

from the pain experience? Choose one answer. - appeal to the child's belief in magic by using a "magic" blanket to take away pain. Preschool children develop the ability to describe pain and its intensity and location, and may consider pain a. Appeal to the child's as a punishment therefore reasoning with a child belief in magic by using a at this stage is not always successful. "magic" blanket to take Preschoolers are magical thinkers, therefore this away pain. strategy may succeed. Clarifying misconceptions on pain is for the elderly because they perceive pain as part of the aging process. Playing music or tapes of a heartbeat is applicable for infants because of their need for tactile stimulation. Providing a behavioral rehearsal of what to expect and how it will look and feel is for the school-age children because at this age they already rationalize in an attempt to rationalize the pain (Kozier). b. Play music or tapes of a heartbeat.

c. Clarify misconceptions on pain.

d. Provide a behavioral rehearsal of what to expect and how it will look and feel.

Incorrect Marks for this submission: 0/1. Question 14 Marks: 1 Nurses should deliver holistic care to all clients across the lifespan. In doing this, you are putting into practice the theory of: Choose one answer. a. Dorothea Orem

b. Florence Nightingal e

- Martha Rogers. Her Theory of Unitary Human Beings emphasized that the person is an irreducible whole and that the c. Martha nurse should seek to promote interaction between two energy fields (human and environment) to strengthen the coherence and Rogers integrity of the person and to promote maximum health potential. Imogene King, on the other hand, described the nature of and standard for nurse-patient interactions that lead to goal attainment (Goal Attainment Theory). Florence Nightingale stressed the importance of utilizing the environment in assisting the patient towards recovery. Lastly, Dorothea Orem's General Theory of Nursing included three related concepts of self-care, self-care deficit, and nursing systems (Kozier). d. Imogene King

Incorrect

Marks for this submission: 0/1. Question 15 Marks: 1

Leadership is commonly defined as a process of influence in which the leader influences others toward goal achievement. Three leadership styles are still widely recognized today: the autocratic, democratic, and laissez-faire leadership. A laissez-faire leader is differentiated from other types of leadership through which of the following? Choose one answer.

In
a. The leader decides for the whole unit by himself. b. The leader leaves the decision-making up to the group. - the leader leaves the decision-making up to the group. Laissez-faire leadership is passive and permissive and the leader defers decision-making. Members may work independently and possibly at cross purposes because there is no planning or coordination and little cooperation. Chaos is likely to develop unless an informal leader emerges. Authoritarian leadership maintains strong control, does the planning, makes the decisions, and gives the orders. Meanwhile, democratic leaders maintain less control; ask questions and make suggestions rather than issue orders (Marriner-Tomey and KellyHeidenthal). c. There is less control applied in terms of handling the

subordinates.

16

Marks: 1

Republic Act No. 9173, otherwise known as the Philippine Nursing Act of 2002 was implemented to define the scope of nursing practice, licensing requirements, and standards of nursing care. The features of this Law include the following, EXCEPT: Choose one answer.

I
d. The leader acts primarily as a facilitator and a resource person. a. A member of the Board of Nursing should have at least ten (10) years of continuous practice of the nursing profession provided that the last five (5) years of which shall be in the Philippines.

b. The Board of Nursing shall - the Board of Nursing shall designate designate the places and dates of the the places and dates of the Nursing Nursing Licensure Examination. Licensure Examination. It is the responsibility of the Professional Regulation Commission (PRC) to

designate the places and dates of the licensure exam. The Board of Nursing is primarily responsible in the conduction of the licensure exam. All the other choices are features of R.A. 9173. c. The Certificate of Registration may be revoked or suspended if the nurse demonstrated unprofessional or unethical conduct.

d. A faculty in a college of nursing should have at least one (1) year of clinical practice in a field of specialization.

Incorrect Marks for this submission: 0/1. Question 17 Marks: 1 In health care delivery, basic ethical principles assist the health professionals to determine the right or wrong in regard to value issues involving the pursuit of health, alleviation of suffering, and assisting patients toward peaceful death. Explanation by the attending physician of the important findings, management, prognosis and evaluations to a newly-diagnosed breast cancer patient is an application of: Choose one answer. a. nonmalefic

ence

Incorrect

Marks for this submission: 0/1. Question 18 Marks: 1

The nursing care of older adults poses special challenges because of great variation in their physiological, cognitive, and psychosocial health. Which of the following describes GERONTICS? Choose one answer.

In
- veracity. The principle of veracity refers to the practice of telling b. veracity the truth. Autonomy refers to the right to make one's own decisions. Nurses who follow this principle should recognize that each client is unique. Non-maleficence is the duty to "do no harm". This requires nurses to act in such a way as to avoid causing harm to clients. Meanwhile, the bioethical principle of beneficence means "doing good" and it requires nurses to act in ways that benefit clients. (Balita's Ultimate Learning Guide and Kozier). c. autono my d. benefic ence

a. It is the study of all aspects of aging process and its consequences.

I
b. It considers the nursing care of older adults to be the art and practice of nurturing, caring, and comforting rather than merely the treatment of disease. - Gerontics considers the nursing care of older adults to be the art and practice of nurturing, caring, and comforting rather than merely the treatment of disease. Geriatrics is the branch of medicine that deals with the physiological and psychological aspects of aging and with diagnosis and treatment of diseases affecting older adults. Gerontological Nursing is concerned with the assessment of health and functional status of older adults; diagnosis, planning and implementing health care and services, and; evaluating the effectiveness of such care. Gerontology is the study of all aspects of aging process and its consequences (Potter & Perry). c. It is concerned with the assessment of health and functional status of older adults; diagnosis, planning and implementing health care and services, and; evaluating the effectiveness of such care. d. It is the branch of medicine that deals with the physiological and psychological aspects of aging and with diagnosis and treatment of diseases affecting older adults.

Incorrect

Marks for this submission: 0/1. Question 19 Marks: 1

Vital signs, which should be looked at in total, are checked to monitor the functions of the body. A client with end-stage renal disease is undergoing hemodialysis. He has an AV fistula on his right arm and the client's chart reads "save both arms" for a possible creation of another fistula on his left arm. When taking his blood pressure, you know that the only possible way is to take it on his thighs. Where should you place the BP cuff? Choose one answer.

I
a. Around the knee of the client b. Around his mid-thigh with the bladder over the posterior aspect of the thigh and the bottom edge around the knee - around his mid-thigh with the bladder over the posterior aspect of the thigh and the bottom edge around the knee. There are some situations wherein there is a need to take the blood pressure other than that in the arms, which is the one normally done. This position of the cuff should be done to ensure that the bladder is directly over the posterior popliteal artery if the reading is to be accurate (Kozier). c. Around the upper portion of his mid-thigh with the bladder over the posterior aspect the thigh and the bottom edge in line with the

knee.

Incorrect

Marks for this submission: 0/1. Question 20 Marks: 1

Liver biopsy is the removal of a small amount of liver tissue, usually needle aspiration. During the procedure, you instruct the client to inhale and exhale deeply several times and to hold his breath after the final exhalation for up to 10 seconds as the needle is inserted. What is the main purpose why the client needs to holds his breath during needle insertion? Choose one answer.

I
d. Around his lower thigh with the bladder over the posterior aspect of the thigh and the upper edge around the knee a. Holding the breath minimizes the pain experienced by the client as the needle is inserted. b. This immobilizes the chest wall - Holding the breath while inserting the and liver and keeps the diaphragm needle during liver biopsy immobilizes in its highest position, avoiding the chest wall and liver and keeps the injury to the lung and liver. diaphragm in its highest position, avoiding injury to the lung and liver.

Also, penetration of the diaphragm is avoided when it is in its highest position and the risk of lacerating the liver is minimized (Kozier and Brunner). The other choices are unrelated. c. For comfort purposes only

d. Holding the breath contracts the abdominal muscles so that the contour of the liver will be welldefined for easier insertion of the needle.

Incorrect Marks for this submission: 0/1. 21 Marks: 1 Clients who have been immobilized for even a few days may require assistance with ambulation. You are assisting your client while she tries to ambulate after her surgery 2 days ago. Suddenly, she feels weak and seems to be fainting, however, there is no nearby chair or wheelchair wherein you can lower the patient. You can assist the client to a horizontal position on the floor before fainting occurs. You can do the following, EXCEPT: Choose one answer. a. Assume a broad stance - Assume a broad stance with both feet parallel to with both feet parallel to each other. Assuming a broad stance is correct, each other. however one foot should be in front of the other. A broad stance widens your base of support while placing one foot behind the other allows you to rock backward and use the femoral muscles when

supporting the client's weight and lowering the center of gravity, thus preventing back strain. Meanwhile, bringing the client's weight backward against your body allows gradual movement to the floor without injury to the client (Kozier). b. Allow the client to slide down your leg, and lower the person gently to the floor.

c. Bring the client backward so that your body supports the person.

d. All of the above.

Incorrect Marks for this submission: 0/1. Question 22 Marks: 1 Nurses document evidence of the nursing process in a variety of forms throughout the clinical record. You are a nurse in the medical ward and you are assigned to a new patient for the shift. You want to know about the case of the patient and the kind of nursing care and therapeutic management already done to help her in her condition throughout her stay at the hospital. You therefore read the:

Choose one answer. a. Nursing discharge summary

b. Progress notes

c. Kardex

- Kardex. The Kardex is a widely used, concise method of organizing and recording data about a client, making information quickly accessible to all members of the health team. The Kardex reveals specific data about the client, including the therapeutic management done and a nursing care plan to be able to meet the goals and relieve the problems. The flow sheet enables nurses to record nursing data quickly and concisely to provide an easy-to-read record of the client's condition over time. Progress notes, on the other hand, provide information about the progress a client is making. Lastly, the nursing discharge summary is completed only when the client is being discharged (Kozier) .

d. Flow sheet

Incorrect Marks for this submission: 0/1. Question 23 Marks: 1

Arterial blood gases (ABGs) are performed to evaluate the client's acid-base balance and oxygenation. Your client is experiencing a prolonged, severe, asthma attack. Which of the following ABG results would you anticipate? Choose one answer. a. Decreased PaCO2, decreased PaO2, and increased pH.

b. Decreased PaCO2, increased PaO2, and decreased pH.

c. Increased PaCO2, decreased - increased PaCO2, decreased PaO2, and decreased pH. As the severe asthma attack worsens, the client becomes PaO2, and fatigued and there is the development of alveolar decreased pH. hypoventilation. This can eventually lead to carbon dioxide retention (thus increased PaCO2) and hypoxemia (decreased PaO2). The pH also decreases because the hydrogen ions are also retained as the carbon dioxide is retained (Lippincott NCLEX, Kozier). d. Increased PaCO2, increased PaO2, and increased pH.

Incorrect Marks for this submission: 0/1. Question 24 Marks: 1 Nursing practice is governed by many legal concepts and it is important for nurses to know the basics of legal concepts, because nurses are accountable for their professional judgments and actions. You inserted a Foley catheter to a patient with urinary retention even though the patient refused to. You informed the patient that this will benefit her. Even if you have a good intention, you are liable for: Choose one answer. a. battery - battery. Battery is the willful touching of a person (or the person's clothes or even something the person is carrying) that may or may not cause harm. It may be actionable by law if the touching is done in a wrong way, such as touching without permission or consent. Assault, on the other hand, can be described as an attempt or threat to touch another person unjustifiably. Assault precedes battery; it is the act that causes the person to believe a battery is about to occur. Negligence and malpractice are examples of unintentional torts (Kozier).

b. invasion of privacy

c. assault

d. unintenti onal tort

Incorrect

Marks for this submission: 0/1. Question 25 Marks: 1

Negligence is defined as a misconduct or practice that is below the standard expected of an ordinary, reasonable, and prudent person. Which of the following situations will most likely cause the nurse to be sued for negligence? Choose one answer.

In
a. Nurse Pat gave the wrong antibiotic and after 30 minutes the patient experienced an anaphylactic reaction. - Nurse Pat gave the wrong antibiotic and after 30 minutes the patient experienced an anaphylactic reaction. An act of negligence can be seen if the following are present: a) duty - the nurse must have (or should have had) a relationship with the client that involves providing care and following an acceptable standard of care; b) breach of duty - there must be a standard of care that is expected in the specific situation but that the nurse did not observe; c) foreseeability - a link must exist between the nurse's act and the injury suffered; d) causation - it must be proved that the harm occurred as a direct result of the nurse's failure to meet standard of care, and; e) injury - the client must demonstrate some type of harm or injury (physical, financial, or emotional) (Kozier). b. Nurse Anne noticed that she aspirated 1 ml of the medication instead of the prescribed 0.5 ml in a syringe.

Incorrect

Marks for this submission: 0/1. 26

Marks: 1

Electrolytes, charged ions capable of conducting electricity, are present in all body fluids and fluid compartments. When you are assessing your patient, you noticed the following signs and symptoms: numbness, tingling of the extremities and around the mouth, muscle tremors, cramps and hyperactive deep tendon reflexes. Which of the following electrolytes is most likely to be imbalanced in your patient? Choose one answer. a. Magn esium

I
c. Nurse Cecille is about to administer an oral medication when the patient complained that it is not the one usually given to her. d. Nurse Abby mixed a medication in the wrong kind of IV fluid during preparation in the medication room.

Incorrect

Marks for this submission: 0/1. Question 27 Marks: 1

The level of consciousness, which is a component of the neurologic examination, can be measured using the Glasgow Coma Scale (GCS). Marie, a 45-year-old woman, was brought in at the Emergency Department after a vehicular accident. She opens her eyes to verbal command, moves to localized pain and makes sounds that are incomprehensible. What will be her score on the GCS that you, as her

In
b. Potas sium c. Sodiu m

d. Calci Calcium. Numbness, tingling of the extremities and around the mouth, muscle tremors, cramps and hyperactive deep tendon um reflexes, with positive Trousseau's and Chvostek's signs and cardiac dysrhythmias are manifestations of hypocalcemia or decreased serum calcium. Clients at greatest risk for hypocalcemia are those whose parathyroid glands have been removed. Hypomagnesemia and chronic alcoholism also increase the risk of hypocalcemia (Kozier).

nurse, will document on her chart? Choose one answer. a. 11

- 10. The Glasgow Coma Scale tests three (3) major areas that determines the client's level of consciousness: eye opening b. (spontaneous-4, to verbal command-3, to pain-2, no response-1), motor 10 response (to verbal command-6, to localized pain-5, flexes and withdraws-4, flexes abnormally-3, extends abnormally-2, no response-1) and verbal response (oriented and converses-5, disoriented and converses-4, uses inappropriate words-3, makes incomprehensible sounds-2 and no response-1). The higher the score, the more improved or normal the level of functioning. An assessment totaling 15 points indicates that the client is alert and completely oriented. A comatose patient scores 7 or less (Kozier and Potter & Perry) c. 8

d. 9

Incorrect Marks for this submission: 0/1. Question 28 Marks: 1

Total parenteral nutrition (TPN) affords the provision of energy and nutrients intravenously and it is an important responsibility of the nurse to monitor the patient regularly for possible complications related to this therapeutic management. You are regularly assessing your client who is receiving TPN for signs of hyperglycemia. You will note which of the following if your client is already hyperglycemic? Choose one answer. a. Fever, nausea, vomiting

b. Nausea, vomiting and chills

c. Cold clammy skin, sweating and weakness

d. Thirst, increased urine output and warm flushed skin

thirst, increased urine output and warm flushed skin. These are the main symptoms of hyperglycemia. The most common metabolic complication in TPN is hyperglycemia. Metabolic complications in TPN is most common because metabolic requirements (electrolytes and energy) differ from patient to patient. Hyperglycemia can be treated by adding insulin to the solution, reducing the dextrose load, or ensuring the total kcaloric load is not excessive (Goodner). TIP: If it's hot and dry, the sugar is high. Cold and clammy, give some candy.

Incorrect

Marks for this submission: 0/1. Question 29 Marks: 1 When using an instrument or tool in quantitative research, it is important to ensure its reliability and validity. Reliability is the consistency with which it measures the target attribute, while validity refers to the degree to which an instrument measures what it is supposed to measure. Harry, a nurse researcher, has found an instrument to measure the level of self-esteem of school-age children. He wants to test its validity so he consults a panel of experts to evaluate the validity of the instrument. This is called: Choose one answer. a. None of the above

b. content validity

- content validity. Content validity concerns the degree to which an instrument has an appropriate sample of items for construct being measured. This kind of validity is necessarily based on judgment and it is becoming increasingly common to use a panel of substantive experts to evaluate and document the content validity of instruments. Face validity, on the other hand, refers to whether the instrument looks as though it is measuring the appropriate construct (Polit & Beck).

c. face validity

d. Both content and face validity

Incorrect

Marks for this submission: 0/1. Question 30 Marks: 1

Rhonchi (gurgles) are continuous, low-pitched, coarse, gurgling, harsh, louder sounds with a moaning or snoring quality. A nurse understands that the cause of this adventitious breath sounds is: Choose one answer.

In
a. Air passing through - The primary cause of rhonchi (gurgles) is the narrowed air passages as a passage of air through narrowed air passages result of secretions, as a result of secretions, swelling, and tumors. It swelling, and tumors. can be best heard during expiration but can be heard on both inspiration and expiration. Crackles (rales) and crackles are caused by air passing through fluid or mucus in any air passage. It can be described as fine, short, interrupted crackling sounds that are best heard on inspiration. Wheezes are caused by air passing through a constricted bronchus as a result of secretions, swelling, and tumors, which can be described as continuous, high-pitched, squeaky musical sounds and are best heard on expiration. Meanwhile, Rubbing together of inflamed pleural surfaces is the cause of pleural friction rubs which are superficial grating or creaking sounds that can be heard during inspiration and expiration (Kozier). b. Rubbing together of inflamed pleural surfaces.

c. Air passing through fluid or mucus in any air passage.

Incorrect

Marks for this submission: 0/1. 31

Marks: 1

Because nursing research usually focuses on humans, a major nursing responsibility is to be aware of and to advocate on behalf of client's rights. Which of the following statements best describe the right to self-determination in research? Choose one answer.

I
d. Air passing through a constricted bronchus as a result of secretions, swelling, and tumors. a. assurance of the anonymity of a study participant b. avoidance of any exposure to the possibility of injury beyond everyday

situations

Incorrect

Marks for this submission: 0/1. Question 32 Marks: 1

The nurse-manager decides what task should be done, when, where and by whom. This can be done through delegation. Which of the following is NOT a reason for delegating?

I
d. provision of complete information about the research/study

- freedom from constraints, or any undue influences to participate in a study. The right to self-determination means that prospective participants have the right to decide voluntarily whether or not to participate in a study, without c. freedom from risking any penalty or prejudicial treatment. The constraints, or any undue right not to be harmed refers to avoidance of any influences to participate exposure to the possibility of injury beyond in a study everyday situations. This involves avoidance of any risk that can be physical, emotional, legal, financial, or social. The right to privacy and confidentiality is wherein participants have the right to expect that any data they provide will be kept in strictest confidence. The right to full disclosure is wherein the researcher should fully describe the nature of the study, the person's right to refuse treatment, the researcher's responsibilities, and likely risks and benefits (Kozier and Polit & Beck).

Choose one answer. a. Delegation reduces managerial costs.

b. Delegation maximizes the use of talents of staff associates.

c. Delegation is an opportunity to transfer one's accountability to a task.

- Delegation is an opportunity to transfer one's accountability to a task. Accountability is still retained by the nurse-manager even if tasks are delegated. They are still accountable for the performance of the task, the selection of the person to complete it, and both the staff's and their own performance (Tomey). When delegating, authority is transferred, responsibility shared and accountability retained.

d. Delegation saves time and can help develop others.

Incorrect Marks for this submission: 0/1. Question 33 Marks: 1 Cardiopulmonary resuscitation (CPR) is a combination of oral

resuscitation (mouth-to-mouth breathing), which supplies oxygen to the lungs, and external cardiac massage (chest compressions), which is intended to reestablish cardiac function and blood circulation. Based on the latest guidelines in CPR, what is the compression-to-ventilation ratio for all clients except that of newborns? Choose one answer. a. 10 compressio ns:2 breaths

b. 15 compressio ns:2 breaths

c. 5 compressio ns:1 breath

d. 30 compressio - 30 compressions:2 breaths. Based on the latest guidelines of ns:2 the American Heart Association (2005), this is changed from breaths 15:2 (adults) and 5:1 (child and infant) to simplify training and to ensure a longer series of uninterrupted chest compressions. This is also changed because of the rationale that blood flow to the lungs is less than normal during CPR, therefore the victim needs less ventilation than normal (Kozier).

Incorrect Marks for this submission: 0/1. Question 34 Marks: 1 Nursing care oriented to health promotion, wellness, and illness prevention can be understood in terms of health activities on the different levels of preventive care. A client is a known hypertensive for 5 years. She has been taught at the health center about the possible complications of the disease if it is not managed properly and this includes stroke. She has modified her lifestyle since then by quitting smoking, exercising regularly and by eating a balanced diet. These activities demonstrate: Choose one answer. a. Tertiary preventio n - tertiary prevention. Tertiary prevention involves minimizing the effects of long-term disease or disability by interventions directed at preventing complications and deterioration. Primary prevention, meanwhile, is aimed at health promotion that includes health education programs, immunization and physical and nutritional fitness programs. Secondary prevention, on the other hand, focuses on individuals who are experiencing health problems or illnesses and includes screening techniques and treating early stages of disease to limit disability by preventing or delaying the consequences of advanced disease (Potter & Perry).

b. Secondar y preventio n

c. Primary preventio n

Incorrect

Marks for this submission: 0/1. Question 35 Marks: 1

An enema is a solution introduced into the rectum and the large intestine to distend the intestine and sometimes to irritate the intestinal mucosa, thereby increasing peristalsis and the excretion of feces and flatus. Which of the following clients is more likely to receive a retention enema? Choose one answer.

In
d. None of the choices a. A 45-year-old man who will undergo colonoscopy on the following day b. A 5-year-old child with ascariasis - A 5-year-old child with ascariasis. A retention enema introduces oil or medication into the rectum and sigmoid colon. The liquid is retained for a relatively long period (e.g. 1 to 3 hours). Antihelminthics can be given through enema to kill helminths such as worms and intestinal parasites. Those who will undergo surgery or have an invasive

diagnostic procedure (e.g., colonoscopy) are more likely to receive a cleansing enema and this kind of enema is intended to remove feces. Cleansing enemas are given chiefly to prevent the escape of feces during surgery; prepare the intestine for certain diagnostic tests such as x-ray or visualization tests, and; remove feces in instances of constipation or impaction. Other types of enema include carminative (to expel flatus) and return-flow (used occasionally to expel flatus and is repeated several times until it is expelled) enemas (Kozier). c. A client experiencing excessive flatus.

d. A 28-year-old female client to undergo Caesarian section the next day

Incorrect Marks for this submission: 0/1. 36 Marks: 1 Several positions are frequently required during the physical assessment, however it is important to consider the client's ability to assume a position as well as their physical condition, energy level, and age. Mrs. AF, 65 years old, is diagnosed to have cervical cancer for 5 years already, however, there are no co-morbidities present. She is in the clinic for a follow-up and the nurse is to assess her genital area. What position should the nurse ask Mrs. AF to assume? Choose one answer.

a. Dorsal recumb ent

- dorsal recumbent position. This position, together with lithotomy, are the most common positions to assume when assessing the female genitals, rectum and the female reproductive tract. However, the lithotomy position may be uncomfortable and tiring for elders and often embarrassing. The dorsal recumbent position is contraindicated only to those with cardiopulmonary problems. The prone position is often not tolerated by elders and the sim position may be difficult for them because of limited joint movement (Kozier).

b. Lithoto my

c. Sims

d. Prone

Incorrect Marks for this submission: 0/1. Question 37 Marks: 1 Because of their unique position in the health care system, nurses experience conflicts among their loyalties and obligations to clients, families, primary care providers, employing institutions, and licensing bodies. The attending physician of your patient placed an order in the chart to administer a pain medication STAT. However, when you arrived in your patient's room, he seems to not be in pain. What will

be your action? Choose one answer. a. Wait until the patient experiences pain again then administer the medication.

b. Withhold the medication, further assess the patient and - withhold the medication, further assess the patient and inform the attending physician of inform the attending your findings. According to the nurses code physician of your findings. of ethics, the nurse's first loyalty is to the client. The nurse's actions should always give the highest priority to the client's needs before that of other members of the health team (Kozier). c. Administer the medication anyway because you might get reprimanded by the physician.

d. Inform the charge nurse of the condition.

Incorrect Marks for this submission: 0/1. Question 38 Marks: 1

Informed consent is an agreement by a client to accept a course of treatment or a procedure after being provided complete information, including the benefits and risks of treatment, alternatives to the treatment, and prognosis if not treated by a health care provider. An informed consent needs to be acquired for the performance of endoscopy for a 17-year-old male adolescent, however, there are no other significant others present during that time. A nurse should remember that the patient: Choose one answer. a. Is able to give voluntary consent when his parents are not available

b. Will most likely be unable to choose between alternatives when asked to consent

c. Is not able to make an acceptable or intelligent choice

d. Does not have the legal capacity to give consent

- does not have the legal capacity to give consent. An individual is legally unable to sign a consent until the age of 18 years. The only exception wherein a minor is allowed to sign a consent is when he/she is considered an emancipated minor - a minor who is self-sufficient or married. Other kinds of people who cannot sign an informed

consent are those who are unconscious or injured in such a way that they are unable to give consent, and those who are mentally ill persons who have been judged by professionals to be incompetent (Kozier and Mosby NCLEX) Incorrect Marks for this submission: 0/1. Question 39 Marks: 1 Thoracentesis is used to remove the excess fluid or air to ease breathing. Your client is to undergo thoracentesis and you assist him to assume a position that is indicated for the procedure. Which of these can be his position during the procedure? 1 Sitting position with the arms above the head 2 Sitting position with the arm elevated and stretched forward 3 Sitting position in which the client leans over a pillow or overbed table 4 Sitting position with both arms crossed in front of the chest Choose one answer. a. All of the above

b. 3 and 4

c. 2, 3 and 4

Incorrect

Marks for this submission: 0/1. Question 40 Marks: 1

Determining the different variables in research is an important task of the researcher. In the research question, "What is the effect of progressive muscle relaxation on the blood pressure of hypertensive residents in an urban community in Metro Manila?", which is the independent variable? Choose one answer. a. hypertensive residents

In
d. 1, 2 and 3 The client undergoing thoracentesis can assume the sitting position with his arms above the head, with his arm elevated and stretched forward and in which he leans over a pillow or overbed table. These positions allow easy access to the intercostal spaces wherein the needle will be inserted to remove the excess fluid or air (Kozier). b. urban community in Metro Manila

c. blood pressure

Incorrect

Marks for this submission: 0/1. 41

Marks: 1

Urinary incontinence, or involuntary urination is a symptom, not a disease. It can have a significant impact on the client's life, creating physical problems such as skin breakdown and possibly leading to psychosocial problems such as embarrassment, isolation, and social withdrawal. Your client, Melanie, is 8 months pregnant and she reports dribbling of urine whenever she laughs, coughs, or sneezes. What will be your appropriate nursing diagnosis and intervention for her? Choose one answer.

In
d. progressive muscle relaxation - progressive muscle relaxation. The independent variable is the presumed cause of or influence on the dependent variable. The dependent variable, on the other hand, is the behavior, characteristic, or outcome that the researcher wishes to explain or predict. In the statement above, the blood pressure is the dependent variable (Kozier). a. Urge urinary incontinence increase bladder's ability to hold urine and the client's ability to suppress urination. b. Reflex urinary incontinence regular periodic use of a catheter

to empty the bladder

I
c. Stress urinary incontinence encourage pelvic muscle exercises - stress urinary incontinence - encourage pelvic muscle exercises. Stress urinary incontinence is the sudden loss of urine occurring with activities that increase abdominal pressure, such as that in pregnancy. The main goal for this kind of incontinence is symptom control that may be done through strengthening and training the levator ani and urogenital muscles through repetitive contractions to decrease stress, urge or mixed types of incontinence (pelvic muscle exercises). Functional incontinence is the inability of the usually continent person to reach toilet in time to avoid unintentional loss of urine and this can be avoided through prompted voiding . Reflex incontinence, on the other hand, is the involuntary loss of urine at somewhat predictable intervals when a specific bladder volume is reached and this can be managed through intermittent urinary catheterization. Urge incontinence is the involuntary passage of urine occurring after a strong sense of urgency to void and the best way to manage this is through urinary bladder training (Kozier). d. Functional urinary incontinence - encourage timed verbal toileting reminders and positive social feedback for successful toileting

Incorrect Marks for this submission: 0/1. Question 42 Marks: 1 Peplau's theory involves the use of a therapeutic relationship between the nurse and the client. When the client assumes the dependent client role, the nurse and client are on which phase of the therapeutic relationship? Choose one answer. a. Resolut ion

b. Orienta tion

c. Identifi - identification. During the identification phase, the client assumes a cation position of dependence, interdependence, or independence in relation to the nurse. When the client assumes this role, he/she is completely dependent on his/her caregiver until he/she derives understanding of his/her condition, which is achieved in the exploitation phase. New needs and goals are adopted during the resolution phase (Kozier). d. Exploit ation

Incorrect

Marks for this submission: 0/1. Question 43 Marks: 1

One of the most potentially hazardous procedures that health care personnel face is using and disposing of needles and sharps. Needlestick injuries can be prevented if these guidelines are followed, EXCEPT: Choose one answer.

In
a. Never bend or break needles before disposal. b. When recapping a needle, use a one-handed "scoop" method. c. Recap used needles. - recap used needles. Used needles (i.e. has been inserted into a client) should NEVER be recapped except under special circumstances (e.g. when transporting a syringe to the laboratory for an arterial blood gas or blood culture). Recapping a needle is also done only after drawing up a medication into a syringe prior to administration (hence, use the onehanded "scoop" method). The other 2 choices are other essential guidelines to prevent

needlestick injuries (Kozier). d. Use appropriate puncture-proof disposal containers to dispose of uncapped needles and sharps.

Incorrect Marks for this submission: 0/1. Question 44 Marks: 1 R.A. 9173 (The Philippine Nursing Act of 2002)defines the scope of nursing practice in the Philippines. Which of the following statements is NOT included in the scope of nursing? Choose one answer. a. A community health nurse teaching a group of hypertensive clients about regular monitoring of blood pressure.

b. A nurse performing internal examination of a woman during labor who experienced minimal vaginal bleeding during the second trimester.

- a nurse performing internal examination of a woman during labor who experienced minimal vaginal bleeding during the second trimester. This is not included in the scope of nursing as defined by R.A. 9173 because the law states that a nurse is only to do internal examinations during labor provided that there is an absence of antenatal bleeding and delivery. All the other choices are correct: a nurse can conduct essential

health teachings, administration of written prescription for medications and executing comfort measures (Bellosillo, et al.) c. A nurse administering an IV antibiotic after being ordered by the physician.

d. A nurse providing oral hygiene to a bed-ridden patient.

Incorrect Marks for this submission: 0/1. Question 45 Marks: 1 Preoperative teaching is a vital part of nursing care to reduce clients' anxiety and postoperative complications and increases their satisfaction with the surgical experience. Which of the following statements by the client indicates that preoperative teaching regarding gallbladder surgery has been effective? Choose one answer. a. "After surgery, I can immediately resume the diet I had before I was operated."

b. "I am not going to cough after surgery because if I do, my wound will tear apart."

- "I cannot eat or drink anything at least 8 hours before my surgery." There is a need to restrict food and oral fluids (NPO) at least 8 hours before surgery to prevent aspiration during the c. "I cannot eat or drink operation. Usually, dietary alterations are anything at least 8 hours necessary post-operatively because some foods are not yet tolerated immediately after surgery. before my surgery." Deep breathing and coughing exercises post-op are needed to enhance lung expansion and mobilize secretions, thereby preventing atelectasis and pneumonia. The client should be taught how to splint his/her incision to reduce pain while coughing if the incision is near any of the breathing muscles (ie, diagphragm). Clients should also be encouraged to carry out deep breathing and coughing exercises at least every 2 hours. (Kozier). d. "I will only do deep breathing exercises when I am experiencing pain."

Incorrect Marks for this submission: 0/1. 46 Marks: 1 Confidentiality is the protection of the participants in a study such as that individual identities are not linked to information provided and

are never publicly divulged. Which of the following statements best manifest confidentiality? Choose one answer. a. The research study was done in the pediatric wards of a tertiary hospital in Manila. - the research study was done in the pediatric wards of a tertiary hospital in Manila. This statement best manifests confidentiality as there were no specific information given to link the individual identities. Measures to be included to ensure confidentiality is the use of pseudonyms, code numbers or reporting only aggregate or group data in published research ((Kozier and Polit & Beck).

b. The respondents are employees of the only private hospital in the municipality of San Juan in Batangas.

c. The patient is L.C.R., 34, female of 5 Jupiter Street, Makati.

d. none of the above

Incorrect Marks for this submission: 0/1. Question 47

Marks: 1 Evidence-based practice is the conscientious, explicit, and judicious use of current best evidence in making decisions about the care of individual patients. You are a nurse-manager in the pay ward who uses evidence-based practice in planning care for your clients. Which of the following is your priority in planning for these clients? Choose one answer. a. Care plans are individualized according to the client's needs.

b. Care plans follow the standards of care established by the institution's nursing service.

c. Care plans that are standardized are used on all patients.

d. Care plans are based on the nurse's clinical experience and from the latest research findings.

- care plans are based on the nurse's clinical experience and from the latest research findings. Evidenced-based practice uses outcome research and other current research findings to guide the development of appropriate strategies to deliver quality, costeffective care. Research provides evidence about benefits, risks, and results of treatments so individuals can make informed decisions and

choices to improve their quality of life (KellyHeidenthal). Incorrect Marks for this submission: 0/1. Question 48 Marks: 1 It is important that nurses make nursing diagnoses with a high level of accuracy. Which of these guidelines should be followed when writing a nursing diagnostic statement? Choose one answer. a. Use medical terminology rather than nursing terminology to describe the probable cause of the client's response

b. Use statements based on nurse's perception of the client's response

c. Word the statement so that - word the statement so that it is legally it is legally advisable advisable. The statement should not imply that there is legal liability in the part of the health care team. Nursing terminology should be used rather than medical terminology to describe the probable cause of the client's response. The diagnosis should be specific and precise to provide direction for planning the nursing intervention. Nursing diagnostic statements should not be

judgmental, but instead objective (Kozier). d. Word the diagnosis as generally as possible

Incorrect Marks for this submission: 0/1. Question 49 Marks: 1 Planning is a deliberative, systematic phase of the nursing process that involves decision making and problem solving. This phase includes establishing client goals and objectives. Which of the following objectives for the nursing diagnosis of Ineffective Airway Clearance related to poor cough effort is NOT properly stated? Choose one answer. a. The client will not experience aspiration. - the client will not experience aspiration. This statement is more of a goal because goals are broad statements about the client's status while objectives are more specific, observable criteria used to evaluate whether the goals have been met. Also, objectives should be measurable and time-bound (Kozier).

b. The client will show no signs of pallor or cyanosis by 12 hours post-surgery.

c. The client will demonstrate effective coughing and increased air exchange within 24 hours after surgery.

Incorrect

Marks for this submission: 0/1. Question 50 Marks: 1

During physical assessment of the skin, the nurse may palpate it to locate skin lesions, which are any pathological skin changes. During palpation of the skin, the nurse assesses skin lesions, which she documented in the client's chart as a nodule. Which of the following describes a nodule? Choose one answer.

I
d. The client will have lungs clear upon auscultation during entire postoperative period. a. Irregularly shaped, elevated area or superficial localized edema that varies in size. b. Solid mass that may extend deep through the subcutaneous tissue, larger than 1-2 cm.

Incorrect

Marks for this submission: 0/1.

Marks: 1

Prostatitis is the inflammation of the prostate gland and is a common problem. Acute bacterial prostatitis causes severe localized prostatic

I
c. A palpable, circumscribed, solid elevation in the skin, smaller than 0.5 cm.

- A nodule is an elevated solid mass, deeper and firmer than a papule, that is 0.5-2 cm. A common d. An elevated solid mass, example of a nodule is a wart. A wheal is an deeper and firmer than a irregularly shaped, elevated area or superficial papule, that is 0.5-2 cm. localized edema that varies in size and common examples of which are hives and mosquito bites. A papule is a palpable, circumscribed, solid elevation in the skin, smaller than 0.5 cm and an elevated nevus is an example of which. A tumor is a solid mass that may extend deep through the subcutaneous tissue, larger than 1-2 cm and a common example of which is an epithelioma (Potter and Perry).

inflammation that may interfere with the client's ability to void. Most people also experience systemic illness. Which of the following is NOT necessarily at risk for acute bacterial prostatitis? Choose one answer. a. A 25 year old gay man who has had his first unprotected penetrative anal intercourse. b. A 31 year old single man with who has had a cystoscopy. c. A 27 year old single man with gonorrhea. d. A 29 year old man with priapism. Question 2 Marks: 1 Hemodialysis is used for clients with acute or irreversible renal failure and fluid and electrolyte imbalances. The following are among the overall therapeutic effects of hemodialysis Except: Choose one answer. a. Restore fluid, electrolyte, and acid-base balances b. Reverse some of the untoward manifestations of irreversible renal failure. c. Clear waste products from the body d. Provide hydration and additional blood supply to the systemic circulation Question 3 Marks: 1 Infective endocarditis refers to a microbial infection involving the endocardium. The following are considered the most reliable criteria for diagnosis of endocarditis EXCEPT: Choose one answer. a. Positive blood cultures b. A new regurgitant murmur c. Evidence of endocardial involvement by echocardiography. d. Elevated cardiac enzymes CK-MB and serum markers troponin and myoglobin

Question 4 Marks: 1 Compartment syndrome is a condition of compromised circulation related to progressively increasing pressure in a confined space. Your patient, Rod, broke his ulna due to a bad fall during a soccer game. Which of the following statements made by Rod would alert you to the possible presence of compartment syndrome? Choose one answer. a. "I hear voices from my cast saying I should wash it." b. "I feel a burning sensation on my wrist." c. "I can smell a fishy odor coming from my cast." d. "I can't feel my fingers." Question 5 Marks: 1 The visual pathway is a multidimensional system with many structures and processes subject to trauma or disorders. You are working at a cardiology clinic when a woman in her late 50s was accompanying her husband. While waiting for her husband to be called, the woman approached you and said, "Nurse, what do you think is happening to me? It started as a burst of black spots, and now I have been seeing something like a curtain that falls across my vision. But I don't feel any pain, so I don't think it's really anything serious." You advise it would be best to consult an ophthalmologist as soon as possible because you think most likely, she is having/experiencing: Choose one answer. a. Diabetic retinopathy b. Retinal detachment c. Cataract d. Open angle glaucoma Question 6 Marks: 1 In a lumbar puncture (LP), also known as spinal tap, a needle is inserted into the subarachnoid space in the lumbar region of the spine

below the level of the spinal cord. Cerebrospinal fluid (CSF) can be withdrawn or substances may be injected into this space. Your patient is post-LP. Which of the following is NOT helpful? Choose one answer. a. Encourage fluid intake. b. Instruct client to ambulate as soon as possible to improve circulation of CSF. c. If a spinal headache continues, anticipate the need for an epidural blood patch. d. If a spinal headache develops, put client in a dark, quiet room. Question 7 Marks: 1 Thyroid storm (thyrotoxicosis) is a potentially fatal acute episode of thyroid activity. You work in a charity ward and you encounter patients who cannot buy their maintenance medications continuously. Which symptoms indicative of thyroid storm should you regularly assess and watch out for? Choose one answer. a. Extreme irritability and loss of ocular muscle control b. High fever and severe tachycardia c. Both A and B d. Both A and C e. Delirium and dehydration Question 8 Marks: 1 Arthritis is generally the inflammation of the joints that causes pain and swelling. There are four types of arthritis. This kind is chronic, systemic autoimmune disorder whose major distinctive feature is chronic, symmetrical and erosive inflammation of the synovial tissue of joints. Choose one answer. a. Osteoarthritis

b. Gouty arthritis c. Rheumatoid arthritis d. Juvenile arthritis Question 9 Marks: 1 One of the most important aspects of providing nursing care to the HIV-infected client is helping with the compliance of the antiretroviral regimen. Your patient, Dave, is taking antiretroviral medications. Dave should take note of the following correct instructions EXCEPT: Choose one answer. a. Take the drug anytime you feel symptoms. b. Do not skip a dose. c. If you take the drug only periodically, it would be better not to take them at all. This matter should be discussed with the physician. d. Take the drug at prescribed intervals. Question 10 Marks: 1 Nephrolithiasis is a condition in which one or more kidney stones are present in the urinary system. Among the nursing interventions is ensuring a correct diet for the patient. Agnes has a history of uric acid kidney stones. Which food should be avoided by the client on a purine-restricted diet? Choose one answer. a. Tapioca b. Peanut butter c. Milk d. Liver Question 11 Marks: 1 In emphysema, the impaired gas exchange of oxygen and carbon

dioxide results from destruction of walls of overdistended alveoli. You are a nurse in a medicine ward taking care of Amy, admitted for emphysema. The doctor ordered oxygen at 2 liters per minute via nasal cannula. When administering the oxygen to this patient, the nurse should observe closely for: Choose one answer. a. Hemoptysis and low-grade fever b. Hyperemia and increased respirations c. Anxiety and diaphoresis d. Drowsiness and decreased respirations Question 12 Marks: 1 With advanced age, the body does not respond as vigorously to illness or disease because of the diminished physiological reserve. Which of the following is a structural and functional change in the respiratory system of the older adult? Choose one answer. a. Increased vital capacity b. Decreased rigidity of the thoracic cage c. Increased cough efficiency d. Increased residual volume Question 13 Marks: 1 Injuries that result from direct contact with or exposure to any thermal, chemical, or radiation source are termed burns. You are a nurse rotating at the hospital's burn unit. Bed 4, Mario, is a third year student nurse who was part of a burn accident. He asked you what the classification of his burns are, pointing to a part of his skin that have variable colors ranging from deep red, white, black and brown, has a dry surface, has visible thrombosed vessels, is not blanchable, and is not painful. You answer him that this is a: Choose one answer. a. Fourth degree full thickness burn

b. Third degree full thickness burns c. Third degree partial thickness burn d. Second degree partial thickness burn Question 14 Marks: 1 Third degree atrioventricular (AV) block occurs when no atrial pulse is conducted through the AV node into the ventricles. You are assigned to a patient with third degree AV block. Your priority nursing problem will most likely be: Choose one answer. a. Activity intolerance b. Decreased cardiac output c. Anxiety d. Ineffective tissue perfusion Question 15 Marks: 1 Medications used in the treatment of hyperthyroid conditions are used to reduce thyroid hormone levels. This drug offers a more permanent option for patients with hyperthyroidism because it destroys thyroid tissue. Choose one answer. a. Radioactive Iodine (131 I) b. Levothyroxine (Synthroid) c. Propylthiouracil or PTU d. Methimazole (Topazole) Question 16 Marks: 1 Ovarian cancer, although not the highest in incidence among reproductive tumors, is the leading cause of death from genital reproductive malignancies, and accounts for about 4% of all cancer

deaths in women. Which of the following is least likely to be at risk for ovarian cancer? Choose one answer. a. A graduating medical intern who uses oral contraceptives to prevent pregnancy. b. A nun who has been celibate all her life. c. A fitness instructor who has a grandmother who died of hereditary nonpolyposis colorectal cancer (HNPCC). d. A married woman who has trouble conceiving and uses ovulationstimulating medications. Question 17 Marks: 1 Seizure is defined as a sudden attack of a condition, as in a convulsion. Sandra is one of your patients in the community who has a seizure disorder. She takes maintenance drugs phenytoin (Dilantin) and carbamazepine (Tegretol). She asks you about getting married with her boyfriend and having children. Which of the following would be your most appropriate response? Choose one answer. a. "You should wait until your seizures are cured before you consider having children." b. "You will have a hard time conceiving because of your meds. You should stop having birth control 6 months prior to actually conceive." c. "Your children would not necessarily have a seizure disorder. Just make sure you go to prenatal checkups regularly." d. "When you decide to have children, ask your doctor about changing your medication." Question 18 Marks: 1 Amnesia means loss of memory. This can be a manifestation or a complication in brain injury. Memory loss associated with events prior to the time of injury is referred to as Choose one answer.

a. Pre-retrograde amnesia b. Traumatic amnesia c. Antegrade amnesia d. Retrograde amnesia Question 19 Marks: 1 Antihypertensive drugs, which act to reduce blood pressure, are used to treat hypertension, a disorder characterized by elevation in systolic, diastolic, or both blood pressures. Treatment for hypertension typically begins with a diuretic or a calcium channel blocker. Which of the following drugs is neither a diuretic nor a calcium channel blocker? Choose one answer. a. Indapamide b. Imipramine c. Amlodipine d. Verapamil Question 20 Marks: 1 Allergies are among the most common disorders seen in the medical community. The client often requires a combination of treatments ranging from avoidance of allergens to immunotherapy. You are educating Andrea's mother about controlling the environment. The following are correct tips on desensitizing a room EXCEPT: Choose one answer. a. Using wallpaper on walls instead of paint. b. Toys should be wood, metal, or plastic. c. Using rubberized canvas or plastic upholstered furniture instead of fabric upholstery. d. Using instant roll-up washable cotton or synthetic window shades instead of Venetian blinds. Question 21

Marks: 1 Mucositis is inflammation with erythema or ulceration of the mucous membrane resulting from cancer therapy. You are the primary nurse of Helena, who was diagnosed with acute myeloid leukemia and was undergoing chemotherapy. You identified Knowledge Deficit: Self-Care as one of your nursing problems. After nursing intervention, which of the following statements, if stated by the client, would indicate the need for further teaching? Choose one answer. a. "Toothbrushing can be dangerous. I will use lemon-glycerine swabs instead." b. "I will use a soft bristled toothbrush for oral hygiene." c. "I will carefully cleanse my perirectal area after each bowel movement." d. "The doctor prescribed meds to prevent yeast infection. I should take them as prescribed." Question 22 Marks: 1 The eyelids, lacrimal gland, and conjunctiva are also important to include in assessment. The following disorders/symptoms involve the need for artificial tears in their management, EXCEPT: Choose one answer. a. Absence of blinking b. Chalazion c. Lagophthalmos d. Ptosis Question 23 Marks: 1 Tuberculosis (TB) is considered as one of the world's deadliest disease and remains as a major public health problem in the Philippines. A mother has an active TB and you are a community health nurse giving health teaching to her and her family. Which statement made by the family best indicates understanding of your teachings? Choose one answer.

a. "She should be kept in a room for as long as the disease is active" b. "She can't cook our meals" c. "We all need to wash our hands carefully, but especially our mother" d. "We won't share clothes with mother." Question 24 Marks: 1 Hearing impairment ranges from minor difficulty in understanding words or hearing certain sounds to total deafness. This type of hearing loss is described as loss of hearing for which no organic reason can be found. Choose one answer. a. Functional hearing loss b. Fluctuating hearing loss c. Neural hearing loss d. Mixed hearing loss Question 25 Marks: 1 Angiography is an x-ray examination using contrast dye to visualize patency of the arteries. You just received your patient from the cardiac catheterization lab and is status post angiography. The following are part of your nursing care EXCEPT: Choose one answer. a. Assess the circulatory status of the extremities. b. Encourage immediate ambulation. c. Maintain pressure over site. d. Check the injection site for bleeding and inflammation. Question 26 Marks: 1 Intradermal testing is the most accurate skin test but is linked to a higher incidence of severe allergic reactions. You administered a skin

test of Cefuroxime on Peter. You conclude an immediate reaction. You therefore assessed that: Choose one answer. a. Reaction appeared within 20-30 minutes after injection, marked by vesicle formation greater than 3 mm of the positive control. b. Reaction appeared in less than 5 minutes after injection, marked by patch formation greater than 5 mm of the positive control. c. Reaction appeared within 10-20 minutes after injection, marked by bulla formation greater than 5 mm of the positive control. d. Reaction appeared within 10-20 minutes after injection, marked by erythema and wheal formation greater than 3 mm of the positive control. Question 27 Marks: 1 A colostomy is a procedure that involves creating an opening between the colon and abdominal wall, from which fecal contents will pass. You are a nurse in the surgical ward and you are preparing Bart for discharge. You just gave your health teachings for Bart, who just has had a colostomy. You are now going to evaluate what he learned. Which of the following statements made by Bart is correct? Choose one answer. a. "The stoma should be beefy red, as this indicates good blood supply." b. "The stoma should always be dry, since moisture can make me at risk for infection." c. "I should only be alarmed if bleeding is more than 1000ml." d. "Excessive flatus will be present in about 1 week." Question 28 Marks: 1 Wound dehiscence and evisceration are surgical complications, and are especially serious when they involve abdominal incisions or wounds. When this occurs, the patient is instructed to lie quietly in this position: Choose one answer. a. Supine, to even out the flesh.

b. Prone, to provide even pressure on the operative site. c. Low Fowlers, to minimize protrusion of body tissues. d. Dorsal recumbent, to minimize undue tension on the incision. Question 29 Marks: 1 Triage is the process of assessing patients to determine management priorities. Which of the following patients will be classified as Urgent (yellow)? Choose one answer. a. A unconscious 50 year old man caught in the act of a robbery with a gunshot wound in the abdomen and has no vital signs. b. A relatively healthy 15 year old in status asthmaticus. c. A 5 year old who fell from the monkey bars arms first with comminuted fractures. d. A 20 year old female who fell from her bicycle resulting to an open fracture on her right arm and is bleeding. Question 30 Marks: 1 Peptic Ulcer Disease (PUD) involves a break in continuity of the esophageal, gastric, or duodenal mucosa. The following are characteristics of gastric ulcer EXCEPT: Choose one answer. a. Melena more common than hematemesis b. Decreased acid secretion c. Recurrence unlikely after surgery d. Has a malignancy potential (occurs in approximately 10% of clients) Question 31 Marks: 1 Thoracentesis is the removal of air or fluid from the pleural space. This is done for diagnostic purposes or to alleviate respiratory distress. The

following are true about thoracentesis EXCEPT: Choose one answer. a. Not less than 1000ml of fluid are removed at a time to ensure adequate amount of specimen to be sent to laboratory and diagnostics. b. Nursing care includes obtaining an informed consent. c. Ensure that chest X-ray is done before and after the procedure. d. Complications include pneumothorax from trauma to the lung and pulmonary edema resulting from sudden fluid shifts. Question 32 Marks: 1 An intravenous pyelogram (IVP) is a series of x-rays of the kidneys, ureters, and bladder following the administration of an intravenous iodine preparation. Your patient, Mark, is suspected to have renal calculi. An intravenous pyelogram was ordered. Mark asked you what will the experience be like. Your best answer should be: Choose one answer. a. "It will be different, but cool. Are you scared?" b. "You are going to feel an increased urgency to void, but you have to hold it." c. "You may feel some facial flushing and a warm sensation." d. "It feels just a bit itchy and you may experience shortness of breath. That is just a normal reaction to the dye." Question 33 Marks: 1 Hemorrhage is the escape of copious amounts of blood from the blood vessels. The surgical client is at risk for hemorrhage at anytime during the post operative phase. You are informed that a post op patient has had hemorrhage described as intermediary. You understand that this means that the hemorrhage occurred: Choose one answer. a. Before the surgery b. At the time of surgery.

c. During the first few hours after the surgery when the rise of blood pressure to its normal level dislodges insecure clots from untied vessels. d. Some time after surgery if a ligature slips because the blood vessel is insecurely tied, became infected, or was eroded by a drainage tube. Question 34 Marks: 1 Metastatic cancers spread to the liver in three ways: by extension from adjacent organs, via the hepatic arterial system, or via the portal venous system. Also, as a result of cell migration, the surface of the liver may become seeded with metastatic cells. In metastatic hepatic cancers, diagnostic studies and physical examination may reveal: Choose one answer. a. All of the above b. Reversal of albumin-globulin ratio c. Elevated alkaline phosphatase d. A friction rub or bruit over the liver Question 35 Marks: 1 An isotonic solution is a solution compatible with body tissues and one in which red blood cells can be placed without causing them to burst or shrivel. Myrna was admitted in the ER because of severe isotonic dehydration. As her nurse, which IV fluid would you anticipate to be ordered by her doctor? Choose one answer. a. 0.45% sodium chloride b. D5LR c. 3% sodium chloride d. 0.9% PNSS Question 36 Marks: 1 Primary skin lesions are visually recognizable structural changes in

the skin that have specific characteristics. You are assessing a patient's trunk when you saw elevated, sharply defined lesions containing serous fluid with diameters that are less than 1 cm. You document these as: Choose one answer. a. Small cysts b. Vesicles c. Bullae d. Papules Question 37 Marks: 1 The surgical management of bladder cancer involves the removal of the bladder and creation of a urinary diversion. Your client, Mari was diagnosed with bladder cancer and chose to have a urinary diversion. The student nurse assigned to Mari read about this in the chart and asked you, "what is an ileal reservoir?" You would correctly reply by saying: Choose one answer. a. "Urine is diverted directly to the large intestines." b. "Urine is diverted into a surgically created pouch." c. "Urine is diverted through a portion of the intestine to the skin surface." d. "Urine is diverted directly to the skin surface" Question 38 Marks: 1 A variety of medications including antacids, histamine receptor antagonists, cholinergics, gastrointestinal stimulants, and proton pump inhibitors may be used for treatment of gastroesophageal reflux disease (GERD). Beatriz, your patient who has GERD, eats her breakfast at 8AM. When is the best time to administer Metoclopramide (Reglan)? Choose one answer. a. 6:00 AM

b. 7:30 AM c. Before sleeping the night before. d. Immediately before she eats. Question 39 Marks: 1 Pain is a complex, multifaceted phenomenon. It is an individual, unique experience that may be difficult for clients to describe or explain and is difficult for others to recognize, understand, and assess. The characteristics of a person's pain experience depend, in part, on the source of noxious stimulation. Therefore, it is helpful to understand the typical characteristics of each pain type. Jacob's ECG shows S-T elevation but is complaining not of pain on the chest but pain on his left arm and shoulders. This type of pain is called: Choose one answer. a. Neuropathic pain b. Deep somatic pain c. Visceral pain d. Referred pain Question 40 Marks: 1 Buerger's disease and Raynaud's disease are often compared with each other since both are diseases of blood vessels and both can lead to complications such as gangrene. Which of the following is true about Raynaud's disease? Choose one answer. a. The cause of this disease is unknown, although there is a strong association with tobacco smoking. b. It usually occurs unilaterally. c. Distal numbness is observed. d. It is the episodic spasm of small arteries and arterioles due to exposure to cold and emotional distress. Question 41

Marks: 1 Hysterectomy is the surgical removal of the uterus. Your client, Marissa has had an abdominal hysterectomy 8 hours ago. Which position should you teach Marissa to avoid? Choose one answer. a. Lateral recumbent b. Supine c. High Fowlers d. Side-lying Question 42 Marks: 1 Mastectomy is one of the surgical interventions related to breast cancer. It is the treatment of choice for the following, EXCEPT: Choose one answer. a. When the client cannot comply with daily radiation therapy. b. The tumor exhibits extensive intraductal disease involving multiple quadrants of the breast. c. The tumor involves the nipple-areolar complex. d. The tumor is smaller than 7cm. e. All of the above. Question 43 Marks: 1 The body's electrolytes are regulated by the kidneys, endocrine system, gastrointestinal system, and the vascular system. If there is an alteration in electrolytes, the client experiences a state of disequilibrium. Valerie has Addison's disease. She is a patient in your ward. Youve had a busy last two hours in your shift, and it was about to end. Her watcher approached you and told you that Valerie has cramps. You checked upon her and found out she has a slow pulse rate, muscle weakness, paresthesia and muscle twitching. You could hypothesize that she is having which electrolyte imbalance? Choose one answer.

a. Hypochloremia b. Hypernatremia c. Hypocalcemia d. Hyperkalemia Question 44 Marks: 1 Craniotomy is the surgical creation of an opening into the skull. Your patient has oligodendroglioma and is status post craniotomy. You have a nursing diagnosis of Risk for ineffective cerebral tissue perfusion related to mechanical pressure on vessels of the brain secondary to edema. Which of the following is an INCORRECT intervention for this patient? Choose one answer. a. Administer steroids as ordered. b. Maintain head and neck in neutral alignment. c. Assess neurologic status frequently. d. Keep patient in supine position. Question 45 Marks: 1 The nails are important in assessment as they reflect the overall health of the client, indicating nutrition, and respiratory status. Your patient has chronic obstructive pulmonary disease (COPD) for 4 years. You would most likely observe this in his nails: Choose one answer. a. Clubbing b. Spoon shape c. Paronychia d. Splinter hemorrhages Question 46 Marks: 1

Diabetes mellitus (DM) is a chronic systemic disease characterized by either a deficiency of insulin or a decreased ability of the body to use insulin. Which of the following values are significant in DM? Choose one answer. a. Glycosylated hemoglobin of 5% b. 2-hour postload (postprandial) glucose level of 180 mg/dl c. Fasting blood glucose of 130 mg/dl d. Casual blood glucose of 178mg/dl Question 47 Marks: 1 Closed reduction of fractures is followed by immobilization to relieve or reduce pain and to prevent rotation and shearing at the fracture site. Immobilization is most often accomplished through use of a cast or splint. Mandy broke her tibia and cast was applied. Which of the following is an INCORRECT intervention for her? Choose one answer. a. Create a turning schedule for position changes every 3 hours. b. Exercise her toes. c. Support Mandy's foot with the ankle in 90 degrees of flexion. d. Elevate entire extremity higher than the client's heart for the first 24-48 hours. Question 48 Marks: 1 Vitamins are organic compounds or chemicals, found in various food stuff, necessary for the maintenance of normal life. Deficit in this/these vitamins has manifestations of sore reddened mouth, and megaloblastic anemia. Choose one answer. a. Vitamin B12 - Cyanocobalamin b. Vitamin B1 - Thiamine c. Folic Acid

d. Both A and C Question 49 Marks: 1 Acute pharyngitis is the sudden inflammation of the pharynx that is more common in patients younger than 25 years of age. The following can be expected to be manifested by a patient with acute pharyngitis, EXCEPT: Choose one answer. a. Fever b. Swelling in the uvula c. Rhinorrhea d. Swollen lymphoid follicles flecked with white-purple exudates. Question 50 Marks: 1 Normal cranial nerve reflexes require an appropriately received stimulus that produces an appropriate response. You are a nurse at a neurology unit. The nurse from the shift before you endorsed her assessment that Jimmy, admitted due to a traumatic head injury resulting in compression fracture of the supraorbital area, has anesthesia of the cheek and upper lip. Recalling the cranial nerves, you think that most likely this is caused by an injury to which cranial nerve? Choose one answer. a. CN VII - Facial b. CN VI - Abducens c. CN IV - Trochlear d. CN V - Trigeminal

Marks: 1 Prostatitis is the inflammation of the prostate gland and is a common problem. Acute bacterial prostatitis causes severe localized prostatic inflammation that may interfere with the client's ability to void. Most people also experience systemic illness. Which of the following is NOT necessarily at risk for acute bacterial prostatitis? Choose one answer. a. A 25 year old gay man who has had his first unprotected penetrative anal intercourse.

b. A 31 year old single man with who has had a cystoscopy.

c. A 27 year old single man with gonorrhea.

- A 29 year old man with priapism. Acute d. A 29 year old man with bacterial prostatitis is typically caused by priapism. gastrointestinal bacteria or sexually transmitted bacteria. Although most cases have no apparent causes, risk factors include a recent bout of epididymitis, unprotected penetrative anal intercourse, urethral stricture or meatal stenosis, and recent catheterization or cystocopy. (Black & Hawks)

Incorrect Marks for this submission: 0/1. Question 2 Marks: 1 Hemodialysis is used for clients with acute or irreversible renal failure and fluid and electrolyte imbalances. The following are among the overall therapeutic effects of hemodialysis Except: Choose one answer. a. Restore fluid, electrolyte, and acidbase balances

b. Reverse some of the untoward manifestations of irreversible renal failure.

c. Clear waste products from the body

d. Provide hydration and - Provide hydration and additional blood supply to additional blood supply the systemic circulation. Excess fluid, potassium, to the systemic urea nitrogen, and acid ions are removed, but only circulation temporarily as these build up between dialyses. (Black & Hawks)

Incorrect Marks for this submission: 0/1. Question 3 Marks: 1 Infective endocarditis refers to a microbial infection involving the endocardium. The following are considered the most reliable criteria for diagnosis of endocarditis EXCEPT: Choose one answer. a. Positive blood cultures

b. A new regurgitant murmur

c. Evidence of endocardial involvement by echocardiography.

d. Elevated cardiac enzymes CK-MB and

- Elevated cardiac enzymes CK-MB and serum markers troponin and myoglobin. Events leading to

cellular injury cause a release of enzymes from serum markers intracellular storage and circulating levels of these troponin and myoglobin are elevated. Acute myocardial infarction can be confirmed by abnormally high levels of these. Blood cultures would be positive because of presence of organisms. Regurgitant murmurs and abnormal findings in echocardiography are results of the destruction of valves and endocardium due to growth of vegetative lesions (Ignatavicius). Incorrect Marks for this submission: 0/1. Question 4 Marks: 1 Compartment syndrome is a condition of compromised circulation related to progressively increasing pressure in a confined space. Your patient, Rod, broke his ulna due to a bad fall during a soccer game. Which of the following statements made by Rod would alert you to the possible presence of compartment syndrome? Choose one answer. a. "I hear voices from my cast saying I should wash it."

b. "I feel a burning sensation on my wrist."

c. "I can smell a fishy odor coming

from my cast."

Incorrect

Marks for this submission: 0/1. Question 5 Marks: 1

The visual pathway is a multidimensional system with many structures and processes subject to trauma or disorders. You are working at a cardiology clinic when a woman in her late 50s was accompanying her husband. While waiting for her husband to be called, the woman approached you and said, "Nurse, what do you think is happening to me? It started as a burst of black spots, and now I have been seeing something like a curtain that falls across my vision. But I don't feel any pain, so I don't think it's really anything serious." You advise it would be best to consult an ophthalmologist as soon as possible because you think most likely, she is having/experiencing: Choose one answer. a. Diabetic retinopat hy

I
d. "I can't feel my fingers."

- "I can't feel my fingers." "I can smell a fishy odor coming from my cast" and "I feel a burning sensation on my wrist" are not associated with compartment syndrome. "I hear voices from my cast saying I should wash it" is associated with psychotic disorders. TIP: The 5 P's of compartment syndrome are: Pain, Pallor, Pulselessness, Paresthesia, and Paralysis. (Balita's Ultimate Nursing Guide)

- Retinal detachment. Retinal detachment is the separation of the b. Retinal retina from the choroid. The onset is usually sudden and detachme accompanied by burst of black spots indicating bleeding that nt results from the detachment. The "curtain" is a result of separation of visual receptors from the neural pathway. Diabetic retinopathy is a progressive disorder of the retina characterized by microscopic damage to the retinal vessels. Open angle glaucoma occurs when aqueous humor outflow through the trabecular meshwork is impaired. Cataract is the opacity of the lens (Black & Hawks). c. Cataract

d. Open angle glaucoma

Incorrect Marks for this submission: 0/1. Question 6 Marks: 1 In a lumbar puncture (LP), also known as spinal tap, a needle is inserted into the subarachnoid space in the lumbar region of the spine below the level of the spinal cord. Cerebrospinal fluid (CSF) can be withdrawn or substances may be injected into this space. Your patient is post-LP. Which of the following is NOT helpful? Choose one answer. a. Encourage fluid intake.

Incorrect

Marks for this submission: 0/1. Question 7 Marks: 1

Thyroid storm (thyrotoxicosis) is a potentially fatal acute episode of thyroid activity. You work in a charity ward and you encounter

I
b. Instruct client to ambulate as soon as possible to improve circulation of CSF. c. If a spinal headache continues, anticipate the need for an epidural blood patch. d. If a spinal headache develops, put client in a dark, quiet room.

- Instruct client to ambulate as soon as possible to improve circulation of CSF. Bed rest is advised to post-LP clients. Although doctors' orders vary, the average duration of bed rest is 3 hours. Encourage fluid intake to replace CSF withdrawn during the test. If a spinal headache develops, put client in a dark, quiet room to decrease irritating stimuli. If a spinal headache continues, an epidural blood patch may be required for it to act as a fibrin patch to seal the hole in the dura and to prevent further CSF leakage. (Black & Hawks)

patients who cannot buy their maintenance medications continuously. Which symptoms indicative of thyroid storm should you regularly assess and watch out for? Choose one answer. a. Extreme irritability and loss of ocular muscle control

b. High fever and severe tachycardia

c. Both A and B - High fever and severe tachycardia and delirium and dehydration. Overfunctioning of the thyroid gland increases the rate of metabolism. (Black & Hawks)

d. Both A and C

e. Delirium and dehydration

Incorrect

Marks for this submission: 0/1. Question 8 Marks: 1 Arthritis is generally the inflammation of the joints that causes pain and swelling. There are four types of arthritis. This kind is chronic, systemic autoimmune disorder whose major distinctive feature is chronic, symmetrical and erosive inflammation of the synovial tissue of joints. Choose one answer. a. Osteoart hritis

b. Gouty arthritis

c. Rheumat - Rheumatoid arthritis. Rheumatoid arthritis (RA) is an inflammatory disorder that primarily involves the synovial oid arthritis membrane. it is believed to be autoimmune. Osteoarthritis is also known as degenerative joint disease and is characterized by a progressive loss of joint cartilage. Gout is a heterogeneous group of condition related to a genetic defect of purine metabolism resulting to hyperuricemia. It has 4 stages and acute gouty arthritis is the second. (Brunner) d. Juvenile arthritis

Incorrect

Marks for this submission: 0/1. Question 9 Marks: 1

One of the most important aspects of providing nursing care to the HIV-infected client is helping with the compliance of the antiretroviral regimen. Your patient, Dave, is taking antiretroviral medications. Dave should take note of the following correct instructions EXCEPT: Choose one answer.

In
a. Take the drug anytime you feel - Take the drug anytime you feel symptoms. symptoms. The regimen ordered are commonly complex and require the patient to take in large numbers of pills daily (as many as 30-40), often at exactly spaced intervals and with different requirements as to timing the pills related to food. (Black & Hawks) b. Do not skip a dose. c. If you take the drug only periodically, it would be better not to take them at all. This matter should be discussed with the physician.

d. Take the drug at prescribed intervals.

Incorrect

Marks for this submission: 0/1. Question 10 Marks: 1

Nephrolithiasis is a condition in which one or more kidney stones are present in the urinary system. Among the nursing interventions is ensuring a correct diet for the patient. Agnes has a history of uric acid kidney stones. Which food should be avoided by the client on a purine-restricted diet? Choose one answer. a. Tapioc a

b. Peanut butter

c. Milk

Incorrect

Marks for this submission: 0/1. Question 11 Marks: 1

In emphysema, the impaired gas exchange of oxygen and carbon dioxide results from destruction of walls of overdistended alveoli. You are a nurse in a medicine ward taking care of Amy, admitted for emphysema. The doctor ordered oxygen at 2 liters per minute via nasal cannula. When administering the oxygen to this patient, the nurse should observe closely for: Choose one answer. a. Hemoptysis and low-grade fever

In
d. Liver - Liver. Liver is high in purine and should be avoided by clients with uric acid stones. Tapioca, milk, and peanut butter are sources of protein and are low in purine. (Rinehart, et al) b. Hyperemia and increased respirations

c. Anxiety and diaphoresis

Incorrect

Marks for this submission: 0/1. Question 12 Marks: 1

With advanced age, the body does not respond as vigorously to illness or disease because of the diminished physiological reserve. Which of the following is a structural and functional change in the respiratory system of the older adult? Choose one answer. a. Increased vital capacity

In
d. Drowsiness and decreased - Drowsiness and decreased respirations. Emphysema is a respirations kind of chronic obstructive pulmonary disease (COPD). Clients with COPD respond only to chemical stimulus of low oxygen levels. Administering high concentrations of oxygen eliminate the stimulus to breath thus leading to decrease in respirations and lethargy. (Mosby) b. Decreased rigidity of the thoracic cage

Incorrect

Marks for this submission: 0/1. Question 13 Marks: 1

Injuries that result from direct contact with or exposure to any thermal, chemical, or radiation source are termed burns. You are a nurse rotating at the hospital's burn unit. Bed 4, Mario, is a third year student nurse who was part of a burn accident. He asked you what the classification of his burns are, pointing to a part of his skin that have variable colors ranging from deep red, white, black and brown, has a dry surface, has visible thrombosed vessels, is not blanchable, and is not painful. You answer him that this is a: Choose one answer. a. Fourth degree full thickness burn

In
c. Increased cough efficiency d. Increased residual volume - Increased residual volume. Along with this, they also have increased rigidity of the thoracic cage, decreased vital capacity, decreased cough efficiency, decreased elasticity, and decreased ciliary action, making them at risk for problems such as pneumonia, chronic obstructive pulmonary disease, and dyspnea. (Black & Hawks)

Incorrect

Marks for this submission: 0/1. Question 14 Marks: 1

Third degree atrioventricular (AV) block occurs when no atrial pulse is conducted through the AV node into the ventricles. You are assigned to a patient with third degree AV block. Your priority nursing problem will most likely be:

In
b. Third degree full thickness burns - Third degree full thickness burns. First degree partial thickness burn is blanchable with pressure, is painful and has erythema. Second degree partial thickness burns have wet, shiny weeping surfaces. These are very sensitive and painful. Blisters are also observed here and skin is still blanchable. Fourth degree full thickness burns have charring visible in the deepest areas and extremity movement is limited. (Black & Hawks) c. Third degree partial thickness burn d. Second degree partial thickness burn

Choose one answer. a. Activity intolerance

b. Decreased cardiac output

- Decreased cardiac output. Decreased cardiac output is defined as a state in which the individual experiences a reduction in the amount of blood pumped by the heart, resulting in compromised cardiac function (Carpenito). Ineffective tissue perfusion may result from decreased cardiac output. Activity intolerance and anxiety may both be present but are not prioritized over the physiologic problem of decreased cardiac output.

c. Anxiety

d. Ineffective tissue perfusion

Incorrect Marks for this submission: 0/1. Question 15 Marks: 1 Medications used in the treatment of hyperthyroid conditions are used to reduce thyroid hormone levels. This drug offers a more permanent option for patients with hyperthyroidism because it destroys thyroid

tissue. Choose one answer. a. Radioactive Iodine (131 - Radioactive Iodine (131 I). This drug offers a more permanent option for patients with hyperthyroidism because it destroys I) thyroid tissue. The effects of PTU and Methimazole (Topazole) are more temporary; they inhibit thyroid hormone synthesis. Levothyroxine (Synthroid) is used for clients with hypothyroidism. (Black & Hawks)

b. Levothyrox ine (Synthroid)

c. Propylthiou racil or PTU

d. Methimazol e (Topazole)

Incorrect Marks for this submission: 0/1. Question 16

Marks: 1 Ovarian cancer, although not the highest in incidence among reproductive tumors, is the leading cause of death from genital reproductive malignancies, and accounts for about 4% of all cancer deaths in women. Which of the following is least likely to be at risk for ovarian cancer? Choose one answer. - A graduating medical intern who uses oral contraceptives to prevent pregnancy. Risk factors of ovarian cancer include age older than 40 years old, family history of ovarian or breast cancer, family history of hereditary a. A graduating medical intern nonpolyposis colorectal cancer (HNPCC), who uses oral contraceptives history of infertility, and history of dysmenorrhea. The use of ovulationto prevent pregnancy. stimulating medications is also a risk factor because a high number of ovulations increases the chances that a tumor suppressor gene called p53 can be mutated. Health promotion factors include telling clients that ovarian cancer may be prevented by anything that interrupts constant ovulatory cycles such as more than one full term pregnancy, oral contraceptive use, breast feeding, and bilateral oophorectomy. (Black & Hawks) b. A nun who has been celibate all her life.

c. A fitness instructor who has a grandmother who died of hereditary nonpolyposis colorectal cancer (HNPCC).

d. A married woman who has trouble conceiving and uses ovulation-stimulating medications.

Incorrect Marks for this submission: 0/1. Question 17 Marks: 1 Seizure is defined as a sudden attack of a condition, as in a convulsion. Sandra is one of your patients in the community who has a seizure disorder. She takes maintenance drugs phenytoin (Dilantin) and carbamazepine (Tegretol). She asks you about getting married with her boyfriend and having children. Which of the following would be your most appropriate response? Choose one answer. a. "You should wait until your seizures are cured before you consider having children."

b. "You will have a hard time conceiving because of your meds. You should stop having birth control 6 months prior to actually conceive."

c. "Your children would not necessarily have a seizure disorder. Just make sure you go to prenatal checkups regularly."

d. "When you decide to have children, ask your doctor about changing your medication."

- "When you decide to have children, ask your doctor about changing your medication." Phenytoin sodium is a known teratogenic agent. This causes numerous problems on the fetus. Therefore, a proper consultation with the doctor regarding changing her medications is needed. Anticonvulsant medications increase during pregnancy. "You should wait until your seizures are cured before you consider having children" is wrong because seizure disorders cannot be cured, although they can be controlled. Seizure disorders and infertility are not related. (Lippincott NCLEX)

Incorrect Marks for this submission: 0/1. Question 18 Marks: 1 Amnesia means loss of memory. This can be a manifestation or a complication in brain injury. Memory loss associated with events prior to the time of injury is referred to as Choose one answer. a. Preretrograde amnesia

Incorrect

Marks for this submission: 0/1. Question 19 Marks: 1

Antihypertensive drugs, which act to reduce blood pressure, are used to treat hypertension, a disorder characterized by elevation in systolic, diastolic, or both blood pressures. Treatment for hypertension typically begins with a diuretic or a calcium channel blocker. Which of the following drugs is neither a diuretic nor a calcium channel blocker?

In
b. Traumatic amnesia c. Antegrade amnesia d. Retrograd e amnesia - Retrograde amnesia. Antegrade amnesia is memory loss associated with events after the time of injury. Traumatic amnesia is having both retrograde and antegrade amnesia. (Bullock)

Choose one answer. a. Indap amide

b. Imipra mine

- Imipramine. Imipramine is a tricyclic antidepressant. Indapamide is a thiazide-like diuretic, while amlodipine and verapamil are calcium channel blockers.(Clinical Pharmacology Made Easy)

c. Amlod ipine

d. Verap amil

Incorrect Marks for this submission: 0/1. Question 20 Marks: 1 Allergies are among the most common disorders seen in the medical community. The client often requires a combination of treatments

ranging from avoidance of allergens to immunotherapy. You are educating Andrea's mother about controlling the environment. The following are correct tips on desensitizing a room EXCEPT: Choose one answer. a. Using wallpaper on walls - Using wallpaper on walls instead of paint. instead of paint. Paint or washable wallpaper are preferred. Wallpaper can swell and this indicates presence of molds. Other choices are ways of controlling the environment. Combined with air filters that remove small particles from the air, these can help eliminate many allergens. (Black & Hawks) b. Toys should be wood, metal, or plastic.

c. Using rubberized canvas or plastic upholstered furniture instead of fabric upholstery.

d. Using instant roll-up washable cotton or synthetic window shades instead of Venetian blinds.

Incorrect Marks for this submission: 0/1. Question 21

Marks: 1 Mucositis is inflammation with erythema or ulceration of the mucous membrane resulting from cancer therapy. You are the primary nurse of Helena, who was diagnosed with acute myeloid leukemia and was undergoing chemotherapy. You identified Knowledge Deficit: Self-Care as one of your nursing problems. After nursing intervention, which of the following statements, if stated by the client, would indicate the need for further teaching? Choose one answer. - "Toothbrushing can be dangerous. I can use lemon-glycerine swabs instead." Research dating back to the 1960s showed lemonglycerine swabs do more harm than good because they decalcify the teeth, make the pH of the mouth more acidic and dry mucous membranes. A soft toothbrush has been found to be better than foam brushes and is preferred by clients. Mucositis often involves not only the oral cavity but also the mucous membrane of the entire GI tract, hence carefully cleansing the perirectal area after each bowel movement is needed. Mucositis may further be complicated by infection so doctors usually prescribe prophylactic antibiotics. (Black & Hawks)

a. "Toothbrushing can be dangerous. I will use lemon-glycerine swabs instead."

b. "I will use a soft bristled toothbrush for oral hygiene."

c. "I will carefully cleanse my perirectal area after each bowel movement."

d. "The doctor prescribed

meds to prevent yeast infection. I should take them as prescribed."

Incorrect

Marks for this submission: 0/1. Question 22 Marks: 1

The eyelids, lacrimal gland, and conjunctiva are also important to include in assessment. The following disorders/symptoms involve the need for artificial tears in their management, EXCEPT: Choose one answer. a. Absence of blinking

- Chalazion. Chalazion is the chronic granuloma of the meibomian b. gland and is manifested by painless, localized swelling of the Chalazion eyelid margin, which may be surgically removed when cosmetically distracting. Ptosis is the drooping of the eyelids by several causes, while lagophthalmos is the inadequate closure of eyelids. With ptosis and lagophthalmos, the eyes become irritated because of drying and loss of tears. Absence of blinking is blinking fewer than 20 times a minute and is seen on patients with Parkinson's disease and hyperthyroidism. (Black & Hawks) c. Lagophth almos

Incorrect

Marks for this submission: 0/1. Question 23 Marks: 1

Tuberculosis (TB) is considered as one of the world's deadliest disease and remains as a major public health problem in the Philippines. A mother has an active TB and you are a community health nurse giving health teaching to her and her family. Which statement made by the family best indicates understanding of your teachings? Choose one answer.

In
d. Ptosis a. "She should be kept in a room for as long as the disease is active" b. "She can't cook our meals"

c. "We all need to wash our hands carefully, - "We all need to wash our hands carefully, but but especially our especially our mother". Hand washing is the best toll mother" for prevention of infection. The client should wash their hands very carefully after any contact with body substances, masks, or soiled tissues. (Thomson Delmar Learning's Nursing Review Series)

d. "We won't share clothes with mother."

Incorrect Marks for this submission: 0/1. Question 24 Marks: 1 Hearing impairment ranges from minor difficulty in understanding words or hearing certain sounds to total deafness. This type of hearing loss is described as loss of hearing for which no organic reason can be found. Choose one answer. a. Functional - Functional hearing loss. This is defined as the loss of hearing hearing for which no organic reason can be found. Mixed hearing loss loss means both sensorineural and conductive hearing are lost. Fluctuating hearing loss is sensorineural hearing loss that varies with time. Neural hearing loss is a sensorineural hearing loss originating in the 8th cranial nerve or brain stem. (Black & Hawks) b. Fluctuating hearing loss

Incorrect

Marks for this submission: 0/1. Question 25 Marks: 1

Angiography is an x-ray examination using contrast dye to visualize patency of the arteries. You just received your patient from the cardiac catheterization lab and is status post angiography. The following are part of your nursing care EXCEPT: Choose one answer.

In
c. Neural hearing loss d. Mixed hearing loss a. Assess the circulatory status of the extremities.

b. Encourage immediate ambulation. - Encourage immediate ambulation. Bed rest is enforced after angiography to prevent complications such as bleeding. (Ignatavicius)

c. Maintain pressure over site.

d. Check the injection site for bleeding and inflammation.

Incorrect Marks for this submission: 0/1. Question 26 Marks: 1 Intradermal testing is the most accurate skin test but is linked to a higher incidence of severe allergic reactions. You administered a skin test of Cefuroxime on Peter. You conclude an immediate reaction. You therefore assessed that: Choose one answer. a. Reaction appeared within 2030 minutes after injection, marked by vesicle formation greater than 3 mm of the positive control.

Incorrect

Marks for this submission: 0/1. Question 27 Marks: 1

A colostomy is a procedure that involves creating an opening between

I
b. Reaction appeared in less than 5 minutes after injection, marked by patch formation greater than 5 mm of the positive control. c. Reaction appeared within 10-20 minutes after injection, marked by bulla formation greater than 5 mm of the positive control. d. Reaction appeared within 1020 minutes after injection, marked by erythema and wheal formation greater than 3 mm of the positive control. - Reaction appeared within 10-20 minutes after injection, marked by erythema and wheal formation greater than 3 mm of the positive control. This denotes a positive reaction. This indicates an antibody response to previous exposure to this antigen and suggest that the client is allergic to the particular substance that caused the reaction. Vesicles, bulla, and patches are not associated reactions. (Black & Hawks)

the colon and abdominal wall, from which fecal contents will pass. You are a nurse in the surgical ward and you are preparing Bart for discharge. You just gave your health teachings for Bart, who just has had a colostomy. You are now going to evaluate what he learned. Which of the following statements made by Bart is correct? Choose one answer. a. "The stoma should be beefy red, as this indicates good blood supply." - "The stoma should be beefy red, as this indicates good blood supply." A pale red or pink stoma indicates a limited blood supply. A healthy stoma is moist and dryness suggests dehydration. Some bleeding is normal; application of pressure should stop the bleeding. Excessive bleeding should be reported to the physician immediately. Excessive flatus should be expected for 4-8 weeks. (Black & Hawks)

b. "The stoma should always be dry, since moisture can make me at risk for infection."

c. "I should only be alarmed if bleeding is more than 1000ml."

d. "Excessive flatus will be present in about 1 week."

Incorrect

Marks for this submission: 0/1. Question 28 Marks: 1 Wound dehiscence and evisceration are surgical complications, and are especially serious when they involve abdominal incisions or wounds. When this occurs, the patient is instructed to lie quietly in this position: Choose one answer. a. Supine, to even out the flesh.

b. Prone, to provide even pressure on the operative site.

c. Low Fowlers, to minimize protrusion of body tissues. - Low Fowlers, to minimize protrusion of body tissues. (Brunner)

d. Dorsal recumbent, to minimize undue tension on the incision.

Incorrect Marks for this submission: 0/1.

Question 29 Marks: 1 Triage is the process of assessing patients to determine management priorities. Which of the following patients will be classified as Urgent (yellow)? Choose one answer. a. A unconscious 50 year old man caught in the act of a robbery with a gunshot wound in the abdomen and has no vital signs.

b. A relatively healthy 15 year old in status asthmaticus.

c. A 5 year old who fell from the monkey bars arms first with comminuted fractures.

d. A 20 year old female who fell from her bicycle resulting to an open fracture on her right arm and is bleeding.

- A 20 year old female who fell from her bicycle resulting to an open fracture on her right arm and is bleeding. Yellow-tagged clients have major injuries such as open fractures and large wounds that need treatment within a 30-minute to 1-hour time frame. Red-tagged clients have immediate threats to life such as airway compromise or hemorrhagic shock that

require immediate treatment. Greentagged clients have minor injuries that can be managed in a delayed fashion, such as closed fractures and abrasions. (Ignatavicius) Incorrect Marks for this submission: 0/1. Question 30 Marks: 1 Peptic Ulcer Disease (PUD) involves a break in continuity of the esophageal, gastric, or duodenal mucosa. The following are characteristics of gastric ulcer EXCEPT: Choose one answer. a. Melena more common than hematemesis

- Bleeding pattern: melena more common than hematemesis. This is associated more to duodenal ulcers. Hematemesis is more common than melena in gastric ulcers. (Black & Hawks)

b. Decreased acid secretion

c. Recurrence unlikely after surgery

d. Has a malignancy potential (occurs in approximately 10% of

clients)

Incorrect

Marks for this submission: 0/1. Question 31 Marks: 1

Thoracentesis is the removal of air or fluid from the pleural space. This is done for diagnostic purposes or to alleviate respiratory distress. The following are true about thoracentesis EXCEPT: Choose one answer.

I
a. Not less than 1000ml of fluid are removed at a time to ensure adequate amount of specimen to be sent to laboratory and diagnostics. b. Nursing care includes obtaining an informed consent. c. Ensure that chest X-ray is done before and after the procedure.

- Not less than 1000ml of fluid are removed at a time to ensure adequate amount of specimen to be sent to laboratory and diagnostics. The amount of fluid removed is limited to 1200 to 1500 ml at one time. The amount of fluid is limited to reduce risk of cardiovascular collapse from rapid removal too much fluid. (Lemone & Burke)

d. Complications include pneumothorax from trauma to the lung and pulmonary edema resulting from sudden fluid shifts.

Incorrect Marks for this submission: 0/1. Question 32 Marks: 1 An intravenous pyelogram (IVP) is a series of x-rays of the kidneys, ureters, and bladder following the administration of an intravenous iodine preparation. Your patient, Mark, is suspected to have renal calculi. An intravenous pyelogram was ordered. Mark asked you what will the experience be like. Your best answer should be: Choose one answer. a. "It will be different, but cool. Are you scared?"

b. "You are going to feel an increased urgency to void, but you have to hold it."

c. "You may feel some facial flushing and a warm sensation."

- "You may feel some facial flushing and a warm sensation." The client undergoing an IVP may feel facial flushing, a warm sensation, and a metallic taste in the mouth. "It will be different, but cool. Are you

scared?" is nontherapeutic. C is associated with anaphylaxis and though uncommon, is never normal. D is not associated with the contrast media used in IVP. (Rinehart, et al) d. "It feels just a bit itchy and you may experience shortness of breath. That is just a normal reaction to the dye."

Incorrect Marks for this submission: 0/1. Question 33 Marks: 1 Hemorrhage is the escape of copious amounts of blood from the blood vessels. The surgical client is at risk for hemorrhage at anytime during the post operative phase. You are informed that a post op patient has had hemorrhage described as intermediary. You understand that this means that the hemorrhage occurred: Choose one answer. a. Before the surgery

b. At the time of surgery.

c. During the first few hours after - During the first few hours after the the surgery when the rise of blood surgery when the rise of blood pressure

pressure to its normal level dislodges insecure clots from untied vessels.

to its normal level dislodges insecure clots from untied vessels. Primary hemorrhages occur at the time of surgery. Secondary hemorrhages occur some time after surgery if a ligature slips because the blood vessel is insecurely tied, became infected, or was eroded by a drainage tube. (Brunner)

d. Some time after surgery if a ligature slips because the blood vessel is insecurely tied, became infected, or was eroded by a drainage tube.

Incorrect Marks for this submission: 0/1. Question 34 Marks: 1 Metastatic cancers spread to the liver in three ways: by extension from adjacent organs, via the hepatic arterial system, or via the portal venous system. Also, as a result of cell migration, the surface of the liver may become seeded with metastatic cells. In metastatic hepatic cancers, diagnostic studies and physical examination may reveal: Choose one answer. a. All of the above - all of the above. Clients with primary (benign and malignant) and secondary (metastatic tumors) often show similar manifestations. Many clients with metastatic with metastatic malignancy of the liver have the following three types of manifestations: 1. Manifestations that are specific only to the primary tumor, hepatic involvement being discovered incidentally in the course of a diagnostic evaluation. 2. Nonspecific manifestations of anorexia, diaphoresis, fever, weight loss, and weakness. 3. Manifestations of active liver disease, such as abdominal

pain, ascites, and hepatomegaly. (Black & Hawks) b. Reversal of albuminglobulin ratio

c. Elevated alkaline phosphatase

d. A friction rub or bruit over the liver

Incorrect Marks for this submission: 0/1. Question 35 Marks: 1 An isotonic solution is a solution compatible with body tissues and one in which red blood cells can be placed without causing them to burst or shrivel. Myrna was admitted in the ER because of severe isotonic dehydration. As her nurse, which IV fluid would you anticipate to be ordered by her doctor? Choose one answer. a. 0.45% sodium chloride

Incorrect

Marks for this submission: 0/1. Question 36 Marks: 1

Primary skin lesions are visually recognizable structural changes in the skin that have specific characteristics. You are assessing a patient's trunk when you saw elevated, sharply defined lesions containing serous fluid with diameters that are less than 1 cm. You document these as: Choose one answer. a. Small

In
b. D5LR c. 3% sodium chloride d. 0.9% PNSS - 0.9% PNSS. This solution is isotonic. 0.45% sodium chloride is hypotonic while D5LR and 3% sodium chloride are both hypertonic solutions. (Rinehart, et al)

cysts

Incorrect

Marks for this submission: 0/1. Question 37 Marks: 1

The surgical management of bladder cancer involves the removal of the bladder and creation of a urinary diversion. Your client, Mari was diagnosed with bladder cancer and chose to have a urinary diversion. The student nurse assigned to Mari read about this in the chart and asked you, "what is an ileal reservoir?" You would correctly reply by saying:

In
c. Bullae d. Papule s

b. Vesicl - Vesicles. Vesicles are elevated, sharply defined lesions containing es serous fluid with diameters that are less than 1 cm, usually seen in chickenpox and herpes simplex. Bullae are large, elevated fluid-filled lesions greater than 1cm. Cysts are elevated, thick-walled lesions containing fluid or semisolid matter. Papules are elevated, solid lesions of less than 1 cm. (Black & Hawks)

Choose one answer. a. "Urine is diverted directly to the large intestines."

b. "Urine is diverted into - "Urine is diverted into a surgically created pouch." An ileal reservoir (Kock pouch) is a surgically a surgically created created pouch where urine is collected and pouch." removed through a one-way stoma. Urine that is diverted through a portion of the intestine to the skin surface is an ileal conduit. In ureterostomy, urine is diverted directly to the skin while sigmoidostomy is when urine is diverted directly to the large intestines. (Rinehart, et al) c. "Urine is diverted through a portion of the intestine to the skin surface."

d. "Urine is diverted directly to the skin surface"

Incorrect Marks for this submission: 0/1. Question 38 Marks: 1

A variety of medications including antacids, histamine receptor antagonists, cholinergics, gastrointestinal stimulants, and proton pump inhibitors may be used for treatment of gastroesophageal reflux disease (GERD). Beatriz, your patient who has GERD, eats her breakfast at 8AM. When is the best time to administer Metoclopramide (Reglan)? Choose one answer. a. 6:00 AM

b. 7:30 AM

- 7:30 AM. Metoclopramide, an antiemetic and cholinergic drug, may be prescribed to patients with GERD because it increases lower esophageal sphincter (LES) pressure by stimulating the smooth muscle of the GI tract and increases the rate of gastric emptying (GI stimulant). This medication is administered 30-60 minutes before meals. (Black & Hawks)

c. Before sleeping the night before.

d. Immediately before she eats.

Incorrect Marks for this submission: 0/1. Question 39

Marks: 1 Pain is a complex, multifaceted phenomenon. It is an individual, unique experience that may be difficult for clients to describe or explain and is difficult for others to recognize, understand, and assess. The characteristics of a person's pain experience depend, in part, on the source of noxious stimulation. Therefore, it is helpful to understand the typical characteristics of each pain type. Jacob's ECG shows S-T elevation but is complaining not of pain on the chest but pain on his left arm and shoulders. This type of pain is called: Choose one answer. a. Neuropa thic pain

b. Deep somatic pain

c. Visceral pain

d. - Referred pain. The fibers innervating these areas (left arm and Referred shoulders) are close to those innervating the myocardium, result pain in referred pain. Deep somatic pain is poorly localized, generally diffuse and may radiate (e.g. rheumatoid arthritis). Visceral pain refers to pain coming from body organs e.g. acute appendicitis and renal colic. Neuropathic pain is caused by damage or injury to nerve fibers in the periphery or damage to the CNS. (Black &

Incorrect

Marks for this submission: 0/1. Question 40 Marks: 1

Buerger's disease and Raynaud's disease are often compared with each other since both are diseases of blood vessels and both can lead to complications such as gangrene. Which of the following is true about Raynaud's disease? Choose one answer.

Co
Hawks) a. The cause of this disease is unknown, although there is a strong association with tobacco smoking. b. It usually occurs unilaterally. c. Distal numbness is observed.

d. It is the episodic spasm of small arteries and arterioles - It is the episodic spasm of small arteries and due to exposure to cold and arterioles due to exposure to cold and emotional distress. emotional distress. Raynaud's disease occurs bilaterally while Raynaud's phenomenon is unilateral. Distal numbness and tobacco smoking as a risk factor are observed in Buerger's disease. (Ignatavicius) Incorrect Marks for this submission: 0/1. Question 41 Marks: 1 Hysterectomy is the surgical removal of the uterus. Your client, Marissa has had an abdominal hysterectomy 8 hours ago. Which position should you teach Marissa to avoid? Choose one answer. a. Lateral recumbe nt

b. Supine

c. High - High Fowlers. This position can cause pelvic congestion, thereby Fowlers increasing the risk for complications. Lateral recumbent, supine, and side-lying do not contribute to pelvic congestion and are therefore desirable for the client. (Black & Hawks)

Incorrect

Marks for this submission: 0/1. Question 42 Marks: 1

Mastectomy is one of the surgical interventions related to breast cancer. It is the treatment of choice for the following, EXCEPT: Choose one answer.

Co
d. Sidelying a. When the client cannot comply with daily radiation therapy. b. The tumor exhibits extensive intraductal disease involving multiple quadrants of the breast. c. The tumor involves the

nipple-areolar complex.

Incorrect

Marks for this submission: 0/1. Question 43 Marks: 1

The body's electrolytes are regulated by the kidneys, endocrine system, gastrointestinal system, and the vascular system. If there is an alteration in electrolytes, the client experiences a state of disequilibrium. Valerie has Addison's disease. She is a patient in your ward. Youve had a busy last two hours in your shift, and it was about to end. Her watcher approached you and told you that Valerie has cramps. You checked upon her and found out she has a slow pulse rate, muscle weakness, paresthesia and muscle twitching. You could hypothesize that she is having which electrolyte imbalance? Choose one answer. a. Hypoch loremia

I
d. The tumor is smaller than 7cm. e. All of the above.

- The tumor is smaller than 7cm. Mastectomy is usually recommended if the tumor involves the nipple-areolar complex and the tumor exhibits extensive intraductal disease involving multiple quadrants of the breast. Mastectomy is also the treatment of choice when the tumor is larger than 7cm, and when the client cannot comply with daily radiation therapy. (Black & Hawks)

Incorrect

Marks for this submission: 0/1. Question 44 Marks: 1

Craniotomy is the surgical creation of an opening into the skull. Your patient has oligodendroglioma and is status post craniotomy. You have a nursing diagnosis of Risk for ineffective cerebral tissue perfusion related to mechanical pressure on vessels of the brain secondary to edema. Which of the following is an INCORRECT

In
b. Hypern atremi a c. Hypoca lcemia d. - Hyperkalemia. Potassium is the most abundant cation in the body. Hyperk Weak pulse, lethargy, and confusion are some of the symptoms of alemia deficit in potassium. Hypocalcemia is manifested by increased heart rate, anxiety, (+) Trosseau's sign, Chvostek's sign, among others. Hypernatremia is manifested by decreased myocardial control, diminished cardiac output, agitation, etc. Hypochloremia accompany the loss of sodium but have characteristics nonspecific to chloride. (Rinehart, et al)

intervention for this patient? Choose one answer. a. Administer steroids as ordered.

b. Maintain head and neck in neutral alignment.

c. Assess neurologic status frequently.

d. Keep patient in supine position.

- Keep patient in supine position. The head of the bed should be elevated to 30 degrees to facilitate venous drainage and reduce edema. Assess neurologic status frequently since a change in this is the first sign of increasing intracranial pressure. Administer steroids as ordered; steroids reduce cerebral edema. Maintain head and neck in neutral alignment to facilitate venous drainage and reduce edema. (Black & Hawks)

Incorrect Marks for this submission: 0/1. Question 45

Marks: 1 The nails are important in assessment as they reflect the overall health of the client, indicating nutrition, and respiratory status. Your patient has chronic obstructive pulmonary disease (COPD) for 4 years. You would most likely observe this in his nails: Choose one answer. a. - Clubbing. Clubbing is increased angle between nail plate and Clubbing nail base. This is caused by long-standing hypoxia. Spoon shaped nails are concave as the nail curves upward from the nail bed, caused by use of strong detergents, syphilis, or iron deficiency anemia. Paronychia is the inflammation of the skin fold at the nail margin and is caused by trauma or skin infection at the nail base. Splinter hemorrhages are vertical linear red/ brown streaks in the nail bed seen in minor trauma to the nail, subacute bacterial endocarditis, or trichinosis. (Black & Hawks) b. Spoon shape

c. Paronychi a

d. Splinter hemorrha ges

Incorrect

Marks for this submission: 0/1. Question 46 Marks: 1 Diabetes mellitus (DM) is a chronic systemic disease characterized by either a deficiency of insulin or a decreased ability of the body to use insulin. Which of the following values are significant in DM? Choose one answer. a. Glycosylated hemoglobin of 5%

b. 2-hour postload (postprandial) glucose level of 180 mg/dl

c. Fasting blood glucose of 130 mg/dl

- Fasting blood glucose of 130 mg/dl. Fasting blood glucose generally reflects glucose of hepatic production. A diagnosis of diabetes is made when a client's blood glucose is greater than 126 mg/dl. 2-hour postload (postprandial) glucose level greater than 200 mg/dl after an oral glucose tolerance test is a confirmation for a diagnosis of diabetes because normally, blood glucose should return to fasting levels within 2 hours. Casual blood glucose is drawn at random and may be affected by many factors such as presence of an IV site near where the sample was drawn, stressful events, or meals. A casual blood glucose value greater than 200mg/dl is significant. Glycosylated hemoglobin (HbA1C) is stated as a percentage and is useful in evaluating long-term glycemic control. the American Diabetes Association (ADA) recommends keeping the HbA1C level below 7%.

(Black & Hawks) d. Casual blood glucose of 178mg/dl

Incorrect Marks for this submission: 0/1. Question 47 Marks: 1 Closed reduction of fractures is followed by immobilization to relieve or reduce pain and to prevent rotation and shearing at the fracture site. Immobilization is most often accomplished through use of a cast or splint. Mandy broke her tibia and cast was applied. Which of the following is an INCORRECT intervention for her? Choose one answer. a. Create a turning schedule for position changes every 3 hours. - Create a turning schedule for position changes every 3 hours. Position change should at least be every 2 hours. Elevate entire extremity higher than the client's heart for the first 24-48 hours to prevent or relieve swelling. Exercise her toes to encourage circulation. Support Mandy's foot with the ankle in 90 degrees of flexion to prevent foot drop. (Black & Hawks)

b. Exercise her toes.

c. Support Mandy's foot with the ankle in 90 degrees of flexion.

Incorrect

Marks for this submission: 0/1. Question 48 Marks: 1

Vitamins are organic compounds or chemicals, found in various food stuff, necessary for the maintenance of normal life. Deficit in this/these vitamins has manifestations of sore reddened mouth, and megaloblastic anemia. Choose one answer. a. Vitamin B12 Cyanocobal amin

b. Vitamin B1 Thiamine

I
d. Elevate entire extremity higher than the client's heart for the first 24-48 hours.

Incorrect

Marks for this submission: 0/1. Question 49 Marks: 1

Acute pharyngitis is the sudden inflammation of the pharynx that is more common in patients younger than 25 years of age. The following can be expected to be manifested by a patient with acute pharyngitis, EXCEPT: Choose one answer. a. Fever

In
c. Folic Acid d. Both A and C - Vitamin B12 - cyanocobalamin and vitamin B1 - Thiamine. Deficit in vitamin B1 is manifested by paresthesias, peripheral neuropathies, mental confusion, heart failure and edema. (Black & Hawks) b. Swelling in the uvula

Incorrect

Marks for this submission: 0/1. Question 50 Marks: 1

Normal cranial nerve reflexes require an appropriately received stimulus that produces an appropriate response. You are a nurse at a neurology unit. The nurse from the shift before you endorsed her assessment that Jimmy, admitted due to a traumatic head injury resulting in compression fracture of the supraorbital area, has anesthesia of the cheek and upper lip. Recalling the cranial nerves, you think that most likely this is caused by an injury to which cranial nerve? Choose one answer. a. CN VII - Facial

I
c. Rhinorrhea d. Swollen lymphoid follicles flecked with white-purple exudates.

- Rhinorrhea. Most cases of acute pharyngitis are caused by viral infections, though sometimes this can also be caused by bacteria. The body responds by triggering an inflammatory response in the pharynx. This results in pain, fever, vasodilation, edema, and tissue damage, manifested by redness, swelling in the tonsillar pillar, uvula, and soft palate. A creamy exudate may be present on the tonsillar pillars. (Brunner)

Marks: 1

A peak expiratory flow rate meter is used to provide an objective measure of the clients peak expiratory flow. A nurse is teaching a

In
b. CN VI Abducen s c. CN IV Trochlea r d. CN V - - trigeminal. The trigeminal nerve is responsible for the sensation Trigemin over the entire face and the innervation of muscles of mastication al (Bullock). CN IV - Trochlear and CN VI - Abducens both have motor components. Together with CN III they coordinate to control eye movements in all six cardinal directions of gaze. The motor division of CN VII - Facial innervates muscles controlling facial expression while the sensory division includes sense of taste on the anterior 2/3 of the tongue. (Black & Hawks)

client with asthma how to perform a peak expiratory flow rate measurement. Which of the following statements made by the client indicates understanding of the health teaching? Choose one answer. a. "I should inhale as normally as possible." b. "I will not completely cover the peak flow meter with my mouth." c. "I will make sure that I will exhale as slowly as possible." d. "I will take note of the final position in the indicator." Question 2 Marks: 1 An enema is a solution introduced into the rectum and the large intestine to distend the intestine and sometimes to irritate the intestinal mucosa, thereby increasing peristalsis and the excretion of feces and flatus. A nurse is preparing to administer a cleansing enema. The nurse positions the client in the: Choose one answer. a. Right lateral position with the left leg acutely flexed b. Dorsal recumbent position c. Left lateral position with the right leg acutely flexed d. Lithotomy position Question 3 Marks: 1 Frequently, nurses assess a specific body area instead of the entire body, especially when a certain health condition calls for it. You are a nurse in the emergency department and a client was admitted due to head injury. What is your PRIORITY physical assessment for this patient? Choose one answer. a. Assess peripheral perfusion of toes, capillary blanch test, pedal pulses and vital signs. b. Inspect the appearance of the head and skull. Assess vital signs.

c. Assess apical pulse and note any arrhythmias. d. Assess level of consciousness using the Glasgow Coma Scale, the pupils for reaction to light and accommodation, and vital signs. Question 4 Marks: 1 It is an ethical responsibility of the nurse to ensure that every human being is respected and must always work for the client's best interest. Your patient was recently diagnosed with terminal cancer. She then asks you, "Am I going to die soon?". Your most appropriate response would be: Choose one answer. a. "The doctors are doing all they can so that you will live a longer life. Don't worry about it." b. "Well, dying is inevitable. All of us die anyway." c. "What did your doctor tell you about your diagnosis?" d. "You seem concerned. What is it about dying that you want to talk about?" Question 5 Marks: 1 A closed bladder irrigation system provides for frequent intermittent irrigation or continuous irrigation without disruption of the sterile catheter system through use of a three-way catheter, as seen in clients after genitourinary surgery. A client with benign prostatic hyperplasia undergoes transurethral resection of the prostate (TURP). The nurse orders which of the following solutions from the pharmacy so it is available postoperatively for continuous bladder irrigation (CBI)? Choose one answer. a. Sterile Dakins solution b. Sterile water with 5% dextrose c. Sterile 0.9% NSS d. Sterile water Question 6

Marks: 1 Physicians often order a therapy (e.g., a medication) for a client by telephone. Particularly, a telephone order involves: Choose one answer. a. No liability on the part of the nurse taking a telephone order b. Clarification, accuracy, and verification c. Use in only an acute emergency situation d. A physician giving any health care worker an order via the telephone Question 7 Marks: 1 Instillations or irrigations of the external auditory canal are referred to as otic and are generally carried out for cleaning purposes. You are a nurse preparing to administer ear drops to a 8-month-old infant. You administer the medication by: Choose one answer. a. Pulling down and back on the auricle and direct the solution onto the eardrum b. Pulling up and back on the ear lobe and direct the solution toward the wall of the canal c. Pulling down and back on the ear lobe and direct the solution toward the wall of the canal d. Pulling up and back on the auricle and direct the solution onto the eardrum Question 8 Marks: 1 Nursing care focuses on prevention, restoration of health, and maintaining wellness, and because of this, nursing diagnoses are formulated that represent these different spheres of nursing care. Nurses also help to manage care where collaboration with the physician is necessary. Which type of nursing diagnosis is most appropriate for an elderly woman in a nursing home who is thin, eats poorly, is confined to bed, is not using pressure-relieving devices, and sometimes wets the bed at night; however, skin is observed to be intact during assessment of the nurse?

Choose one answer. a. Actual nursing diagnosis b. Risk nursing diagnosis c. Collaborative problem d. Wellness nursing diagnosis Question 9 Marks: 1 Nurses must understand the legal limits influencing their daily practice. This, coupled with good judgment and sound decision making, ensures safe and appropriate nursing care. The physician places an order of "NPO after midnight" for a patient in the pay ward who is scheduled for a surgery the following day. During the morning of the surgery, the patient eats breakfast. The surgery is canceled, thus the patient must extend his hospital stay. The client is disappointed and he insists that he will not pay the extension of his stay because of this error. In situations such as this: Choose one answer. a. The client is responsible for the hospital bill and he must pay b. The physician should have written the order more clearly c. These kinds of things happen, and hospital personnel are not responsible unless there is gross negligence d. A full and complete explanation of tests and treatments is the right of the client Question 10 Marks: 1 Bioethical principles are statements about broad, general, philosophical concepts that guide one's particular actions. An earthquake just hit the nearby area where you work. Services from your hospital were called to help the casualties of the calamity. When you arrived in the area, you began to classify the victims according to their health conditions and provided prompt interventions to those who needed it most. In emergency cases like these, which of the following bioethical principles most likely be applied? Choose one answer.

a. Veracity b. Justice c. Autonomy d. Non-maleficence Question 11 Marks: 1 Jaundice is the yellowish discoloration of the integumentary system associated with some health conditions (e.g., liver diseases, destruction of red blood cells). The nurse is providing care for a darkskinned client with hemolytic anemia. The nurse can best assess the presence of jaundice in the client's: Choose one answer. a. Hard palate b. Palm of the hand c. Sole of the feet d. Sclera Question 12 Marks: 1 Nursing theories have been developed throughout the past years to provide framework in the delivery of care to various health settings. Which nursing theorist stated that care is the essence of nursing but it varies among cultures in its expressions, processes, and patterns? Choose one answer. a. Madeleine Leininger b. Betty Neuman c. Jean Watson d. Sister Callista Roy Question 13 Marks: 1 Clinical expertise takes time and commitment. According to Patricia

Benner (1984), an expert nurse passes through five levels of proficiency when acquiring and developing generalist or specialized nursing skills. Nurse Ruth has worked as an operating room nurse for 2 years, however, now she decided to pursue a career as a homehealth nurse. Nurse Ruth is considered: Choose one answer. a. Novice b. Advanced Beginner c. Competent d. Expert e. Proficient Question 14 Marks: 1 Nasogastric tubes are used for feeding clients who have adequate gastric emptying, and who require short-term feedings. The attending physician ordered to discontinue the nasogastric tube of a client. After explaining the procedure to the client, the nurse positions him to a semi-Fowler's position, places a towel across the chest, clears the tube with normal saline, clamps the tube, and removes the tube: Choose one answer. a. After the client takes a deep breath and holds it b. After expiration, but before inspiration c. As the client inhales d. As the client exhales Question 15 Marks: 1 In planning for a conduct of a study, be sure to first examine the literature for information about previous studies relevant to the clinical problem. Which of the following is NOT a purpose of reviewing related literature? Choose one answer. a. Manipulation of the data collection tool used in a research study

b. Identification of a research problem c. Orientation to what is known and not known about an area of inquiry d. Determination of any gaps or inconsistencies in a body of knowledge Question 16 Marks: 1 Determining the different variables in research is an important task of the researcher. Nurse Mary wants to do a qualitative research. Which of the following is a good example of a qualitative variable? Choose one answer. a. Burnout experience of ICU nurses b. Blood pressure measurement of diabetic patients in an urban community c. Level of self-esteem of pediatric cancer patients d. Rate of nosocomial infection in a medical ward Question 17 Marks: 1 Because of increasing educational opportunities for nurses, the growth of nursing as a profession, and a greater concern for job enrichment, the nursing profession offers expanded roles and different kinds of career opportunities. Nurse Christine works in the community and provides primary, ambulatory care to adults with nonemergent acute or chronic illnesses. She works collaboratively with the rural health physicians in cases of referrals. Nurse Christine practices what expanded career role of nurses? Choose one answer. a. Nurse practitioner b. Clinical nurse specialist c. Nurse administrator d. Nurse educator Question 18 Marks: 1

It is significant for the nurses to understand legal responsibilities as it applies to nursing practice. A neonatal ICU nurse was responsible for the death of a 2-day-old newborn after she administered the wrong medication which proved to be lethal. The nurse will most likely be charged of the crime of: Choose one answer. a. Parricide b. Murder c. Homicide d. Infanticide Question 19 Marks: 1 The pulse rate may be measured in nine sites all over the body: temporal, carotid, apical, brachial, radial, femoral, popliteal, posterior tibial and dorsalis pedis. The radial site is most commonly used in adults because it is easily found in most people and is readily accessible; however, there are some situations wherein other sites are more preferable. Which of the following clients meets the criteria for the selection of the apical site for assessment of the pulse instead of the radial pulse? Choose one answer. a. The pulse changes with body position changes. b. It is less than 24 hours since a client's appendectomy operation. c. The client is in shock. d. A client is having a sinus bradycardia according to the client's heart monitor. Question 20 Marks: 1 Exercise involves the active contraction and relaxation of muscles. You are encouraging your client to perform isometric exercises by: Choose one answer. a. Lifting the buttocks off the bed by pushing with the hands against the mattress, and pushing the body to a sitting position.

b. All of the choices c. Lifting weights d. Squeezing a towel or pillow between the knees while at the same time tightening the muscles in the fronts of the thighs by pressing the knees backwards, and holding for several seconds. Question 21 Marks: 1 Assessment is the systematic and continuous collection, validation, and documentation of data. Data collection is the process of gathering information about a client's health status. Which of the following set of data is considered as subjective cues? Choose one answer. a. (+) nausea, (+) cramps, BP=130/80 mmHg b. (+) pain, (+) feelings of anxiety, (+) pruritus c. (+) dizziness, RR=20 breaths/min, (+) pallor d. (+) tinnitus, HR=102 beats/min, (+) ptosis Question 22 Marks: 1 By familiarizing themselves of theories of leadership, nurses can select and adapt the most suitable approach for dealing with different situations. Nurse Florence is the head nurse of a pay ward in a tertiary hospital. She is known as an individual who motivate others by behaving in accordance with values, provides vision that reflects the mutual values of the group and empowers her staff to contribute. Nurse Florence is considered to be a: Choose one answer. a. Charismatic leader b. Transformational leader c. Transactional leader d. Situational leader Question 23

Marks: 1 Healing is a quality of the living tissue and is also referred to as regeneration (renewal) of tissues. Healing can be considered in terms of types of healing, having to do with the caregiver's decision on whether to allow the wound to seal itself or to purposefully close the wound. Your client has a stage 2 pressure ulcer that is already healing. You understand that a pressure ulcer heals by: Choose one answer. a. Primary intention b. Tertiary intention c. Secondary intention d. Delayed primary intention Question 24 Marks: 1 A quality assurance (QA) program is an ongoing, systematic process designed to evaluate and promote excellence in the health care provided to clients. QA requires evaluation of three components of care: structure, process, and outcome. Which of the following statements describe process evaluation? Choose one answer. a. "Suction equipment are adequate in the nursing unit." b. "Checks the client's identification band before giving the medication." c. "Staff nurses are assigned adequately according to the patient acuity." d. "5% of clients who have a colostomy experience an infection that delays discharge" Question 25 Marks: 1 Sources of power are a combination of conscious and unconscious factors that allow an individual to influence others to do as the individual wants. Effective nurses view their ability to understand and use power as a significant part of their responsibilities to patients, their coworkers, the nursing profession, and themselves. Which of the following situations describe legitimate power?

Choose one answer. a. A stoma nurse is always called for if there is a need for colostomy care. b. The charge nurse's instructions regarding patient care are being followed by the newly-hired staff nurse. c. A nurse is respected in the unit because the nursing director of the hospital is her relative. d. The head nurse is being looked up by her staff because she inspires trust and confidence in them. Question 26 Marks: 1 An eye assessment should be carried out as part of the client's initial physical examination. In assessing each pupil's direct and consensual reaction to light, which cranial nerves are being tested for their function? Choose one answer. a. CN IV and V b. CN III and IV c. CN II and IV d. CN II and III Question 27 Marks: 1 Nursing interventions include both direct and indirect care, as well as nurse-initiated, physician-initiated, and other provider-initiated treatments. The attending physician wrote on your client's chart the order, "Progressive ambulation, as tolerated". Which of the following nursing interventions is NOT applicable to the physician's order? Choose one answer. a. Check pulse before and after ambulating. Do not continue if pulse is > 110 beats/min. b. None of the above. c. Dangle patient's legs at bedside for 5 minutes, 12 h post-op.

d. Encourage standing at bedside 24 h post-op; observe for pallor, dizziness, and fatigue. Question 28 Marks: 1 An intradermal (ID) injection is the administration of a drug into the dermal layer of the skin just beneath the epidermis. A nurse is preparing to administer a skin test to a client to determine if the client is allergic to the prescribed antibiotic. The nurse determines that which area is most appropriate for injection of the medication? Choose one answer. a. Dorsal aspect of the upper arm that has a small amount of hair b. Inner aspect of the forearm that is close to a burn scar c. Inner aspect of the forearm that is not heavily pigmented d. Dorsal aspect of the upper arm near a mole Question 29 Marks: 1 Fluid volume excess occurs when the body retains both water and sodium in similar proportions to normal extracellular fluid. A client with nephrotic syndrome is ordered to be on a fluid restriction of 1800 ml per day. The nurse best plans to assist the client with maintaining the fluid restriction by: Choose one answer. a. Removing the water pitcher from the bedside b. Using mouthwash with alcohol for mouth care. c. Asking the client to calculate the IV fluids into the total daily allotment d. Prohibiting beverages with sugar to minimize thirst Question 30 Marks: 1 Theories of management do not remain static. Since the introduction of the earliest principles of scientific management nearly a century ago, management thought has been marked by constant change. This management theorist is considered the "Father of Scientific

Management" and he applied the principles of observation, measurement, and scientific comparison to determine the most efficient way to accomplish a task. Who is this theorist? Choose one answer. a. Frederick Taylor b. James Mooney c. Henri Fayol d. Max Weber Question 31 Marks: 1 Assessment of the abdomen involves all four methods of examination. Which of the following is the proper order of examining the abdomen? Choose one answer. a. Inspection - percussion - auscultation - palpation b. Inspection - auscultation - palpation - percussion c. Inspection - auscultation - percussion - palpation d. Inspection - percussion - palpation - auscultation Question 32 Marks: 1 Priority setting is part of the planning phase wherein there is the establishment of a preferential sequence for addressing nursing diagnoses and interventions. You are a staff nurse at a charity male medical ward in a tertiary hospital and you are assigned 10 patients for the morning shift. Which of these patients require your most immediate intervention? Choose one answer. a. A 65-year-old with septic encephalopathy and is on mechanical ventilation. b. A 45-year-old with diabetes who has a wound on his left foot in need of wound dressing. c. A 73-year-old who is already for discharge and is in need of home

instructions for his medication regimen d. A 28-year-old with nephrotic syndrome who has generalized edema and is reading the newspaper. Question 33 Marks: 1 In health care delivery, basic ethical principles assist the health professionals to determine the right or wrong in regard to value issues involving the pursuit of health, alleviation of suffering, and assisting patients toward peaceful death. Nurse Mariel is conducting a research and she is making sure that the participants in her study are not exposed to any potential harm or risk. She is applying the bioethical principle of: Choose one answer. a. Beneficence b. Right to privacy c. Autonomy d. Non-maleficence Question 34 Marks: 1 Informed consent is an agreement by a client to accept a course of treatment or a procedure after being provided complete information, including the benefits and risks of treatment, alternatives to the treatment, and prognosis if not treated by a health care provider. When obtaining an informed consent for surgery, initially the nurse should: Choose one answer. a. Explain the risks and benefits involved in the surgery b. Witness the signing of the patient of the informed because this is what the nurses signature documents c. Evaluate if the clients knowledge level is sufficient to give consent for the surgery to be performed d. Explain that obtaining the signature is routine for any surgery Question 35

Marks: 1 An object is sterile only when it is free of all microorganisms. It is wellknown that sterile technique is practiced in operating rooms and special diagnostic areas, while it is also employed for many procedures in the general care area. Which of the following situations break the principle of surgical asepsis? Choose one answer. a. A nurse considers all objects that are still in her vision are sterile. b. A scrub nurse sees the surgeon touch an unsterile part of the operating area while wearing gloves, however she does nothing because she assumed that the surgeon noticed this. c. A nurse was not sure that the pair of gloves to be used for wound care were sterile. She opened a new pack of sterile gloves to use. d. A nurse with a mild upper respiratory infection scrubbed in for a surgery. She made sure she donned a mask before entering the OR. Question 36 Marks: 1 Researchers should ensure that their research is not more intrusive than it needs to be and that the participants' privacy is maintained throughout the study. Which among the following situations best describes anonymity? Choose one answer. a. The data gathered from the interview will use general terms in describing the participants in the study. b. The setting of the research study will not be revealed in the research paper. c. Coded questionnaires are used to gather data for the study. d. Research assistants were employed to gather data for the study. Question 37 Marks: 1 The Nurses' Code of Ethics is a formal statement of a group's ideals and values. It is a set of ethical principles that is shared by members of the group, reflects their moral judgments over time, and serves as a standard for their professional actions. Which of the following

statements reflect the way of conduct applied by a nurse when dealing with his/her co-workers? Choose one answer. a. The nurse considers the individuality and totality of clients when administering care. b. The nurse must contribute to the professional growth and development of other members of the health care team. c. The nurse must not demand and receive any commission, fee on the emolument for recommending or referring a patient to another health care worker. d. The nurse must know his/her responsibilities in the practice of his/her profession as provided by the Nursing Law. Question 38 Marks: 1 Nurses require knowledge of the most common diagnostic tests because one primary role of the nurse is to teach the client and family or significant other how to prepare for the test and the care that may be required following the test. A client is to undergo angiography. Which of the following nursing interventions is unnecessary in preparation for the test? Choose one answer. a. Obtain an informed consent from the client. b. Inform patient of the expected duration of the procedure and advise that it will involve lying on a hard table for more than 3 hours. c. Assess for allergy to iodine and seafood d. Instruct patient to fast, usually for 8 to 12 hours, before the procedure. Question 39 Marks: 1 A blood transfusion is the introduction of whole blood or whole blood components into the venous circulation. There is an order in your client's chart to infuse a unit of blood. You check your clients intravenous line to make sure that the gauge of the intravenous catheter is at least: Choose one answer.

a. gauge 18 b. gauge 23 c. gauge 24 d. gauge 14 Question 40 Marks: 1 Republic Act No. 9173, otherwise known as the Philippine Nursing Act of 2002 was implemented to define the scope of nursing practice, licensing requirements, and standards of nursing care. It also defines the powers and duties of the Board of Nursing (BON). Which of the following statements is NOT part of the BON's power and duty? Choose one answer. a. Close any school of nursing that is not meeting the expected standards of quality nursing education. b. Conduct the licensure examination for nurses. c. Revoke certificates of registration for the practice of nursing. d. Investigate nurse practitioners who were complained for unethical and unprofessional conduct. Question 41 Marks: 1 Nurses assume a number of roles when they provide care to clients. You are the nurse of a hypertensive client in the community setting. You assist her in modifying her lifestyle habits including diet, exercise and activity to prevent development of complications of hypertension. By this, you are assuming the role of: Choose one answer. a. Caregiver b. Change agent c. Counselor d. Client advocate Question 42

Marks: 1 The purpose of the research design is to maximize control over factors that can interfere in the validity of the findings. A team of nurseresearchers wants to study the factors contributing to the onset of the irritable bowel syndrome (IBS). They compared samples of women with and without IBS in terms of their history of sexual and physical abuse. The study found out that abusive experiences were more prevalent among women with IBS. Which type of research design was most likely used in the given research study? Choose one answer. a. Prospective design b. Crossover design c. Ex post facto d. Retrospective design Question 43 Marks: 1 Accurate diagnoses of client problems are necessary so that appropriate interventions to address the problems are initiated. A client who will be undergoing mastectomy the following day expresses sadness about losing her breast. Based on this information, the nurse would identify that the client is at risk for the nursing diagnosis of: Choose one answer. a. Anticipatory grieving b. Ineffective coping c. Fear d. Disturbed body image Question 44 Marks: 1 Many factors can facilitate or hinder learning by a client. The nurse should be aware of these factors, particularly when available teaching time is limited. A patient underwent surgery for creation of a colostomy. Three (3) days post-op, he stated, "Nurse, will I be able to clean my colostomy bag by myself? I can do it now. I do not experience any more pain as I have experienced in the past 2 days"

The patient demonstrates: Choose one answer. a. Active involvement b. Motivation to learn c. Expression of relevance d. Readiness to learn Question 45 Marks: 1 Republic Act No. 9173, otherwise known as the Philippine Nursing Act of 2002, also defines the requirement for inactive nurses when they want to return to practice. Nurse Lyn has not practiced the profession for five (5) consecutive years and she wants to practice again. What will be required of her? Choose one answer. a. She will have to take a special type of oral and practical examination to practice again. b. She is required to undergo one (1) month of didactic training and three (3) months of practicum. c. She is required to undergo three (3) months of didactic training and one (1) month of practicum. d. She is required to undergo two (2) months of didactic training and two (2) months of practicum. Question 46 Marks: 1 Bathing removes accumulated oil, perspiration, dead skin cells, and some bacteria. In addition to cleaning the skin, bathing also stimulates circulation and produces a sense of well-being. Nurse Lara is about to perform a bed bath on Mrs. A, who is on strict bed rest. In order to promote circulation, the nurse bathes the client extremities by using: Choose one answer. a. Firm circular strokes from proximal to distal areas b. Short, patting strokes from distal to proximal areas

c. Long, firm strokes from distal to proximal areas d. Long, firm strokes from proximal to distal areas Question 47 Marks: 1 To ensure that nursing care is provided to patients, the work must be organized. A care delivery model organizes the work of caring for patients. The nurse manager and staff of a 25-bed medical unit have decided to change the client care delivery model from primary nursing to team nursing. The nurse manager states which of the following statements for the administration that is most appropriate to support this change? Choose one answer. a. The staffing goal is to have four groups, with a total of 3 to 4 RNs, 2 to 3 nursing assistants and 2 to 3 utility workers per shift. b. "One RN will serve as the admission nurse, one will be the medication nurse, and the charge or head nurse will make the decisions." c. "An RN is responsible for the total care of her assigned patients for the shift." d. The registered nurses on the unit will be assigned 24 hours of responsibility for client care planning. Question 48 Marks: 1 Nursing practice is governed by many legal concepts and it is important for nurses to know the basics of legal concepts, because nurses are accountable for their professional judgments and actions. A nurse is conducting health teaching to a parent group. When teaching about child abuse, the nurse tells the parents that the best legal definition of assault is: Choose one answer. a. Threats to do bodily harm to the person by another person b. A legal wrong committed by one person against the property of another c. A legal wrong committed against the public and punishable by law through the state and courts

d. The application of force to another person without lawful justification Question 49 Marks: 1 Magnetic Resonance Imaging (MRI) is a noninvasive diagnostic scanning technique in which the client is placed in a magnetic field. Which of the following statements made by the client who is to undergo MRI needs further instruction from the nurse? Choose one answer. a. "I will be able to talk with the technician during the procedure." b. "I will remove all my jewelry before the procedure." c. "I must lie very still while I go into the machine." d. "I am afraid because I will be exposed to strong radiation while inside the machine." Question 50 Marks: 1 Because nursing research usually focuses on humans, a major nursing responsibility is to be aware of and to advocate on behalf of client's rights. Nurse Gerard is conducting a research regarding the home conditions of families in a slum area and how these affect their health beliefs. However, when these subjects refuse to participate in the study, he maligns and threatens them. What human right as a research subject is he violating? Choose one answer. a. Right to protection from discomfort or harm b. Right to fair treatment c. Right to anonymity and confidentiality d. Right to privacy

1 Marks: 1 A peak expiratory flow rate meter is used to provide an objective

measure of the clients peak expiratory flow. A nurse is teaching a client with asthma how to perform a peak expiratory flow rate measurement. Which of the following statements made by the client indicates understanding of the health teaching? Choose one answer. a. "I should inhale as normally as possible."

b. "I will not completely cover the peak flow meter with my mouth."

c. "I will make sure that I will exhale as slowly as possible."

d. "I will take note of the final position in the indicator."

- "I will take note of the final position in the indicator". A peak expiratory flow rate meter is used to provide an objective measure of the clients peak expiratory flow. The client is instructed to take the deepest possible breath, form a tight seal around the mouthpiece with the lips, and exhale forcefully and rapidly. The final position of the indicator is recorded to measure the point of highest flow during maximal expiration (Saunders NCLEX, Potter & Perry).

Incorrect Marks for this submission: 0/1.

Question 2 Marks: 1 An enema is a solution introduced into the rectum and the large intestine to distend the intestine and sometimes to irritate the intestinal mucosa, thereby increasing peristalsis and the excretion of feces and flatus. A nurse is preparing to administer a cleansing enema. The nurse positions the client in the: Choose one answer. a. Right lateral position with the left leg acutely flexed

b. Dorsal recumbent position

c. Left lateral position with the right leg acutely flexed

- left lateral position with the right leg as acutely flexed as possible. The sigmoid and descending colon are located on the left side. Therefore, the left lateral position uses gravity to facilitate the flow of solution into the sigmoid and descending colon. Having the right leg acutely flexed provides for adequate exposure of the anus (Kozier, Saunders NCLEX).

d. Lithotomy position

Incorrect Marks for this submission: 0/1. Question 3 Marks: 1 Frequently, nurses assess a specific body area instead of the entire body, especially when a certain health condition calls for it. You are a nurse in the emergency department and a client was admitted due to head injury. What is your PRIORITY physical assessment for this patient? Choose one answer. a. Assess peripheral perfusion of toes, capillary blanch test, pedal pulses and vital signs.

b. Inspect the appearance of the head and skull. Assess vital signs.

c. Assess apical pulse and note any arrhythmias.

d. Assess level of consciousness using the Glasgow Coma Scale, the pupils for reaction to light and accommodation, and vital signs.

- assess level of consciousness using the Glasgow Coma Scale, the pupils for reaction to light and accommodation, and vital signs. If the client suffered a head injury, assessment of the neurological functioning should be done first. Specific

assessments are made in relation to client complaints, the nurse's own observation of problems, the client's presenting problem, nursing interventions provided, and medical therapies (Kozier). Incorrect Marks for this submission: 0/1. Question 4 Marks: 1 It is an ethical responsibility of the nurse to ensure that every human being is respected and must always work for the client's best interest. Your patient was recently diagnosed with terminal cancer. She then asks you, "Am I going to die soon?". Your most appropriate response would be: Choose one answer. a. "The doctors are doing all they can so that you will live a longer life. Don't worry about it."

b. "Well, dying is inevitable. All of us die anyway."

c. "What did your doctor tell you about your diagnosis?"

d. "You seem concerned. What is it about dying that you want to talk about?"

- "You seem concerned. What is it about dying that you want to talk about?" This will be the most appropriate response because this kind of question invites the client to explore (elaborate, clarify, describe, compare, or illustrate) thoughts or feelings. The other choices (giving false reassurance, making stereotyped comments) reflect nontherapeutic communication that may not be helpful in the expression of feelings, fears, and other thoughts of the clients (Kozier).

Incorrect Marks for this submission: 0/1. Question 5 Marks: 1 A closed bladder irrigation system provides for frequent intermittent irrigation or continuous irrigation without disruption of the sterile catheter system through use of a three-way catheter, as seen in clients after genitourinary surgery. A client with benign prostatic hyperplasia undergoes transurethral resection of the prostate (TURP). The nurse orders which of the following solutions from the pharmacy so it is available postoperatively for continuous bladder irrigation (CBI)? Choose one answer. a. Sterile Dakins solution

b. Sterile water with 5% dextrose

Incorrect

Marks for this submission: 0/1. Question 6 Marks: 1

Physicians often order a therapy (e.g., a medication) for a client by telephone. Particularly, a telephone order involves: Choose one answer.

In
c. Sterile 0.9% NSS - sterile 0.9% NSS. Continuous bladder irrigation is done following TURP using sterile normal saline, which is isotonic solution. Sterile water is not used because the solution could be absorbed systematically, which may cause hemolysis and possibly renal failure. Dakins solution contains hypochlorite and is used only for wound irrigation in selected circumstances. Dextrose-containing solutions are not introduced into the bladder (Potter & Perry, Saunders NCLEX). d. Sterile water a. No liability on the part of the nurse taking a telephone order b. Clarification, accuracy, and -Clarifying for accuracy is important when a registered nurse accepts a physician's order over

verification

the phone or verbally. The order needs to be verified by repeating it clearly and precisely. A telephone order involves a physician stating a prescribed therapy over the phone to a registered nurse. A verbal order may be accepted when there is no opportunity for a physician to write the order, as in emergency situations (e.g., administration of epinephrine during a code). The registered nurse is responsible for writing the order on the physicians order sheet in the client's permanent record and signing it (Potter & Perry).

c. Use in only an acute emergency situation

d. A physician giving any health care worker an order via the telephone

Incorrect Marks for this submission: 0/1. Question 7 Marks: 1 Instillations or irrigations of the external auditory canal are referred to as otic and are generally carried out for cleaning purposes. You are a nurse preparing to administer ear drops to a 8-month-old infant. You administer the medication by: Choose one answer. a. Pulling down and back on the auricle and direct the solution onto the

eardrum

Incorrect

Marks for this submission: 0/1. Question 8 Marks: 1

I
b. Pulling up and back on the ear lobe and direct the solution toward the wall of the canal d. Pulling up and back on the auricle and direct the solution onto the eardrum

- pulling down and back on the ear lobe and c. Pulling down and back on direct the solution toward the wall of the canal. the ear lobe and direct the This technique is used because the external solution toward the wall of auditory canal of children below three (3) years of age differ from that of adults. The external the canal auditory canal of these children is directed upward, while in adults it is an S-shaped structure about 2.5 cm (1 inch) long. Pulling down and back on the ear lobe of the infant straightens the external auditory canal so that the solution can flow the entire length of the canal (Kozier, Saunders NCLEX).

Nursing care focuses on prevention, restoration of health, and maintaining wellness, and because of this, nursing diagnoses are formulated that represent these different spheres of nursing care. Nurses also help to manage care where collaboration with the physician is necessary. Which type of nursing diagnosis is most appropriate for an elderly woman in a nursing home who is thin, eats poorly, is confined to bed, is not using pressure-relieving devices, and sometimes wets the bed at night; however, skin is observed to be intact during assessment of the nurse? Choose one answer. a. Actual nursing diagnosis

b. Risk nursing diagnosis

- risk nursing diagnosis. A risk nursing diagnosis describes a clinical judgment that a problem does not exist, but the presence of risk factors indicates that a problem is likely to develop unless nurses intervene to prevent the problem. An actual nursing diagnosis describes human responses to health conditions/life processes that are present during the time of nursing assessment. A wellness diagnosis is a human response to a state or level of wellness in an individual, family, or community that has the desire to move toward a higher state of being healthy. Collaborative problems are those that require interventions of more than one member of the health care team (Kozier, Harkreader & Hogan).

c. Collaborativ e problem

d. Wellness nursing diagnosis

Incorrect

Marks for this submission: 0/1. Question 9 Marks: 1

Nurses must understand the legal limits influencing their daily practice. This, coupled with good judgment and sound decision making, ensures safe and appropriate nursing care. The physician places an order of "NPO after midnight" for a patient in the pay ward who is scheduled for a surgery the following day. During the morning of the surgery, the patient eats breakfast. The surgery is canceled, thus the patient must extend his hospital stay. The client is disappointed and he insists that he will not pay the extension of his stay because of this error. In situations such as this: Choose one answer.

In
a. The client is responsible for the hospital bill and he must pay b. The physician should have written the order more clearly c. These kinds of things happen, and hospital personnel are not responsible unless there is gross

negligence

Incorrect

Marks for this submission: 0/1. Question 10 Marks: 1

Bioethical principles are statements about broad, general, philosophical concepts that guide one's particular actions. An earthquake just hit the nearby area where you work. Services from your hospital were called to help the casualties of the calamity. When you arrived in the area, you began to classify the victims according to their health conditions and provided prompt interventions to those

I
- a full and complete explanation of tests and treatments is the right of the client. This is negligence and the patient should be informed. An act of negligence can be seen if the following are present: a) duty - the nurse must have (or should have had) a relationship with the client that involves providing care and following an acceptable d. A full and complete standard of care; b) breach of duty - there explanation of tests and must be a standard of care that is expected treatments is the right of the in the specific situation but that the nurse did client not observe; c) foreseeability - a link must exist between the nurse's act and the injury suffered; d) causation - it must be proved that the harm occurred as a direct result of the nurse's failure to meet standard of care, and; e) injury - the client must demonstrate some type of harm or injury (physical, financial, or emotional) (Kozier). The hospital should assume the added expense incurred as a result of negligent actions of its employees. The staff is liable for negligent actions. The physician wrote an appropriate, clear order. The nurse was negligent in carrying it out (Mosby NCLEX).

who needed it most. In emergency cases like these, which of the following bioethical principles most likely be applied? Choose one answer. a. Veracity

- justice. Justice is fair, equitable, and appropriate treatment b. according to what is due or owed to persons. It should be the Justice bioethical principle applied by the health provider during triage. The principle of veracity refers to the practice of telling the truth. Autonomy refers to the right to make one's own decisions. Nurses who follow this principle should recognize that each client is unique. Non-maleficence is the duty to "do no harm". This requires nurses to act in such a way as to avoid causing harm to clients (Kozier). c. Autono my

d. Nonmalefic ence

Incorrect Marks for this submission: 0/1. Question 11 Marks: 1

Jaundice is the yellowish discoloration of the integumentary system associated with some health conditions (e.g., liver diseases, destruction of red blood cells). The nurse is providing care for a darkskinned client with hemolytic anemia. The nurse can best assess the presence of jaundice in the client's: Choose one answer. a. Hard palate

- hard palate. The hard palate is more reliable area to observe jaundice in the dark-skinned client than that of the sclera. Nurses should take care not to confuse jaundice with the normal pigmentation in the sclera of a dark-skinned client (Kozier).

b. Palm of the hand

c. Sole of the feet

d. Sclera

Incorrect Marks for this submission: 0/1. Question 12

Marks: 1 Nursing theories have been developed throughout the past years to provide framework in the delivery of care to various health settings. Which nursing theorist stated that care is the essence of nursing but it varies among cultures in its expressions, processes, and patterns? Choose one answer. a. Madelein e Leininger - Madeleine Leininger. Madeleine Leininger conceptualized the Cultural Care Diversity and Universality Theory which states that care is the essence and the dominant, distinctive, and unifying feature of nursing. She however emphasized that human caring is largely culturally derived and is influenced by elements of social structure. Callista Roy's Adaptation Model focuses on the individual as a biopsychosocial, adaptive system that employs a feedback cycle of input, throughput and output. Betty Neuman's Systems Model is based on the individual's relationship to stress, the reaction to it, and reconstitution factors that are dynamic in nature. Jean Watson's Human Caring Theory believes the practice of caring is central to nursing to preserve human dignity and humanity. This theory includes 10 carative factors that represent both feelings and actions pertaining to the nurse and the client and that include elements to be felt, experienced, expressed, and promoted by every nurse (Kozier, Harkreader & Hogan).

b. Betty Neuman

c. Jean Watson

d. Sister Callista Roy

Incorrect

Marks for this submission: 0/1. Question 13 Marks: 1

Clinical expertise takes time and commitment. According to Patricia Benner (1984), an expert nurse passes through five levels of proficiency when acquiring and developing generalist or specialized nursing skills. Nurse Ruth has worked as an operating room nurse for 2 years, however, now she decided to pursue a career as a homehealth nurse. Nurse Ruth is considered: Choose one answer. a. Novice

In
- novice. A beginning nursing student, or any nurse entering a situation in which there is no previous level of experience is in the novice level. Performance is limited, inflexible, and governed by context-free rules and regulations rather than experience. An advanced beginner is the one who has had some level of experience with the situation. The nurse is able to identify meaningful aspects or principles of nursing care. A competent nurse has 2 or 3 years of experience and is already able to anticipate nursing care and establish long-range goals. A proficient nurse has 3 to 5 years of experience and is able to perceive a client's clinical situation as a whole, assess an entire situation, and can readily transfer knowledge gained from multiple previous experiences to a situation. An expert nurse has a diverse experience and has an intuitive grasp of an existing or potential clinical problem (Kozier, Potter & Perry). b. Advance d Beginne r

Incorrect

Marks for this submission: 0/1. Question 14 Marks: 1

Nasogastric tubes are used for feeding clients who have adequate gastric emptying, and who require short-term feedings. The attending physician ordered to discontinue the nasogastric tube of a client. After explaining the procedure to the client, the nurse positions him to a semi-Fowler's position, places a towel across the chest, clears the tube with normal saline, clamps the tube, and removes the tube:

In
c. Compet ent d. Expert e. Proficien t

Choose one answer. a. After the client - after the client takes a deep breath and holds it. Just takes a deep breath and holds before removing the tube, the client is asked to take a deep breath and hold it. This closes the glottis, thereby it preventing accidental aspiration of any gastric contents. The action is important because the airway is partially occluded during tube removal. The nurse pulls the tube out steadily and smoothly while the client holds the breath (Kozier, Saunders NCLEX). b. After expiration, but before inspiration

c. As the client inhales

d. As the client exhales

Incorrect Marks for this submission: 0/1. Question 15 Marks: 1 In planning for a conduct of a study, be sure to first examine the literature for information about previous studies relevant to the clinical problem. Which of the following is NOT a purpose of reviewing

related literature? Choose one answer. a. Manipulation of the data collection tool used in a research study - manipulation of the data collection tool used in a research study. The review of related literature is done to identify suitable methods of data collection, not manipulate the tool itself. A literature review can also help in the following: a) identification of a research problem or development or refinement of research questions or hypotheses; b) orientation to what is known and not known about an area of inquiry; c) determination of any gaps or inconsistencies in a body of knowledge; d) determination of a need to replicate a prior study in terms of setting or study population; e) identification or development of new or refined clinical interventions to test; f) identification of relevant theoretical or conceptual frameworks; g) identification of experts in the field, and; h) assistance in interpreting study findings and in developing implications and recommendations (Polit & Beck).

b. Identification of a research problem

c. Orientation to what is known and not known about an area of inquiry

d. Determination of any gaps or inconsistencies in a body of knowledge

Incorrect

Marks for this submission: 0/1. Question 16 Marks: 1

Determining the different variables in research is an important task of the researcher. Nurse Mary wants to do a qualitative research. Which of the following is a good example of a qualitative variable? Choose one answer.

I
b. Blood pressure measurement of diabetic patients in an urban community c. Level of self-esteem of pediatric cancer patients

a. Burnout experience - burnout experience of ICU nurses. Qualitative of ICU nurses research is the systematic, subjective methodological approach used to describe life experiences and give them meaning. Quantitative research is the formal, objective, systematic process used to describe variables, test relationships between them, and examine cause-and-effect interactions among variables (Burns & Grove). All the other choices are quantitative variables which will be subjected to quantitative research.

Incorrect

Marks for this submission: 0/1. Question 17 Marks: 1

Because of increasing educational opportunities for nurses, the growth of nursing as a profession, and a greater concern for job enrichment, the nursing profession offers expanded roles and different kinds of career opportunities. Nurse Christine works in the community and provides primary, ambulatory care to adults with nonemergent acute or chronic illnesses. She works collaboratively with the rural health physicians in cases of referrals. Nurse Christine practices what expanded career role of nurses? Choose one answer.

a. Nurse - nurse practitioner. A nurse practitioner is a type of advanced practitione practice nurse (which also includes the clinical nurse specialists, r certified registered nurse anesthetists and nurse-midwives) that provides health care to clients, usually in an outpatient, ambulatory care, or community-based setting. They provide care for clients with complex problems and provide a more holistic approach. A clinical nurse specialist is an advanced practice nurse with nursing expertise in a specialized area of practice and may work in any practice setting. The nurse provides direct client care, educates others, consults, conducts research, and manages care. A nurse educator is employed in nursing programs, at educational institutions, and in hospital staff education. He/She is responsible for classroom and often

I
d. Rate of nosocomial infection in a medical ward

clinical teaching. A nurse administrator manages client care, including the delivery of nursing services (Kozier, Potter & Perry). b. Clinical nurse specialist

c. Nurse administrat or

d. Nurse educator

Incorrect Marks for this submission: 0/1. Question 18 Marks: 1 It is significant for the nurses to understand legal responsibilities as it applies to nursing practice. A neonatal ICU nurse was responsible for the death of a 2-day-old newborn after she administered the wrong medication which proved to be lethal. The nurse will most likely be charged of the crime of: Choose one answer. a. Parrici

de

Incorrect

Marks for this submission: 0/1. Question 19

In
b. Murde r c. Homic ide

- infanticide. Infanticide is the killing of a child who was less than three days (72 hours) of age. Murder is the killing of another person on which there is treachery, taking advantage of superior strength, with the aid of armed men, or employing means to weaken the d. Infanti defense; in consideration of a price, reward, or promise; by means of cide inundation, fire, poison, or with the use of other means involving great waste and ruin; with evident premeditation; and, with cruelty, by deliberately and inhumanly augmenting the suffering of the victim, or outraging or scoffing at his person or corpse. Parricide is committed by any person who shall kill his father, mother, or child, whether legitimate or illegitimate, or a legitimate other ascendant or other descendant, or the legitimate spouse. Homicide is committed provided that the killing was not attended by any of the qualifying circumstances of murder, or by that of parricide or infanticide (Bellosillo).

Marks: 1 The pulse rate may be measured in nine sites all over the body: temporal, carotid, apical, brachial, radial, femoral, popliteal, posterior tibial and dorsalis pedis. The radial site is most commonly used in adults because it is easily found in most people and is readily accessible; however, there are some situations wherein other sites are more preferable. Which of the following clients meets the criteria for the selection of the apical site for assessment of the pulse instead of the radial pulse? Choose one answer. a. The pulse changes with body position changes.

b. It is less than 24 hours since a client's appendectomy operation.

c. The client is in shock.

d. A client is having a sinus bradycardia according to the client's heart monitor.

- a client is having a sinus bradycardia according to the client's heart monitor. The apical rate would confirm the rate and determine the actual cardiac rhythm for a client with an abnormal rhythm; a radial pulse would only reveal the heart rate and suggest an arrhythmia. For clients in shock, the carotid or femoral pulse should be used because these determine circulation to the brain and to a leg, respectively. The radial pulse is

adequate for determining rate in orthostatic heart rate. The radial pulse is also appropriate for routine postoperative vital sign checks for clients with regular pulses (Kozier). Incorrect Marks for this submission: 0/1. Question 20 Marks: 1 Exercise involves the active contraction and relaxation of muscles. You are encouraging your client to perform isometric exercises by: Choose one answer. a. Lifting the buttocks off the bed by pushing with the hands against the mattress, and pushing the body to a sitting position.

b. All of the choices

c. Lifting weights

d. Squeezing a towel or pillow between the knees while at the same time tightening the muscles in the fronts of the thighs by pressing the knees backwards, and holding for several seconds.

- squeezing a towel or pillow between the knees while at the same time tightening the muscles in the fronts of the thighs by pressing the knees backwards, and holding for several seconds. Isometric (static or setting)

exercises are those in which there is muscle contraction without moving the joint (muscle length does not change). Isotonic (dynamic) exercises are those in which the muscles are shortened to produce muscle contraction and active movement (e.g., lifting the buttocks off the bed by pushing with the hands against the mattress, and pushing the body to a sitting position). Isokinetic (resistive) exercises are those that involve muscle contraction or tension against resistance; thus, they can either be isotonic or isometric (e.g., lifting weights) (Kozier). Incorrect Marks for this submission: 0/1. Question 21 Marks: 1 Assessment is the systematic and continuous collection, validation, and documentation of data. Data collection is the process of gathering information about a client's health status. Which of the following set of data is considered as subjective cues? Choose one answer. a. (+) nausea, (+) cramps, BP=130/80 mmHg

b. (+) pain, (+) feelings of anxiety, (+) pruritus

- (+) pain, (+) feelings of anxiety, (+) pruritus. Subjective data, also referred to as symptoms or covert data, are apparent only to the person affected and can be described and verified only by that person. It includes the client's sensations, feelings, values, beliefs, attitudes, and perception of personal health status and life

situation. Objective data, on the other hand, also referred to as signs or overt data, are detectable by an observer or can be measured or tested against an accepted standard. They can be seen, heard, felt, or smelled, and they are obtained by observation or physical examination (Kozier). c. (+) dizziness, RR=20 breaths/min, (+) pallor

d. (+) tinnitus, HR=102 beats/min, (+) ptosis

Incorrect Marks for this submission: 0/1. Question 22 Marks: 1 By familiarizing themselves of theories of leadership, nurses can select and adapt the most suitable approach for dealing with different situations. Nurse Florence is the head nurse of a pay ward in a tertiary hospital. She is known as an individual who motivate others by behaving in accordance with values, provides vision that reflects the mutual values of the group and empowers her staff to contribute. Nurse Florence is considered to be a: Choose one answer. a. Charismati

c leader

Incorrect

Marks for this submission: 0/1. Question 23 Marks: 1

In
- transformational leader. The transformational leadership theory is based on the idea of empowering others to engage in pursuing b. a collective purpose by working together to achieve a vision of a Transform preferred future. This kind of leadership can influence both the ational leader and the follower into a higher level of conduct and leader achievement that transforms them both. A charismatic leader has an inspirational quality that promotes an emotional connection from followers. A transactional leader is the one who identifies needs of followers and provides rewards to meet those needs in exchange for expected performance. The situational leadership theory predicts the most appropriate leadership style from the level of maturity of the followers (Kelly-Heidenthal, Marriner-Tomey). c. Transactio nal leader d. Situational leader

Healing is a quality of the living tissue and is also referred to as regeneration (renewal) of tissues. Healing can be considered in terms of types of healing, having to do with the caregiver's decision on whether to allow the wound to seal itself or to purposefully close the wound. Your client has a stage 2 pressure ulcer that is already healing. You understand that a pressure ulcer heals by: Choose one answer. a. Primary intention

b. Tertiary intention

- secondary intention healing. Primary intention healing occurs where the tissue surfaces have been approximated (closed) and there is minimal tissue loss and is characterized by the c. formation of minimal granulation tissue and scarring. A closed Secondary surgical incision heals by primary intention. A wound that is intention extensive and involves considerable tissue loss, and in which edges cannot or should not be approximated, heals by secondary intention healing. Compared to primary intention, secondary intention repairs longer, the scarring is greater and there is a greater susceptibility to infection. Wounds that are left open for 3 to 5 days to allow edema or infection to resolve or exudate to drain are then closed with sutures, staples or adhesive skin closures, heal by tertiary intention, also called delayed primary intention (Kozier). d. Delayed primary intention

Incorrect

Marks for this submission: 0/1. Question 24 Marks: 1

A quality assurance (QA) program is an ongoing, systematic process designed to evaluate and promote excellence in the health care provided to clients. QA requires evaluation of three components of care: structure, process, and outcome. Which of the following statements describe process evaluation? Choose one answer.

In
a. "Suction equipment are adequate in the nursing unit." - "checks the client's identification band before giving the medication." Process evaluation b. "Checks the client's focuses on how the care was given. Process identification band before standards focus on the manner in which the giving the medication." nurse uses the nursing process. Structure evaluation focuses on the setting in which care is given. Structural standards describe desirable environmental and organizational characteristics that influence care, such as equipment and staffing. Outcome evaluation focuses on demonstrable changes in the client's health status as a result of nursing care (Kozier). c. "Staff nurses are assigned adequately according to the patient

acuity."

Incorrect

Marks for this submission: 0/1. Question 25 Marks: 1

Sources of power are a combination of conscious and unconscious factors that allow an individual to influence others to do as the individual wants. Effective nurses view their ability to understand and use power as a significant part of their responsibilities to patients, their coworkers, the nursing profession, and themselves. Which of the following situations describe legitimate power? Choose one answer.

I
d. "5% of clients who have a colostomy experience an infection that delays discharge" a. A stoma nurse is always called for if there is a need for colostomy care. b. The charge nurse's instructions regarding patient care are being followed by the newly-hired staff nurse.

- the charge nurse's instructions regarding patient care are being followed by the newlyhired staff nurse. Legitimate power is power derived from the position a nurse holds in a group, and it indicates the nurse's degree of

authority. It gives the manager the right to influence and the staff associate an obligation to accept that influence. Power derived from how much others respect and like any individual, group, or organizations referred to as referent power. The extent to which nurses are connected with others having power is called connection power. Power derived from the knowledge and skills nurses possess is referred to as expert power (Kelly-Heidenthal, Marriner-Tomey). c. A nurse is respected in the unit because the nursing director of the hospital is her relative.

d. The head nurse is being looked up by her staff because she inspires trust and confidence in them.

Incorrect Marks for this submission: 0/1. Question 26 Marks: 1 An eye assessment should be carried out as part of the client's initial physical examination. In assessing each pupil's direct and consensual reaction to light, which cranial nerves are being tested for their function? Choose one answer.

a. CN IV and V

In

Incorrect

Marks for this submission: 0/1. Question 27 Marks: 1

Nursing interventions include both direct and indirect care, as well as nurse-initiated, physician-initiated, and other provider-initiated treatments. The attending physician wrote on your client's chart the order, "Progressive ambulation, as tolerated". Which of the following nursing interventions is NOT applicable to the physician's order? Choose one answer.

Co
c. CN II and IV d. CN II and III a. Check pulse before and after ambulating. Do not continue if pulse is > 110 beats/min.

b. CN - cranial nerves III and IV. The direct and consensual responses of III and the pupil are assessed by illuminating one pupil and observing the IV pupillary reactions of the illuminated and non-illuminated pupils. The illuminated pupil should constrict (direct response) and the nonilluminated pupil should also constrict (consensual response). These determine the function of cranial nerve III (oculomotor) and cranial nerve IV (trochlear), which are responsible for the movement of the sphincter of the pupil (Kozier).

Incorrect

Marks for this submission: 0/1. Question 28 Marks: 1

An intradermal (ID) injection is the administration of a drug into the dermal layer of the skin just beneath the epidermis. A nurse is

I
b. None of the above. c. Dangle patient's legs at bedside for 5 minutes, 12 h post-op. d. Encourage standing at bedside 24 h post-op; observe for pallor, dizziness, and fatigue.

- none of the above. All the choices are appropriate for the physician's order of progressive ambulation. These are dependent interventions and are activities carried out under the physician's orders or supervision, or according to specified routines. The nurse is responsible for assessing the need for, explaining, and administering the medical orders. Nursing interventions may be written to individualize the medical order based on the client's status (Kozier).

preparing to administer a skin test to a client to determine if the client is allergic to the prescribed antibiotic. The nurse determines that which area is most appropriate for injection of the medication? Choose one answer. a. Dorsal aspect of the upper arm that has a small amount of hair

b. Inner aspect of the forearm that is close to a burn scar

- inner aspect of the forearm that is not heavily c. Inner aspect of the pigmented. Intradermal injections are most forearm that is not commonly given in the inner surface of the forearm. heavily pigmented Other sites include the dorsal area of the upper arm or the upper back beneath the scapulae. The nurse finds an area that is not heavily pigmented and is clear of hairy areas or lesions that could interfere with reading the results. Also, avoid using sites that are tender, inflamed and swollen (Kozier, Saunders NCLEX). d. Dorsal aspect of the upper arm near a mole

Incorrect

Marks for this submission: 0/1. Question 29 Marks: 1 Fluid volume excess occurs when the body retains both water and sodium in similar proportions to normal extracellular fluid. A client with nephrotic syndrome is ordered to be on a fluid restriction of 1800 ml per day. The nurse best plans to assist the client with maintaining the fluid restriction by: Choose one answer. - removing the water pitcher from the bedside. The nurse can assist the client in maintaining fluid restriction through various ways. One method is to provide frequent mouth care, however, alcohola. Removing the water based products should be avoided because they pitcher from the bedside tend to dry mucous membranes. The use of ice chips or lip ointments are other nursing interventions that may be helpful to the client on fluid restriction. Beverages that the client enjoys are provided and are not restricted based on sugar content. It is the nurse's responsibility to keep track of the IV fluid intake, not the client's. The water pitcher should be removed from the bedside to help in the client's compliance to the fluid restriction (Kozier, Saunders NCLEX). b. Using mouthwash with alcohol for mouth care.

c. Asking the client to calculate the IV fluids into the total daily allotment

d. Prohibiting beverages with sugar to minimize thirst

Incorrect

Marks for this submission: 0/1. Question 30 Marks: 1

Theories of management do not remain static. Since the introduction of the earliest principles of scientific management nearly a century ago, management thought has been marked by constant change. This management theorist is considered the "Father of Scientific Management" and he applied the principles of observation, measurement, and scientific comparison to determine the most efficient way to accomplish a task. Who is this theorist? Choose one answer.

- Frederick Taylor. Frederick Taylor is considered to be the "Father of Scientific Management" and he advocated that work be studied a. scientifically to determine the "one best way" to perform each Frederic task. Henri Fayol is known as the "Father of the Management k Taylor Process School" and he studied the functions of managers and concluded that management is universal. Max Weber is the "Father of Organization Theory" because of his conceptualization of bureaucracy with emphasis on rules instead of individuals and on competence over favoritism as the most efficient basis for organization. James Mooney believed management to be the technique of directing people and organization the technique of relating functions (Marriner-Tomey, Ultimate Learning Guide). b. James Mooney

c. Henri Fayol

Incorrect

Marks for this submission: 0/1. Question 31 Marks: 1

Assessment of the abdomen involves all four methods of examination. Which of the following is the proper order of examining the abdomen? Choose one answer. a. Inspection percussion auscultation palpation

In
d. Max Weber b. Inspection auscultation palpation percussion

Incorrect

Marks for this submission: 0/1. Question 32 Marks: 1

Priority setting is part of the planning phase wherein there is the establishment of a preferential sequence for addressing nursing diagnoses and interventions. You are a staff nurse at a charity male medical ward in a tertiary hospital and you are assigned 10 patients for the morning shift. Which of these patients require your most immediate intervention? Choose one answer.

In
c. Inspection auscultation percussion palpation -inspection-auscultation-percussion-palpation. The order of an abdominal examination differs slightly from previous assessments, which is normally inspection-palpationpercussion-auscultation. Auscultation is done before palpation and percussion because palpation and percussion cause movement or stimulation of the bowel, which can increase bowel motility and thus heighten bowel sounds, creating false results (Kozier). d. Inspection percussion palpation auscultation a. A 65-year-old with septic encephalopathy and is on mechanical ventilation. - a 65-year-old with septic encephalopathy and is on mechanical ventilation. The client here has an impaired airway function. Lifethreatening problems, such as loss of respiratory or cardiac function, are

designated as high priority. Healththreatening problems, such as acute illness and decreased coping ability, are assigned medium priority because they may result in delayed development or cause destructive physical or emotional changes. A low-priority problem is one that arises from normal development needs or that requires only minimal nursing support (Kozier). b. A 45-year-old with diabetes who has a wound on his left foot in need of wound dressing.

c. A 73-year-old who is already for discharge and is in need of home instructions for his medication regimen

d. A 28-year-old with nephrotic syndrome who has generalized edema and is reading the newspaper.

Incorrect Marks for this submission: 0/1. Question 33 Marks: 1

In health care delivery, basic ethical principles assist the health professionals to determine the right or wrong in regard to value issues involving the pursuit of health, alleviation of suffering, and assisting patients toward peaceful death. Nurse Mariel is conducting a research and she is making sure that the participants in her study are not exposed to any potential harm or risk. She is applying the bioethical principle of: Choose one answer. a. Benefice nce

b. Right to privacy

c. Autono my

d. Nonmalefice - non-maleficence. Non-maleficence is not doing harm or inflicting nce evil on someone, especially physical harm. Non-maleficence is avoiding any intent or cause that will lead to death. Obligations of non-maleficence are obligations of not including harm, and not imposing risks of harm (Kuan).

Incorrect

Marks for this submission: 0/1. Question 34 Marks: 1 Informed consent is an agreement by a client to accept a course of treatment or a procedure after being provided complete information, including the benefits and risks of treatment, alternatives to the treatment, and prognosis if not treated by a health care provider. When obtaining an informed consent for surgery, initially the nurse should: Choose one answer. a. Explain the risks and benefits involved in the surgery

b. Witness the signing of the patient of the informed because this is what the nurses signature documents

c. Evaluate if the clients knowledge level is sufficient - evaluate if the clients knowledge level is to give consent for the sufficient to give consent. Informed consent surgery to be performed means the client must comprehend the surgery, the alternatives, and the consequences. Explaining the risks and benefits involved is not within the nurses scope of practice (Mosby NCLEX). d. Explain that obtaining the signature is routine for any surgery

Incorrect

Marks for this submission: 0/1. Question 35 Marks: 1

An object is sterile only when it is free of all microorganisms. It is wellknown that sterile technique is practiced in operating rooms and special diagnostic areas, while it is also employed for many procedures in the general care area. Which of the following situations break the principle of surgical asepsis? Choose one answer.

I
a. A nurse considers all objects that are still in her vision are sterile. b. A scrub nurse sees the surgeon touch an unsterile part of the operating area while wearing gloves, however she does nothing because she assumed that the surgeon noticed this. - a scrub nurse sees the surgeon touch an unsterile part of the operating area while wearing gloves, however she does nothing because she assumed that the surgeon noticed this. This is considered as a break in the principle of surgical asepsis of "sterile objects become unsterile when touched by unsterile objects". Whenever the sterility of an object is questionable, assume the article is unsterile. Sterile objects that are out of vision or table level are considered unsterile. The practice of refraining from carrying out sterile procedures or wearing masks when a member of the health care team has

a mild upper respiratory infection follows the principle that sterile objects can become unsterile by prolonged exposure to airborne microorganisms (Kozier). c. A nurse was not sure that the pair of gloves to be used for wound care were sterile. She opened a new pack of sterile gloves to use.

d. A nurse with a mild upper respiratory infection scrubbed in for a surgery. She made sure she donned a mask before entering the OR.

Incorrect Marks for this submission: 0/1. Question 36 Marks: 1 Researchers should ensure that their research is not more intrusive than it needs to be and that the participants' privacy is maintained throughout the study. Which among the following situations best describes anonymity? Choose one answer. a. The data gathered from the interview will use general terms in describing the participants in the study.

Incorrect

Marks for this submission: 0/1. Question 37 Marks: 1

The Nurses' Code of Ethics is a formal statement of a group's ideals and values. It is a set of ethical principles that is shared by members

I
b. The setting of the research study will not be revealed in the research paper. c. Coded questionnaires are used to gather data for the study. - coded questionnaires are used to gather data for the study. Anonymity refers to the protection of research participants in a study such as that even the researcher cannot link individuals with the information provided. Questionnaires offer the possibility of complete anonymity. Confidentiality is applied wherein there is protection of participants in a study such that individual identities are not linked to information provided and are never publicly divulged (Polit & Beck). d. Research assistants were employed to gather data for the study.

of the group, reflects their moral judgments over time, and serves as a standard for their professional actions. Which of the following statements reflect the way of conduct applied by a nurse when dealing with his/her co-workers? Choose one answer. a. The nurse considers the individuality and totality of clients when administering care.

- the nurse must participate actively in the professional organization for nurses. In dealing with co-workers, the nurse has the perception that he/she and the other members of the health care team are working for the client's best interest. The nurse must also maintain a collegial and collaborative b. The nurse must contribute to the working relationship with colleagues and other health providers. When the professional growth and development of other members of nurse considers the individuality and totality of clients when administering the health care team. care, she is dealing with the people to whom she is serving. When the nurse knows his/her responsibilities in the practice of his/her profession as provided by the Nursing Law, she is demonstrating good conduct in terms of her practice. When the nurse does not demand and receive any commission, fee on the emolument for recommending or referring a patient to another health care worker, she is dealing with her responsibility to the society and environment (Ultimate Learning Guide). c. The nurse must not demand and receive any commission, fee on the emolument for recommending or referring a patient to another health

care worker.

Incorrect

Marks for this submission: 0/1. Question 38 Marks: 1

Nurses require knowledge of the most common diagnostic tests because one primary role of the nurse is to teach the client and family or significant other how to prepare for the test and the care that may be required following the test. A client is to undergo angiography. Which of the following nursing interventions is unnecessary in preparation for the test? Choose one answer.

I
d. The nurse must know his/her responsibilities in the practice of his/her profession as provided by the Nursing Law. a. Obtain an informed consent from the client. b. Inform patient of the expected - inform patient of the expected duration duration of the procedure and of the procedure and advise that it will advise that it will involve lying on involve lying on a hard table for more a hard table for more than 3 than 3 hours. Angiography is a diagnostic hours. technique wherein a contrast agent is injected into the vascular system to

outline the heart and blood vessels. It is usually done in less than 2 hours. It is an invasive technique, therefore it requires an informed consent. A radiopaque dye is injected into the vessels to be examined and this dye is usually iodine-based. Allergy to iodine or seafood is assessed to prevent development of anaphylaxis during the procedure. A client is instructed to fast beforehand to prevent aspiration during the procedure because the client is sedated (Kozier, Brunner). c. Assess for allergy to iodine and seafood

d. Instruct patient to fast, usually for 8 to 12 hours, before the procedure.

Incorrect Marks for this submission: 0/1. Question 39 Marks: 1 A blood transfusion is the introduction of whole blood or whole blood components into the venous circulation. There is an order in your client's chart to infuse a unit of blood. You check your clients intravenous line to make sure that the gauge of the intravenous catheter is at least: Choose one answer. a. - gauge 18. Blood is usually administered through a #18- to #20gaug gauge intravenous needle or catheter; using a smaller needle may

e 18

Incorrect

Marks for this submission: 0/1. Question 40 Marks: 1

Republic Act No. 9173, otherwise known as the Philippine Nursing Act of 2002 was implemented to define the scope of nursing practice, licensing requirements, and standards of nursing care. It also defines the powers and duties of the Board of Nursing (BON). Which of the following statements is NOT part of the BON's power and duty?

Co
b. gaug e 23 c. gaug e 24 d. gaug e 14

slow the infusion and damage blood cells (although a smaller gauge needle may be necessary for small children or clients with small, fragile veins) A larger-bore needle allows infusion of the blood elements without clogging the line or the IV access site. A 22-gauge or 24-gauge is too small to infuse blood (Kozier, Saunders NCLEX).

Choose one answer. - close any school of nursing that is not meeting the expected standards of quality nursing education. The BON has the duty to ensure quality nursing education by those seeking permission to open nursing courses to ensure that standards of nursing education are properly complied and maintained at all times. The authority to open and close colleges of nursing and/or nursing programs shall be the duty of the Commission on Higher Education upon the written recommendation of the Board. The other choices are correct (Ultimate Learning Guide).

a. Close any school of nursing that is not meeting the expected standards of quality nursing education.

b. Conduct the licensure examination for nurses.

c. Revoke certificates of registration for the practice of nursing.

d. Investigate nurse practitioners who were complained for unethical and unprofessional conduct.

Incorrect Marks for this submission: 0/1.

Question 41 Marks: 1 Nurses assume a number of roles when they provide care to clients. You are the nurse of a hypertensive client in the community setting. You assist her in modifying her lifestyle habits including diet, exercise and activity to prevent development of complications of hypertension. By this, you are assuming the role of: Choose one answer. a. Caregiv er

- change agent. The nurse acts as a change agent when he/she assists his/her client/s in making modifications in their behavior. Nurses also often act to make changes in the system, such as b. clinical care, if it is not helping a client return to health. By Change assuming the role of the client advocate, a nurse acts to protect agent the client by representing the client's needs and wishes to other health professionals, assisting clients in exercising their rights and helping them speak up for themselves. The caregiver role, on the other hand, include those activities that assist the client physically and psychologically while preserving the client's dignity. Lastly, being a counselor is helping a client recognize and cope with stressful psychological or social problems, to develop improved interpersonal relationships, and to promote personal growth (Kozier). c. Counsel or

d. Client advocat

Incorrect

Marks for this submission: 0/1. Question 42 Marks: 1

The purpose of the research design is to maximize control over factors that can interfere in the validity of the findings. A team of nurseresearchers wants to study the factors contributing to the onset of the irritable bowel syndrome (IBS). They compared samples of women with and without IBS in terms of their history of sexual and physical abuse. The study found out that abusive experiences were more prevalent among women with IBS. Which type of research design was most likely used in the given research study? Choose one answer. a. Prospecti ve design

In
b. Crossover design c. Ex post facto

Incorrect

Marks for this submission: 0/1. Question 43 Marks: 1

Accurate diagnoses of client problems are necessary so that appropriate interventions to address the problems are initiated. A client who will be undergoing mastectomy the following day expresses sadness about losing her breast. Based on this information, the nurse would identify that the client is at risk for the nursing diagnosis of: Choose one answer. a. Anticipato ry grieving

In
- retrospective design. Studies with a retrospective design are d. ones in which a phenomenon existing in the present is linked to Retrospec phenomena that occurred in the past, before the study was tive initiated. The researcher is interested in a present outcome and design attempts to determine antecedent factors that caused it. A prospective design, on the other hand, starts with a presumed cause and then goes forward in time to the presumed effect. An ex post facto design means that the study has been conducted after variations in the independent variable has occurred. A crossover design is used when it involves exposure of the same subjects more than one experimental treatment (Polit & Beck). - anticipatory grieving. Anticipatory grieving is the state in which an individual experiences reactions in response to an expected significant loss. Disturbed body image is the confusion in the mental picture of one's self and is often characterized by negative responses such as shame, embarrassment, guilt, or revulsion. Fear is usually characterized by feelings of dread, fright, apprehension, or alarm. Ineffective coping is usually characterized by verbalization of inability to cope or ask for help, inappropriate use of defense mechanisms, or inability to meet role expectations (Kozier). b. Ineffective coping

Incorrect

Marks for this submission: 0/1. Question 44 Marks: 1

Many factors can facilitate or hinder learning by a client. The nurse should be aware of these factors, particularly when available teaching time is limited. A patient underwent surgery for creation of a colostomy. Three (3) days post-op, he stated, "Nurse, will I be able to clean my colostomy bag by myself? I can do it now. I do not experience any more pain as I have experienced in the past 2 days" The patient demonstrates: Choose one answer. a. Active involveme nt

In
c. Fear d. Disturbed body image

Incorrect

Marks for this submission: 0/1. Question 45 Marks: 1

Republic Act No. 9173, otherwise known as the Philippine Nursing Act

In
b. Motivation to learn c. Expression of relevance - readiness to learn. Readiness to learn is the demonstration of behaviors or cues that reflect the learner's motivation to learn at d. a specific time. Motivation to learn is the desire to learn. Readiness Motivation is generally greatest when a person recognizes a to learn need and believes the need will be met through learning and the need must be experienced by the client. Active involvement lets the learning be more meaningful because it promotes critical thinking and enables learners to problem-solve more effectively. Relevance is another factor wherein the knowledge or skill to be learned must be personally relevant to the learner. They learn more easily if they can connect new knowledge to that which they already know or have experienced (Kozier).

of 2002, also defines the requirement for inactive nurses when they want to return to practice. Nurse Lyn has not practiced the profession for five (5) consecutive years and she wants to practice again. What will be required of her? Choose one answer. a. She will have to take a special type of oral and practical examination to practice again.

b. She is required to undergo one (1) month of didactic training and three (3) months of practicum.

- she is required to undergo one (1) month of didactic training and three (3) months of practicum. This is a retraining program that is a strategy toward quality assurance and protection of the welfare of clients. The competencies of an RN who has been away from the various practice fields, particularly the clinical area, may be outdated with the advancing technology and progressive scientific developments. The need for an RN to keep pace with these rapid changes has been identified, and a training program is perceived as the best strategy in preparing a nurse to reassume active professional roles (Baldago).

c. She is required to undergo three (3) months of didactic training and one (1) month of practicum.

d. She is required to undergo two (2) months of didactic training and two (2) months of practicum.

Incorrect

Marks for this submission: 0/1. Question 46 Marks: 1

Bathing removes accumulated oil, perspiration, dead skin cells, and some bacteria. In addition to cleaning the skin, bathing also stimulates circulation and produces a sense of well-being. Nurse Lara is about to perform a bed bath on Mrs. A, who is on strict bed rest. In order to promote circulation, the nurse bathes the client extremities by using: Choose one answer. a. Firm circular strokes from proximal to distal areas

b. Short, patting strokes from distal to proximal areas

c. Long, firm - long, firm strokes from distal to proximal areas. Long, strokes from distal firm strokes in the direction of venous flow when to proximal areas bathing the extremities promote circulation by increasing venous blood return. Circular strokes are used on the face. Short, patting strokes and long, firm strokes from proximal to distal areas are not as comfortable for the client and they do not promote

venous return. (Kozier, Saunders NCLEX)

d. Long, firm strokes from proximal to distal areas

Incorrect Marks for this submission: 0/1. Question 47 Marks: 1 To ensure that nursing care is provided to patients, the work must be organized. A care delivery model organizes the work of caring for patients. The nurse manager and staff of a 25-bed medical unit have decided to change the client care delivery model from primary nursing to team nursing. The nurse manager states which of the following statements for the administration that is most appropriate to support this change? Choose one answer. a. The staffing goal is to have four groups, with a total of 3 to 4 RNs, 2 to 3 nursing assistants and 2 to 3 utility workers per shift. - the staffing goal is to have four groups, with a total of 3 to 4 RNs, 2 to 3 nursing assistants and 2 to 3 utility workers per shift. This is indicative of team nursing wherein the nurse manager assigns staffs to teams that then are responsible for a group of patients. Each team is led by an RN and is commonly made up of NAs and UW's and the team leader supervises and coordinates all the care provided by those in his/her team. A primary nurse is responsible for developing with a patient

a plan of care that is followed by other nurses in the unit. Functional nursing divides the nursing work into functional units that are then assigned to one of the team members. Each care provider has specific duties or tasks they are responsible for. In total patient care, the nurse is responsible for the total care for her patient assignment for the shift she is working (Kelly-Heidenthal). b. "One RN will serve as the admission nurse, one will be the medication nurse, and the charge or head nurse will make the decisions."

c. "An RN is responsible for the total care of her assigned patients for the shift."

d. The registered nurses on the unit will be assigned 24 hours of responsibility for client care planning.

Incorrect Marks for this submission: 0/1. Question 48

Marks: 1 Nursing practice is governed by many legal concepts and it is important for nurses to know the basics of legal concepts, because nurses are accountable for their professional judgments and actions. A nurse is conducting health teaching to a parent group. When teaching about child abuse, the nurse tells the parents that the best legal definition of assault is: Choose one answer. a. Threats to do bodily harm to the person by another - threats to do bodily harm to the person by person another person. Assault is a threat or an attempt to do violence to another. Battery is being defined as the application of force to another person without lawful justification. (Mosby NCLEX). b. A legal wrong committed by one person against the property of another

c. A legal wrong committed against the public and punishable by law through the state and courts

d. The application of force to another person without lawful justification

Incorrect Marks for this submission: 0/1. Question 49 Marks: 1 Magnetic Resonance Imaging (MRI) is a noninvasive diagnostic scanning technique in which the client is placed in a magnetic field. Which of the following statements made by the client who is to undergo MRI needs further instruction from the nurse? Choose one answer. a. "I will be able to talk with the technician during the procedure."

b. "I will remove all my jewelry before the procedure."

c. "I must lie very still while I go into the machine."

d. "I am afraid because I will be exposed to strong radiation while inside the machine."

- "I am afraid because I will be exposed to strong radiation while inside the machine." MRI does not involve exposure to ionizing radiation. Any metal objects which the patient has on should be removed because these objects may absorb heat during the procedure and may cause injury to

the client. The client must also lie still during the procedure to prevent any false readings. Because the MRI scanner is a narrow tube, a two-way communication system is used to monitor the client's response and to help relieve feelings of claustrophobia (Kozier, Brunner). Incorrect Marks for this submission: 0/1. Question 50 Marks: 1 Because nursing research usually focuses on humans, a major nursing responsibility is to be aware of and to advocate on behalf of client's rights. Nurse Gerard is conducting a research regarding the home conditions of families in a slum area and how these affect their health beliefs. However, when these subjects refuse to participate in the study, he maligns and threatens them. What human right as a research subject is he violating? Choose one answer. a. Right to protection from discomfort or harm

b. Right to fair treatment

- right to fair treatment. For many years, research was conducted on categories of people who were thought to be especially suitable as research subjects, such as people living in poverty, charity patients, prisoners, dying persons, and others who were considered undesirable. Researchers often treated these subjects carelessly and had little regard for the harm and discomfort they experienced. The right to fair treatment is based on the ethical principle of justice that people must be treated fairly and receive what they are due or owed (Burns & Grove).

c. Right to

anonymity and confidentiality

Incorrect

Marks for this submission: 0/1.

Marks: --/1

Furosemide blocks absorption of sodium and water in proximal renal tubule and interferes with reabsorption of sodium in loop of Henle and in most proximal portion of the distal tubule. An important nursing care during the administration of this drug in a pediatric patient with severe congestive heart failure is: Choose one answer.

a. Monitor for signs of water intoxication.

b. Encourage consumption of foods high in calcium.

c. Monitor for side effects such as hyperchloremia and hyperkalemia. d. Encourage consumption of foods high in potassium. Question 2 Marks: --/1

In
d. Right to privacy

Therapeutic positioning is used to reduce the potential for acquired positional deformities that can affect motor development, play skills, attractiveness, and social attachment. Further teaching is required of a student nurse who places an infant in which position? Choose one answer. a. supine position b. side-lying position c. slightly flexed neck d. hyper abduction and flexion of the arms Question 3 Marks: --/1 Most pregnant woman notice the sensation of nausea as early as the first missed menstrual period and experience it through the first three months of pregnancy. You are evaluating your patient after a health teaching regarding nausea. Which of the following statements when made by the pregnant patient would alert you that she still needs further teaching about the prevention and management of nausea? Choose one answer. a. "During an attack, I should eat highly seasoned food so that it will abruptly stop." b. "I must delay breakfast past the time when my nausea seems to persist." c. "I should avoid sudden movements and fatigue." d. "I should eat a few dry crackers before I get out of bed in the morning to prevent nausea." Question 4 Marks: --/1 As early as the 3rd week of intrauterine life, fetal blood begins to exchange nutrients with the maternal circulation across the chorionic villi. Fetal circulation differs from extrauterine circulation in all but one of the following aspects: Choose one answer. a. fetal oxygen in the blood is derived from the placenta while in an adult the oxygen is from the lungs

b. In fetal circulation, shunting of blood is present while in an adult circulation, their is normally no shunting of blood. c. The blood that enters the lungs are oxygenated in the fetal circulation while it is unoxygenated in an adult circulation. d. The vein (carrying oxygenated blood) in the umbilicus of the fetus carries blood away from the heart and the artery (carrying unoxygenated blood) carries blood toward the fetus. In an adult, it is the vein which carries unoxygenated blood toward the heart and the arteries that carry oxygenated blood away from the heart. Question 5 Marks: --/1 Attachment is the special bonding between an infant and usually the mother that develops in the fist 6 months of life. It is generally accepted as the foundation of social development. According to Bowlby and his colleagues, children in the toddler years (15-30 months) who not only were separated from their mothers, but also for weeks or months were cared for in hospitals or nurseries where there had no mother substitute experience a predictable sequence of behavior. In which stage in this sequence does the child experience extreme restlessness and loud crying? Choose one answer. a. detachment b. despair c. protest d. disorganization Question 6 Marks: --/1 Physiologic changes that occur during pregnancy can be categorized as local (confined to the reproductive organs) or systemic (affecting the entire body). Which of the following signs/symptoms when manifested by a healthy pregnant woman mostly needs further evaluation? Choose one answer. a. burning sensation in the vagina

b. amenorrhea c. white vaginal discharge d. nasal stuffiness Question 7 Marks: --/1 It is universally agreed that breast milk is the preferred method of feeding in newborn, because it provides numerous nutritional health benefits to both mother and infant. It remains the ideal nutritional source for infants through the first year of life. Painful nipples may be a complication of improper breastfeeding. Which of the following is advisable for a mother who has problems with sore nipples? Choose one answer. a. application of vitamin D to the nipples b. expose nipples to air c. place the baby in a single position for each feeding d. use bras with plastic liners Question 8 Marks: --/1 When dealing with a child, the timing and extent of explaining the procedure depends largely on the patient's age and degree of comprehension. Whenever a procedure is to be explained to a toddler and preschooler, it is important to know that although the approaches are almost similar, some differences are also taken into consideration, one of which is: Choose one answer. a. In a toddler patient, the procedure must first be explained to the parents before explaining it to a child, in a preschooler patient, it is better to explain the procedure to the child first before giving details to the parents b. To decrease their anxiety, the reason why the procedure is being done should be explained thoroughly to a preschooler. In a toddler, it is enough to mention the benefits to be gained in the procedure. c. A toddler, younger than the preschooler should be allowed to cry. The latter is expected not to yell or cry since he/she is older and can

understand better. d. A preschooler has longer attention span than a toddler so more comprehensive discussion of the procedure must be done. Question 9 Marks: --/1 Adolescence can be thought of as involving three distinct sub-phases: early adolescence (ages 11 to 14), middle adolescence (ages 15 to 17), and late adolescence (ages 18 to 20). In planning care for a 16year-old patient, the nurse takes into consideration which growth and development changes that occur in a middle adolescence? Choose one answer. a. preoccupation with rapid body changes, usually involved in group rather than intimate dating, growth is rapidly accelerating b. comfortable with physical growth, the structure and reproductive growth is almost complete, intimacy involves commitment rather than exploration and romanticism c. All of the these d. physical growth is decelerating in females, with feelings of being in love, has strong need for identity to affirm self-image Question 10 Marks: --/1 A comprehensive system for classifying pregnancy status is using GTPALM which provides greater detail on a woman 's pregnancy history. By this system, gravida classification remains the same but para is broken down into T P A L M. You are assessing the obstetric history of a woman who is 2 weeks pregnant during her prenatal visit at the health center. She told you that she has 3 previous pregnancies but had miscarriage during her 2nd pregnancy. Her first child was only 28 weeks when he died immediately after caesarian birth. Her third pregnancy resulted in fraternal twins, one of which has congenital heart defect, born at 37th week of pregnancy. She has now learned her lesson that prenatal checkup is very important for herself and her children's well-being. You will write in her chart which of the following information based on this system? Choose one answer.

a. G4 T2 P1 A1 L2 M1 b. G3 T2 P1 A1 L2 M2 c. G4 T1 P1 A1 L1 M2 d. G4 T1 P2 A1 L2 M1 Question 11 Marks: --/1 Unless a woman's job involves exposure to toxic substances, lifting heavy objects, other kind of excessive physical strain, long periods of standing or having to maintain body balance, there are few reasons a woman cannot continue to work throughout pregnancy. Which type of nurse should be discouraged from working as soon as she finds out she is pregnant? Choose one answer. a. nurse in the health center (community) b. ward nurse c. nurse-anesthesiologist d. ICU nurse Question 12 Marks: --/1 A woman with any degree of bleeding needs to be evaluated for the possibility that she is experiencing a significant blood loss and for hypovolemic shock. As a nurse in the ER you would anticipate that the least appropriate emergency intervention for bleeding during pregnancy is: Choose one answer. a. Assisting in vaginal examination. b. Withholding oral fluids c. Administration of oxygen as necessary at 6-10 L/minute. d. Setting aside 5ml of blood drawn intravenously in a clean test tube and observing it for any clot formation for 5 minutes. Question 13

Marks: --/1 Pediatric criteria have been developed for the Glasgow coma scale that consider age and developmental level in assessing the child's ability to open his/her eyes (E), provide verbal response (V) and provide motor response (M). You are caring for 1 year old child with meningitis. You applied the Glasgow coma scale to test for his neurologic status. You observed that she withdraws and opens her eyes after you apply some pressure on her right thumb. She also moans after a while. You have an idea that based from the Glasgow coma scale scoring that this child is Choose one answer. a. conscious b. in vegetative state c. in coma d. in deep coma Question 14 Marks: --/1 Estrogens are hormones that are associated with characteristics contributing to "femaleness". The following are effects of estrogen except for one: Choose one answer. a. maturation of ovarian follicles b. increased uterine size c. widening of hips d. None of the above Question 15 Marks: --/1 Respiratory failure is defined as the inability of the respiratory apparatus to maintain adequate oxygenation of the blood, with or without carbon dioxide retention. This process involves pulmonary dysfunction that generally results in impaired alveolar gas exchange, which can lead to hypoxemia or hypercapnea. One important aspect of care in a pediatric patient having respiratory distress is the assessment of fever and pain. This is because one vital goal of

therapy of this disorder is: Choose one answer. a. correct hypercapnea b. control oxygen demands c. treat the underlying cause d. correct hypoxemia Question 16 Marks: --/1 A teratogen is any factor, chemical or physical, that adversely affects the fertilized ovum, embryo, or fetus. To reach maturity in optimal health, a fetus needs sound genes and a healthy intrauterine environment that protects it from the influence of teratogens. Evidence over the years has shown that when women consume a large quantity of alcohol during pregnancy, their babies show a high incidence of congenital deformities and cognitive impairment. Which of the following best explain the effect of alcohol to the fetus? Choose one answer. a. alcohol causes vasoconstriction of the uterine vessels that limits blood supply to the fetus b. alcohol can interfere with the cell metabolism that can lead to cell mutation c. fetuses cannot remove the breakdown of products of alcohol from their body d. alcohol increases the risk for transmission of infection from the mother to the fetus Question 17 Marks: --/1 Placenta previa is low implantation of the placenta instead of in the upper portion of the uterus. A patient who has placenta previa and in her 32nd week of pregnancy shows signs of labor and was given Betamethasone (Celestone) 12 mg IM. The patient's husband asked you what is the purpose of this drug. The most appropriate response would be: Choose one answer.

a. "It decreases the occurence of painful contractions" b. "It protects your wife and your baby from infection." c. "It hastens the maturity of your baby's lungs" d. "It decreases likelihood of occurence of bleeding since your wife is prone to it. Question 18 Marks: --/1 The immaturity not only places infants at risk for neonatal complications but also may predispose the infant to problems that persist into adulthood. Which of the following factors makes a premature infant more prone to experiencing aspiration than a term infant? Choose one answer. a. muscle tone in the cardiac sphincter is poorer b. more decreased capacity to digest and absorb protein c. the stomach has more limited capacity d. the initial sucking is accompanied by ineffective swallowing Question 19 Marks: --/1 Burn patients experience a hypometabolic phase immediately after injury lasting 2 or 3 days when their metabolic rate and cardiac output decrease. Following this phase, patients experience a hypermetabolic phase. The most appropriate nursing diagnosis during this phase when the patient experiences muscle weakness and poor wound healing is: Choose one answer. a. Acute Pain b. Imbalanced Nutrition: Less than Body Requirements c. Risk for Injury d. Decreased Cardiac output Question 20

Marks: --/1 Although caloric needs are diminished in relation to body size during the school-age period, resources are being laid down for the increased growth needs of the adolescent period. It is important to impress on children and their parents the value of a balanced diet to promote growth. A Grade 4 teacher asks you on how she can promote nutrition education to her students. You know that she should: Choose one answer. a. Ask children to select foods from a fast-food restaurant menu and identify those which are nutritious. b. Encourage parents that they should be the ones to teach their parents regarding nutrition since the subjects in the schools should be spent in more important lessons. c. Avoid the use of food as rewards for behavior. d. Refrain from allowing the child to prepare foods since she will choose delicious, non-nutritious ones according to her preference. Question 21 Marks: --/1 Ultrasonagraphy, a much-used tool in modern-obstetrics, measures the response of sound waves against solid objects. Your patient, in the second month of pregnancy is scheduled for an ultrasound exam for the first time. Your nursing interventions before the procedure include: Choose one answer. a. Ask the patient to void immediately before the procedure since the a full bladder affects the quality of the result. b. Encourage the father to come if the couple wants to since the procedure is safe even for him. c. Explain what will happen during the procedure and advise her that since x-rays are being emitted by the machine, she should not first interact with others during the first three hours after the procedure. d. It is also imperative for the patient to be placed on NPO 6 hours before the procedure. Question 22 Marks: --/1

Juvenile rheumatoid arthritis is a disease of connective tissue with joint inflammation and contracture and is most likely the result of an autoimmune response. Women with this condition frequently take corticosteroids and non-steroidal anti-inflammatory drugs (NSAIDs) to prevent joint pain and loss of mobility. Pregnant patients with this condition should be advised to limit or discontinue taking NSAIDs, particularly Aspirin because of which effects: Choose one answer. a. prolonged pregnancy b. late closure of the ductus arteriosus in the child c. potent carcinogen to the mother d. increased incidence of formation of blood clot in both mother and child Question 23 Marks: --/1 The treatment of Acute Renal Failure is directed toward the treatment of the underlying cause, management of the complications of renal failure and provision of supportive therapy within the constraints imposed by the renal failure. You are caring for a child with Acute Renal Failure (ARF). You know that the mother takes your health teaching sessions seriously when she gives her child which of the following foods? Choose one answer. a. Home-made french fries without salt b. Mango ice cream c. Prune juice d. Buttered popcorn Question 24 Marks: --/1 Restraints must be ordered by a physician, with the type of restraint identified and how often the child can be removed from the restraint noted. Which of the following kind of restraint should be questioned by the nurse if ordered by the doctor for a pediatric patient? Choose one answer.

a. Mummy restraint b. Elbow restraint c. Belt restraint d. Human restraint Question 25 Marks: --/1 After the initial period of shock and restoration of fluid balance in a burn patient, the primary concern is the burn wound. Which intervention is the most vital in the management of a major burn wound in a pediatric patient? Choose one answer. a. debridement b. fluid replacement c. excision d. temporary skin substitute Question 26 Marks: --/1 Because of the complexity of many defects and the variability of their clinical manifestations, the cyanotic-acyanotic classification system of the congenital heart defects has proven to be inadequate and misleading. A more useful classification is based on hemodynamic characteristics, or movements involved in the circulation of blood: increased pulmonary blood flow, decreased pulmonary blood flow, obstruction to blood flow out of the heart, and mixed blood flow. Which of the following congenital heart defect is classified as having decreased pulmonary blood flow? Choose one answer. a. transposition of great arteries b. coarctation of the aorta c. Tetralogy of fallot d. patent ductus arteriosus

Question 27 Marks: --/1 The Lamaze method of prepared childbirth is based on the gate control theory of pain relief. It was originally termed the psychoprophylactic method, as it focuses on preventing pain in labor by use of the mind. Which of the following techniques is not used in the Lamaze method to assist in gating mechanisms thus decreasing the perception of pain? Choose one answer. a. effleurage b. health teaching c. use of pain medications d. vocalization Question 28 Marks: --/1 Medications are an important adjunct to resuscitation, especially in cardiac arrest, and are used during and after resuscitation in children. As a head nurse in the pediatric ward, it is your duty to orient student nurses assigned to your area. During orientation, one of the patients was being resuscitated. You allowed the students to observe for a while. After the patient was successfully revived, one of your students asked you the why did the doctor ordered Atropine sulfate. You know that the primary action of this drug is: Choose one answer. a. It is an antidysrhythmic drug that allows the heart to maintain the regular beat of the heart b. It reverses respiratory arrest brought about by the cessation of the function of the heart c. It increases cardiac output and heart rate which is important since heart ceases to function during an arrest d. It decreases heart rate which abnormally increases during a cardiac arrest Question 29 Marks: --/1

Many factors enter the categorization of high risk pregnancy. Most health care agencies use some tool to determine high risk, but no tool is perfect because the concept of high risk is very individualized. In which of the following situations show the psychosocial factor that is least suggestive of high risk pregnancy? Choose one answer. a. Via, 4 weeks pregnant because her ex-boyfriend raped her b. Gia, 4 weeks pregnant and has hyperthyroidism c. Mia, 4 weeks pregnant, living in a shanty under the bridge d. Ria, 17 years old and is 4 weeks pregnant Question 30 Marks: --/1 One of the most distressing side effects of several drugs is gastrointestinal mucosal cell damage, which results in ulcers anywhere along the alimentary tract. Which of the following interventions for oral ulcers in a toddler undergoing chemotherapy is the most acceptable? Choose one answer. a. To reduce oral pain, massage the back of both hands with an ice cube b. To clean the gums, use lemon glycerin to swab on the gums, palate and inner cheek surfaces c. Using a straw when drinking is usually not advisable because the straw can increase likelihood of damage to the area d. Mouth care should be done two to thrice a day to reduce further injury from the activity Question 31 Marks: --/1 According to Erik Erikson's psychosocial stage of development, a sense of industry, or a stage of accomplishment is achieved somewhere between age 6 years and adolescence. This stage is described by the following except: Choose one answer. a. This stage is the time when children learn the value of doing things with

others and the benefits derived from division of labor in the accomplishment of goals. b. The danger in this period is the occurence of situations that might result in a sense of role confusion. c. In this stage, interests expand and with a growing sense of independence, children want to engage in tasks that can be carried through completion. d. Peer approval is a strong motivating power. Question 32 Marks: --/1 Cancers in children are often difficult to recognize. Therefore, being alert to the persistence of unusual symptoms is essential. All but one of the following is a cardinal symptom of cancer in children: Choose one answer. a. excessive weight gain b. unexplained pallor c. prolonged fever d. sudden eye changes Question 33 Marks: --/1 A post term infant is one born after the 42nd week of pregnancy. Which of the following assessment findings is least likely seen in a post term infant? Choose one answer. a. hypoglycemia b. polycythemia c. meconium aspiration d. decreased hematocrit Question 34 Marks: --/1

The umbilical cord is formed from the fetal membranes and provides a circulatory pathway that connects the embryo to the chorionic villi of the placenta. Its function is to transport oxygen and nutrients to the fetus from the placenta. In the initial assessment of a newborn female child, you found that the umbilical cord has one vein and one artery. You will suspect that this child has a high risk of developing which condition? Choose one answer. a. undescended testes b. congenital heart anomaly c. none of these since this finding is normal d. congenital hearing and visual loss Question 35 Marks: --/1 Children as young as the early teens need guidelines for safer sex because of increasing trends of adolescents who are already sexually active. You are evaluating a group of 15 year old adolescents in a school where you are invited to teach about safe sex practice. You know that further teaching is needed of the student who will verbalize: Choose one answer. a. "Voiding immediately after sex is helpful in washing away contaminants." b. "Anal intercourse carries increased risk for developing AIDS than vaginal sex." c. "Condoms should be inflated before use to test if they are intact and is safe to use." d. "The use of condom is the best protection against infection." Question 36 Marks: --/1 Often a nurse is the one who first identifies a need for counseling and referral by identifying the presence of an inherited disorder in a family history or by noting physical, mental, or behavioral abnormalities when performing a nursing assessment. All of the following findings in the family history of your pediatric patient is considered to be an assessment clue to genetic abnormalities except:

Choose one answer. a. heart palpitation b. attention deficit disorder c. hearing loss d. scoliosis Question 37 Marks: --/1 Menopause is the cessation of menstrual cycles. Perimenopausal is a term used to denote the period wherein menopausal changes are occurring. An obese 42-year old mother, asks you why she experienced menopause at this early age when her friend who is now 50 years old, nullipara, still has regular menstruation. You will ask her which of the following to confirm if the age of her menopause is normal: Choose one answer. a. age of menarche b. the number of children c. age of first sexual activity d. age of thelarche Question 38 Marks: --/1 Because genetic disorders often occur in varying degrees of expression, a careful physical assessment of any family member with a disorder, that child's siblings, and the couple seeking counseling is needed. A child with the following characteristics: slanted eyes, with epicanthal folds and large tongue, and is cognitively challenge is most probably manifesting which genetic disorder? Choose one answer. a. turner syndrome b. klinefelter syndrome c. cri-du-chat syndrome

d. Down's syndrome Question 39 Marks: --/1 Assessing for the reflexes is an important element in the Pediatric Physical Assessment, especially that of an infant. It is a measure of an intact neurologic system. A student nurse caring for a sleeping 6month old patient needs further reading when she noted in the chart that this reflex is positive: Choose one answer. a. Babinski b. Rooting c. Sucking d. Palmar grasp Question 40 Marks: --/1 Radiography is used frequently in diagnostic evaluation in children. Although no definitive information exists on the effects of low-dose radiation, measures are carried out to protect vulnerable areas from possible damage. In caring for a child that is suspected of having brain tumor, the nurse would anticipate which radiologic examination to be done to the patient? Choose one answer. a. brush biopsy b. bronchography c. MRI d. capnography Question 41 Marks: --/1 Although the parent and child are separate and distinct individuals, the nurse's relationship with the child is frequently mediated via the parent, particularly in the case of younger children. Which of the following interventions facilitates effective communication with

parents of pediatric patients the least? Choose one answer. a. active listening b. using silence c. directing the focus of the conversation d. being sympathetic Question 42 Marks: --/1 Menses is actually the end of an arbitrarily defined menstrual cycle. Which of the following statements best describe menses? Choose one answer. a. Both B and C b. The last day of the menstrual flow marks the beginning of a new menstrual cycle. c. The iron loss is so small that the body is still able to adequately replace the iron loss. d. It contains only little amount of blood flow, approximately 30 to 80 ml of blood. Question 43 Marks: --/1 Amniocentesis is a procedure done in a physician's office or in ambulatory clinic and is typically scheduled between the 14th and 16th weeks of pregnancy to allow for a generous amount of amniotic fluid to be present and also at near term. Which of the following information is provided by this test? Choose one answer. a. grading of the placenta b. volume of the amniotic fluid c. biparietal diameter of the fetal head d. fetal lung maturity

Question 44 Marks: --/1 The overall management of infertility involves treating the underlying causes such as chronic disease, inadequate hormone production, endometriosis, or infection. You would advise a patient undergoing hormone therapy with Clomiphene citrate of the increased possibility of occurence of which condition: Choose one answer. a. early menopause b. myoma c. ovarian atrophy d. multiple pregnancy Question 45 Marks: --/1 Cardiac arrest in a pregnant woman is most often related to events at the time of birth such as amniotic fluid embolism, eclampsia, stroke, hemorrhage, and acute respiratory failure. In which aspect is the cardiopulmonary resuscitation different between a pregnant patient and an adult non-pregnant one? Choose one answer. a. the site where the pulse is palpated to establish circulation b. the maneuver used to open the airway c. the position of the patient d. the ratio of compression to breathing to be given by one rescuer. Question 46 Marks: --/1 Simply stated, acidosis (acidemia) results from either accumulation of acid or loss of base, and alkalosis (alkalemia) results from either accumulation of base or loss of acid. A newborn patient having the following ABG results: pH=7.49, pCO2= 46 mmHg and HCO3= 21mEq/ml, would alert you because this is a manifestation of: Choose one answer.

a. metabolic acidosis b. respiratory acidosis c. respiratory alkalosis d. metabolic alkalosis Question 47 Marks: --/1 Many women enter pregnancy with a deficiency of iron stores resulting from a diet low in iron, heavy menstrual periods, or unwise weight-reducing programs. A patient who is in 24th week of pregnancy presents with the following signs and symptoms: generalized pallor, pale conjunctivae and nail beds, with cool clammy skin. Her complete blood count shows decreased hematocrit and hemoglobin levels. She was later diagnosed with iron-deficiency anemia. The most appropriate nursing diagnosis for this patient is: Choose one answer. a. Ineffective tissue perfusion related to decreased circulating hemoglobin b. Activity intolerance related to maternal anemia during pregnancy c. Deficient fluid volume related to decreased hematocrit concentration d. Impaired cellular circulation related to maternal anemia during pregnancy Question 48 Marks: --/1 Although most women are pleased to be pregnant, the symptoms of early pregnancy tend to cause discomfort to a woman rather than provide evidence that she is carrying a child. In a woman with nursing diagnosis of constipation related to reduced peristalsis in pregnancy, which of the following interventions is the most appropriate? Choose one answer. a. Promote use of mineral oil in relieving constipation so that vitamins will not be excreted. b. Inform her that over-the-counter laxatives are contraindicated. c. Encourage her to evacuate her bowels every other day.

d. Advise her to take BRAT (banana, rice, apple and tea) diet to increase roughage in her diet. Question 49 Marks: --/1 When palpating for the testes during a pediatric physical examination, avoid stimulating the cremasteric reflex which pulls the testes higher into the pelvic cavity. A student nurse is having difficulty in her assessment asks you of an effective way of how to avoid eliciting this reflex. You teach her that this is best done by: Choose one answer. a. positioning the patient in dorsal recumbent b. placing the thumb and index finger over the lower part of the scrotal sac c. positioning the patient in lithotomy d. warming her hands Question 50 Marks: --/1 Kidney transplantation is the preferred means of renal replacement therapy in the pediatric age-group. Which of the following manifestations when exhibited by a patient who underwent kidney transplant 5 months ago is most suggestive of transplant rejection? Choose one answer. a. hypotension b. hypertension c. decreased BUN levels d. vomiting

1 Marks: 0/1 Furosemide blocks absorption of sodium and water in proximal renal tubule and interferes with reabsorption of sodium in loop of Henle and in most proximal portion of the distal tubule. An important nursing

care during the administration of this drug in a pediatric patient with severe congestive heart failure is: Choose one answer. a. Monitor for signs of water intoxication.

b. Encourage consumption of foods high in calcium.

c. Monitor for side effects such as hyperchloremia and hyperkalemia.

- Encourage consumption of foods high in potassium. Furosemide causes excretion of chloride and d. Encourage potassium together with sodium, therefore consumption of foods hypochloremia and hypokalemia may result. It is high in potassium. important for the nurse to encourage the child to consume foods rich in potassium such as avocado, banana, and tomatoes. The child cannot be at risk for water intoxication because furosemide is a diuretic that causes excretion of water. It also does not cause changes in calcium levels. (Wong) Incorrect

Marks for this submission: 0/1. Question 2 Marks: 0/1 Therapeutic positioning is used to reduce the potential for acquired positional deformities that can affect motor development, play skills, attractiveness, and social attachment. Further teaching is required of a student nurse who places an infant in which position? Choose one answer. a. supine position

b. side-lying position

c. slightly flexed neck

d. hyper abduction and flexion of the arms

- hyper abduction and flexion of the arms. This position causes upper extremity external rotation, resulting in persistent W positioning of the arms that can interfere with later midline skills that form the foundation for feeding, crawling, reaching. and midline play with objects. Side lying position is an alternative position. It must be well-supported with a swaddling or a heavy blanket roll surrounding the infant's flexed back to promote midline hands-together or hands-to-face movements. Supine sleeping position is recommended for healthy infants in the first year of life as a preventive measure for Sudden Infant Death Syndrome

(SIDS). The neutral or slightly flexed neck is also recommended to open the airway and promote brain circulation. (Wong) Incorrect Marks for this submission: 0/1. Question 3 Marks: 0/1 Most pregnant woman notice the sensation of nausea as early as the first missed menstrual period and experience it through the first three months of pregnancy. You are evaluating your patient after a health teaching regarding nausea. Which of the following statements when made by the pregnant patient would alert you that she still needs further teaching about the prevention and management of nausea? Choose one answer. - "During an attack, I should eat highly seasoned food so that it will abruptly stop." It is a. "During an attack, I recommended to avoid highly seasoned or should eat highly seasoned greasy foods because they increase the food so that it will abruptly likelihood of the occurence of nausea. stop." Increasing carbohydrate intake such as eating dry crackers or toast, seems to relieve nausea better than any other nutrition remedy. Sudden movements and fatigue aggravate its occurence. It is also advisable that the patient eat light breakfast or delay breakfast until 10 or 11 AM, past the time her nausea seems to persist. (Pilliterri) b. "I must delay breakfast past the time when my nausea seems to persist."

c. "I should avoid sudden movements and fatigue."

Incorrect

Marks for this submission: 0/1. Question 4 Marks: 0/1

As early as the 3rd week of intrauterine life, fetal blood begins to exchange nutrients with the maternal circulation across the chorionic villi. Fetal circulation differs from extrauterine circulation in all but one of the following aspects: Choose one answer.

I
d. "I should eat a few dry crackers before I get out of bed in the morning to prevent nausea." a. fetal oxygen in the blood is derived from the placenta while in an adult the oxygen is from the lungs b. In fetal circulation, shunting of blood is present while in an adult circulation, their is normally no shunting of blood.

I
c. The blood that enters the lungs are oxygenated in the fetal circulation while it is unoxygenated in an adult circulation. d. The vein (carrying oxygenated blood) in the umbilicus of the fetus carries blood away from the heart and the artery (carrying unoxygenated blood) carries blood toward the fetus. In an adult, it is the vein which carries unoxygenated blood toward the heart and the arteries that carry oxygenated blood away from the heart. - The vein (carrying oxygenated blood) in the umbilicus of the fetus carries blood away from the heart and the artery (carrying unoxygenated blood) carries blood toward the fetus. In an adult, it is the vein which carries unoxygenated blood toward the heart and the arteries that carry oxygenated blood away from the heart. In a fetal circulation, the umbilical vein (carrying oxygenated blood) carries blood toward the heart of the fetus and the artery (carrying unoxygenated blood) carries blood away from the fetus. In an adult, it is the vein which carries unoxygenated blood toward the heart and the arteries that carry oxygenated blood away from the heart. The other choices are true of fetal circulation. The blood entering the blood vessels of the lungs is not for oxygen exchange but to supply the cells of the lungs themselves. During the fetal life, the lungs are not yet mature enough to function as that during birth. Therefore, specialized structures present in the fetus shunt blood flow to supply the most important organs of the body: brain,

liver, heart and kidneys. (Pilliterri) Incorrect Marks for this submission: 0/1. Question 5 Marks: 0/1 Attachment is the special bonding between an infant and usually the mother that develops in the fist 6 months of life. It is generally accepted as the foundation of social development. According to Bowlby and his colleagues, children in the toddler years (15-30 months) who not only were separated from their mothers, but also for weeks or months were cared for in hospitals or nurseries where there had no mother substitute experience a predictable sequence of behavior. In which stage in this sequence does the child experience extreme restlessness and loud crying? Choose one answer. a. detach ment

b. despair

c. - protest. The protest is the first stage, appearing immediately and protest persists for about 1 week. Loud crying, extreme restlessness, and rejection of all adult figures mark an infant's distress. Despair is the second stage. The child demonstrates a growing hopelessness: monotonous crying, inactivity and steady withdrawal. Detachment is the final stage, materialized when an infant displays renewed interest in the surroundings, but it was a remote, distant kind of interest. (Thies)

There is no disorganization stage. d. disorga nization

Incorrect Marks for this submission: 0/1. Question 6 Marks: 0/1 Physiologic changes that occur during pregnancy can be categorized as local (confined to the reproductive organs) or systemic (affecting the entire body). Which of the following signs/symptoms when manifested by a healthy pregnant woman mostly needs further evaluation? Choose one answer. - burning sensation in the vagina. Vaginal secretions during pregnancy fall from a pH of greater than 7 to 4 or 5, which a. burning favors the growth of Candida albicans, a yeast-like fungi. A sensation in candidal infection is manifested by an itching, burning the vagina sensation in addition to a cream cheese-like discharge. Amenorrhea (absence of menstruation) occurs because of the suppression of follicle-stimulating hormone (FSH) by rising estrogen levels. In response to this increased estrogen, there is also marked congestion or "stuffiness" of the nasopharynx. White vaginal discharge is also common during pregnancy. (Pilliterri) b. amenorrhea

c. white vaginal discharge

Incorrect

Marks for this submission: 0/1. Question 7 Marks: 0/1

It is universally agreed that breast milk is the preferred method of feeding in newborn, because it provides numerous nutritional health benefits to both mother and infant. It remains the ideal nutritional source for infants through the first year of life. Painful nipples may be a complication of improper breastfeeding. Which of the following is advisable for a mother who has problems with sore nipples? Choose one answer. a. application of vitamin D to the nipples

In
d. nasal stuffiness b. expose nipples to air - expose nipples to air. Exposing nipples to air by leaving the bra unstrapped for 10-15 minutes after feeding will help in preventing soreness. The leaking milk will therefore not soak the nipple. The mother should be encouraged to position her baby slightly

differently for each feeding since this helps prevent the same area of the areola from receiving the majority of pressure. Discourage the use of plastic liners that come with some bras since it is preferable to have air always circulating around the breasts. Applying vitamin E lotion after air exposure may toughen the nipples and prevent further irritation. c. place the baby in a single position for each feeding

d. use bras with plastic liners

Incorrect Marks for this submission: 0/1. Question 8 Marks: 0/1 When dealing with a child, the timing and extent of explaining the procedure depends largely on the patient's age and degree of comprehension. Whenever a procedure is to be explained to a toddler and preschooler, it is important to know that although the approaches are almost similar, some differences are also taken into consideration, one of which is: Choose one answer. a. In a toddler patient, the procedure must first be explained to the parents before explaining it to a child, in a preschooler patient, it is better to explain the procedure to the child first

before giving details to the parents

I
b. To decrease their anxiety, the reason why the procedure is being done should be explained thoroughly to a preschooler. In a toddler, it is enough to mention the benefits to be gained in the procedure. c. A toddler, younger than the preschooler should be allowed to cry. The latter is expected not to yell or cry since he/she is older and can understand better. d. A preschooler has longer attention span than a toddler so more comprehensive discussion of the procedure must be done. - A preschooler has longer attention span than a toddler so more comprehensive discussion of the procedure must be done. Preschoolers have a longer attention span than toddlers, so one can spend 10-15 minutes explaining the procedure. In both patients, toddler and preschooler, it would be more logical to discuss the procedure first with the parents or guardians, away from the patients bed. Then the procedure can be explained to the child directly before the procedure is to be done. Both patients should be allowed to yell, cry or express pain

verbally during the conduct of the procedure. Both patients are only able to understand the benefits to be gained from the procedure, not the reason why the procedure is being done. Therefore, the patients' cooperation may be obtained by telling them that they will feel better or that they will go home, as it deem appropriate, after the procedure. (Caballes) Incorrect Marks for this submission: 0/1. Question 9 Marks: 0/1 Adolescence can be thought of as involving three distinct sub-phases: early adolescence (ages 11 to 14), middle adolescence (ages 15 to 17), and late adolescence (ages 18 to 20). In planning care for a 16year-old patient, the nurse takes into consideration which growth and development changes that occur in a middle adolescence? Choose one answer. a. preoccupation with rapid body changes, usually involved in group rather than intimate dating, growth is rapidly accelerating

b. comfortable with physical growth, the structure and reproductive growth is almost complete, intimacy involves commitment rather than exploration and romanticism

Incorrect

Marks for this submission: 0/1. Question 10 Marks: 0/1

A comprehensive system for classifying pregnancy status is using GTPALM which provides greater detail on a woman 's pregnancy history. By this system, gravida classification remains the same but para is broken down into T P A L M. You are assessing the obstetric history of a woman who is 2 weeks pregnant during her prenatal visit at the health center. She told you that she has 3 previous pregnancies but had miscarriage during her 2nd pregnancy. Her first child was only 28 weeks when he died immediately after caesarian birth. Her third

I
c. All of the these d. physical growth is decelerating in females, with feelings of being in love, has strong need for identity to affirm self-image - physical growth is decelerating in females, with feelings of being in love, has strong need for identity to affirm self-image. Comfortable with physical growth, the structure and reproductive growth is almost complete, intimacy involves commitment rather than exploration and romanticism are experienced during the late adolescence and preoccupation with rapid body changes, usually involved in group rather than intimate dating, growth is rapidly accelerating describe an adolescent in the early phase. (Wong)

pregnancy resulted in fraternal twins, one of which has congenital heart defect, born at 37th week of pregnancy. She has now learned her lesson that prenatal checkup is very important for herself and her children's well-being. You will write in her chart which of the following information based on this system? Choose one answer. - G4 T2 P1 A1 L2 M1. G-gravida is the number of times a woman has become pregnant including the present pregnancy. P-para is the number of pregnancies that reached viability (20 weeks or a. G4 T2 more), regardless of whether the infants were born alive or not. It P1 A1 L2 is composed of T (term)- the number of full-term infants born M1 (infants born at 37 weeks or after), P (preterm)- the number of preterm infants born (infants born before 37 weeks), A (abortion)the number of spontaneous or induced abortions, L (living)- the number of living children, M (multiple)- the number of multiple pregnancies. In the situation, there are 4 pregnancies (3 previous + 1 present), 2 term infants (the twins), 1 preterm infant (1st pregnancy), 1 abortion (2nd pregnancy), 2 living children (twins), 1 multiple pregnancy (twins). (Pilliterri) b. G3 T2 P1 A1 L2 M2

c. G4 T1 P1 A1 L1 M2

d. G4 T1 P2 A1 L2 M1

Incorrect

Marks for this submission: 0/1. Question 11 Marks: 0/1

Unless a woman's job involves exposure to toxic substances, lifting heavy objects, other kind of excessive physical strain, long periods of standing or having to maintain body balance, there are few reasons a woman cannot continue to work throughout pregnancy. Which type of nurse should be discouraged from working as soon as she finds out she is pregnant? Choose one answer. a. nurse in the health center (community)

In
b. ward nurse c. nurse- nurse-anesthesiologist. Some occupations are hazardous anesthesiologis during pregnancy because they bring women into contact t with harmful substances. This include nurses working with anesthetic gases in operating rooms or dental offices. They are reported to have a higher incidence of spontaneous miscarriage and possibly, congenital anomalies in children than nurses working in other locales (ward, ICU, health center), probably due to exposure to nitrous oxide.

(Pilliterri) d. ICU nurse

Incorrect

Marks for this submission: 0/1. Question 12 Marks: 0/1

A woman with any degree of bleeding needs to be evaluated for the possibility that she is experiencing a significant blood loss and for hypovolemic shock. As a nurse in the ER you would anticipate that the least appropriate emergency intervention for bleeding during pregnancy is: Choose one answer.

In
a. Assisting in vaginal examination. - Assisting in vaginal examination. Vaginal examination should not be done during emergency situations to prevent tearing of placenta if placenta previa (low-lying placenta) is the cause of the bleeding. Withholding oral fluids anticipate the need for emergency surgery. Administration of oxygen provides adequate fetal oxygenation despite lowered maternal circulating blood volume. Observation of blood clot tests for possible blood coagulation problems such as disseminated intravascular coagulation, which must be suspected if no clot forms within time limit. (Pilliterri) b. Withholding oral fluids

c. Administration of oxygen as necessary at 6-10 L/minute.

Incorrect

Marks for this submission: 0/1. Question 13 Marks: 0/1

Pediatric criteria have been developed for the Glasgow coma scale that consider age and developmental level in assessing the child's ability to open his/her eyes (E), provide verbal response (V) and provide motor response (M). You are caring for 1 year old child with meningitis. You applied the Glasgow coma scale to test for his neurologic status. You observed that she withdraws and opens her eyes after you apply some pressure on her right thumb. She also moans after a while. You have an idea that based from the Glasgow coma scale scoring that this child is Choose one answer. a. consciou s

I
d. Setting aside 5ml of blood drawn intravenously in a clean test tube and observing it for any clot formation for 5 minutes.

b. in vegetativ e state

Incorrect

Marks for this submission: 0/1. Question 14 Marks: 0/1

Estrogens are hormones that are associated with characteristics contributing to "femaleness". The following are effects of estrogen except for one: Choose one answer. a. maturation of ovarian follicles

In
c. in coma - in coma. The total score of the patient is 8: E2, V2, M4. The Pediatric coma scale for a child less than 2 years old assesses eye opening (spontaneously=4, to speech=3, to pain=2, none-1), response to auditory and/or visual stimulus (smiles, listens, follows=5, cries, consolable=4, agitated, resltess=2, no response=1) and motor response (obeys commands=6, localizes pain=5, flexion withdrawal=4, flexion abnormal=3, extension=2, none=1). A score of 8 or below is generally accepted as a definition of coma. A perfect score of 15: E4, V5, M6 is considered an unaltered level of consciousness. The lowest score, 3, indicates deep coma or death. (Wong) and (Mandleco) d. in deep coma

Incorrect

Marks for this submission: 0/1. Question 15 Marks: 0/1

Respiratory failure is defined as the inability of the respiratory apparatus to maintain adequate oxygenation of the blood, with or without carbon dioxide retention. This process involves pulmonary dysfunction that generally results in impaired alveolar gas exchange, which can lead to hypoxemia or hypercapnea. One important aspect of care in a pediatric patient having respiratory distress is the

In
b. increased uterine size c. widening of hips d. None of the above - None of the above. All are effects of estrogen. It controls the female characteristics such as widening of hips, breast development, and deposits of tissue in the buttocks and mons pubis. It also causes the uterus to increase in size and weight because of the increased glycogen, amino acids, electrolytes and water. Blood supply is expanded as well. Estrogen also assist in the maturation of the of the ovarian follicles and cause the endometrial mucosa to proliferate following menstruation. (London)

assessment of fever and pain. This is because one vital goal of therapy of this disorder is: Choose one answer. a. correct hypercapn ea

b. control oxygen demands

- control oxygen demands. Assessments of pain and fever in a child should be frequent. Both conditions (as well as cold stress) can dramatically increase oxygen requirements, especially in younger children and therefore increase respiratory effort. Pain and fever are not the underlying cause but lung diseases such as restrictive and obstructive ones and conditions brought by primary inefficient gas transfer. Assessing alone also does not treat the disorder nor can it correct hypoxemia and hypercapnea. (Wong)

c. treat the underlying cause

d. correct hypoxemia

Incorrect Marks for this submission: 0/1. Question 16 Marks: 0/1

A teratogen is any factor, chemical or physical, that adversely affects the fertilized ovum, embryo, or fetus. To reach maturity in optimal health, a fetus needs sound genes and a healthy intrauterine environment that protects it from the influence of teratogens. Evidence over the years has shown that when women consume a large quantity of alcohol during pregnancy, their babies show a high incidence of congenital deformities and cognitive impairment. Which of the following best explain the effect of alcohol to the fetus? Choose one answer. a. alcohol causes vasoconstriction of the uterine vessels that limits blood supply to the fetus

b. alcohol can interfere with the cell metabolism that can lead to cell mutation

- fetuses cannot remove the breakdown of products of alcohol from their body. Large buildup of the products of alcohol, which the c. fetuses cannot remove the breakdown of products fetus cannot breakdown, happens to the fetus of alcohol from their body because resulting to vitamin B deficiency and accompanying neurologic damage. It is nicotine in the cigarette which causes vasoconstriction of the uterine vessels that limits supply of blood to the fetus. Hyperthermia is another teratogen that interferes with normal cell metabolism. (Pilliterri) d. alcohol increases the risk for transmission of infection from the mother to the

fetus

Incorrect

Marks for this submission: 0/1. Question 17 Marks: 0/1

Placenta previa is low implantation of the placenta instead of in the upper portion of the uterus. A patient who has placenta previa and in her 32nd week of pregnancy shows signs of labor and was given Betamethasone (Celestone) 12 mg IM. The patient's husband asked you what is the purpose of this drug. The most appropriate response would be: Choose one answer.

I
a. "It decreases the occurence of painful contractions" b. "It protects your wife and your baby from infection." c. "It hastens the maturity of your baby's lungs" - "It hastens the maturity of your baby's lungs". Bethamethasone is a corticosteroid that acts as an anti-inflammatory and immunosuppressive agent. It is given in pregnant women 12 to 24 hours before birth to hasten fetal lung maturity if the fetus is less than 34 weeks' gestation and

help prevent respiratory distress syndrome in newborn.

d. "It decreases likelihood of occurence of bleeding since your wife is prone to it.

Incorrect Marks for this submission: 0/1. Question 18 Marks: 0/1 The immaturity not only places infants at risk for neonatal complications but also may predispose the infant to problems that persist into adulthood. Which of the following factors makes a premature infant more prone to experiencing aspiration than a term infant? Choose one answer. a. muscle tone in the cardiac sphincter is poorer

b. more decreased capacity to digest and absorb protein

Incorrect

Marks for this submission: 0/1. Question 19 Marks: 0/1

Burn patients experience a hypometabolic phase immediately after injury lasting 2 or 3 days when their metabolic rate and cardiac output decrease. Following this phase, patients experience a hypermetabolic phase. The most appropriate nursing diagnosis during this phase

I
c. the stomach has more limited capacity d. the initial sucking is accompanied by ineffective swallowing

- the stomach has more limited capacity. The stomach of a preterm has a more limited capacity than a full term and is easily over distended, compromising the respiration and increase the risk of respiration. Although poor muscle tone in the cardiac/esophageal sphincter contribute to aspiration in the newborn, both full term and preterm infants have this. This causes milk in the stomach to be easily regurgitated into the esophagus, where it can trigger the vagal reflex and cause apnea (stimulation) and bradycardia, increasing the risk of aspiration. Physiologically, full term infants have approximately the same capacity to digest and absorb protein as full-term infants. This also does not directly increase occurrence of aspiration. The initial sucking of a preterm infant is not accompanied by any swallowing, increasing the likelihood of aspiration. As infants mature, the sucking-swallow pattern develops but is slow and ineffectual.

when the patient experiences muscle weakness and poor wound healing is: Choose one answer. a. Acute Pain

b. Imbalanced Nutrition: Less than Body Requirements

- Imbalanced Nutrition: Less than Body Requirements. This hypermetabolic response, begins on fifth postburn day and continues up to 9 months after burn, is characterized by a hyper dynamic circulatory response with increased body temperature, oxygen and glucose consumption, carbon dioxide production, glycogenolysis, proteolysis, and lipolysis. In effect, there is erosion of lean body mass, muscle weakness, immunodepression, and poor wound healing. In the child, glycogen stores are limited to meet the burn's increased energy demands. The initiation of protein and lipid catabolism for glycogenolysis is accelerated. This prolonged metabolic dysfunction may lead to loss of lean body mass and increased morbidity. Imbalanced Nutrition: Less than Body Requirements is a state in which a person who is not on NPO experiences or is at risk of experiencing reduced weight related to inadequate intake or metabolism of nutrients for metabolic need which is present in this hypermetabolic state. Decreased cardiac output is a nursing diagnosis during the hypometabolic state. This hypermetabolic state does not cause pain. Risk for injury may be a secondary diagnosis when the patient becomes prone to disabilities caused by poor nutrition.

c. Risk for Injury

d. Decreased Cardiac output

Incorrect

Marks for this submission: 0/1. Question 20 Marks: 0/1

Although caloric needs are diminished in relation to body size during the school-age period, resources are being laid down for the increased growth needs of the adolescent period. It is important to impress on children and their parents the value of a balanced diet to promote growth. A Grade 4 teacher asks you on how she can promote nutrition education to her students. You know that she should: Choose one answer.

I
a. Ask children to select foods from a fast-food restaurant menu and identify those which are nutritious. b. Encourage parents that they should be the ones to teach their parents regarding nutrition since the subjects in the schools should be spent in more important lessons. c. Avoid the use of food as rewards for behavior.

- Avoid the use of food as rewards for behavior. It is more appropriate to use verbal praise and token gifts to reinforce healthy eating and physical activity than using foods as rewards.

Have students prepare nutritious foods, plan menus, and develop a recipe book of healthy foods. This will allow them to discover for themselves those foods which are nutritious and can reinforce creativity. Incorporate nutrition education into other classes (such as using a computer to analyze the nutritional content of foods). Asking children to select foods from a fast-food restaurant menu and identify those foods high in fat, cholesterol, and sodium can discourage them to eat in such places better than asking them to find the nutritious foods in fast-food chains. (Wong) d. Refrain from allowing the child to prepare foods since she will choose delicious, non-nutritious ones according to her preference.

Incorrect Marks for this submission: 0/1. Question 21 Marks: 0/1 Ultrasonagraphy, a much-used tool in modern-obstetrics, measures the response of sound waves against solid objects. Your patient, in the second month of pregnancy is scheduled for an ultrasound exam for the first time. Your nursing interventions before the procedure include: Choose one answer. a. Ask the patient to void immediately before the procedure since the a full bladder affects the quality of the result.

I
b. Encourage the father to come if the couple wants to since the procedure is safe even for him. - Encourage the father to come if the couple wants to since the procedure is safe even for him. Before an ultrasound examination, the patient needs a good explanation of what will happen and reassurance that the process does not involve x-rays. This means it is also safe for the father of the child to remain in the room during the test. For the sound waves to reflect best and the uterus to be held stable, it is helpful if the mother has a full bladder at the time of the procedure. To ensure this, she should drink a full glass of water every 15 minutes beginning 90 minutes before the procedure and should not void before the procedure. There is no need for the woman to be placed on NPO since the procedure does not involve assessing the gastrointestinal tract. (Pilliterri) c. Explain what will happen during the procedure and advise her that since x-rays are being emitted by the machine, she should not first interact with others during the first three hours after the procedure. d. It is also imperative for the patient to be placed on NPO 6 hours before the procedure.

Incorrect

Marks for this submission: 0/1. Question 22 Marks: 0/1

Juvenile rheumatoid arthritis is a disease of connective tissue with joint inflammation and contracture and is most likely the result of an autoimmune response. Women with this condition frequently take corticosteroids and non-steroidal anti-inflammatory drugs (NSAIDs) to prevent joint pain and loss of mobility. Pregnant patients with this condition should be advised to limit or discontinue taking NSAIDs, particularly Aspirin because of which effects: Choose one answer. a. prolonged pregnancy

b. late closure of the ductus arteriosus in the child

c. potent carcinogen to the mother

I
- prolonged pregnancy. Large amounts of salicylates may lead to increased bleeding at birth or prolonged pregnancy because salicylates interfere with prostaglandin synthesis, so labor contractions are not initiated. The drug also causes the infant to be born with bleeding defect and may also experience premature closure of the ductus arteriosus, however it is not a carcinogen. (Pilliteri)

Incorrect

Marks for this submission: 0/1. Question 23 Marks: 0/1

The treatment of Acute Renal Failure is directed toward the treatment of the underlying cause, management of the complications of renal failure and provision of supportive therapy within the constraints imposed by the renal failure. You are caring for a child with Acute Renal Failure (ARF). You know that the mother takes your health teaching sessions seriously when she gives her child which of the following foods? Choose one answer. a. Homemade french fries without salt

b. Mango ice cream

I
d. increased incidence of formation of blood clot in both mother and child

- Mango ice cream. The child with ARF develops oliguria and has a tendency to develop water intoxication, hyponatremia, and hyperkalemia. This is due to decreased excretion of sodium and potassium brought about by the oliguria. Foods

Incorrect

Marks for this submission: 0/1. Question 24 Marks: 0/1

Restraints must be ordered by a physician, with the type of restraint identified and how often the child can be removed from the restraint noted. Which of the following kind of restraint should be questioned by the nurse if ordered by the doctor for a pediatric patient? Choose one answer. a. Mummy restrain t

Co
c. Prune juice d. Buttered popcorn

high in potassium such as avocados, carrots, melons, prune juice, potatoes (french fries) and tomatoes should be avoided, as well as condiments, chips, popcorn, and processed meats which are all high in sodium. Mango has the least amount of sodium and potassium among the following foods. The mango and milk used in the ice cream is also high in carbohydrates which the child needs to supply needed calories and reduce protein catabolism. (Wong and Grodner)

Incorrect

Marks for this submission: 0/1. Question 25 Marks: 0/1

After the initial period of shock and restoration of fluid balance in a burn patient, the primary concern is the burn wound. Which

In
b. Elbow restrain t - Belt restraint. There are two kinds of restraints: human and c. Belt mechanical. Human restraint involves using a trained person to restrain hold the child in a particular position for a procedure rather than t using a mechanical device, which is present in a mechanical restraint. Mummy and elbow restraints are kinds of mechanical restraints. A mummy restraint uses a blanket to cover the entire body except for the head. In an elbow restraint, as the name suggests, a restraint is wrapped snugly over the elbows with a soft padding. There is no belt restraint. (Mandleco) d. Human restrain t

intervention is the most vital in the management of a major burn wound in a pediatric patient? Choose one answer. a. debrideme nt

b. fluid replacemen t

The answer is letter C. In children with large, full thickness burn wounds, the excision is performed as soon as the patient is hemodynamically stable after initial resuscitation. Because the burn wound precipitates the exaggerated physiologic response, c. excision many associated complications do not resolve until the eschar (thick, fibrin-containing necrotic tissue that interferes with healing by preventing wound contraction) is excised and wound is closed. One of the most effective therapies for decreasing mortality from major burn injuries is the early excision of the burn wound and its coverage by skin substitute. Debridement of devitalized tissue, exudate comes secondary to excision. After cleaning the wound through excision and debridement, the wound is closed by temporary skin substitutes. Fluid replacement is an intervention done in the primary stage of burn management when the child is prone to developing shock. d. temporary skin substitute

Incorrect

Marks for this submission: 0/1. Question 26 Marks: 0/1

Because of the complexity of many defects and the variability of their clinical manifestations, the cyanotic-acyanotic classification system of the congenital heart defects has proven to be inadequate and misleading. A more useful classification is based on hemodynamic characteristics, or movements involved in the circulation of blood: increased pulmonary blood flow, decreased pulmonary blood flow, obstruction to blood flow out of the heart, and mixed blood flow. Which of the following congenital heart defect is classified as having decreased pulmonary blood flow? Choose one answer. a. transposition of great arteries

In
b. coarctation of the aorta c. Tetralogy of fallot - Tetralogy of fallot. The classic form of tetralogy of fallot includes four defects: ventricular septal defect (VSD), pulmonic stenosis (Causing the decreased blood flow to the

lungs), overriding aorta, and right ventricular hypertrophy. In transposition of the great arteries, the pulmonary artery leaves the left ventricle and the aorta exits from the right ventricle, with no communication between the systemic and pulmonary circulations so that saturated and desaturated blood mix within the heart and great arteries. In coarctation of the aorta, there is localized narrowing near the insertion of the ductus arteriosus, which results in increased pressure proximal (upper extremities) to the defect and decreased pressure distal (lower extremities) to it. The narrowing causes obstruction of blood flow from the left ventricle to the aorta. Patent ductus arteriousus there is continuous patency of the fetal ductus arteriosus allows blood to flow from the higher pressure aorta to the lower pressure pulmonary artery causing increased blood flow to the pulmonary circulation. (Wong) d. patent ductus arteriosus

Incorrect Marks for this submission: 0/1. Question 27 Marks: 0/1 The Lamaze method of prepared childbirth is based on the gate control theory of pain relief. It was originally termed the psychoprophylactic method, as it focuses on preventing pain in labor by use of the mind. Which of the following techniques is not used in the Lamaze method to assist in gating mechanisms thus decreasing the perception of pain? Choose one answer. a. effleurage

Incorrect

Marks for this submission: 0/1. Question 28 Marks: 0/1

Medications are an important adjunct to resuscitation, especially in cardiac arrest, and are used during and after resuscitation in children. As a head nurse in the pediatric ward, it is your duty to orient student nurses assigned to your area. During orientation, one of the patients

In
b. health teaching - use of pain medications. Administration of pain medications is c. use of not recommended by the Lamaze method because its pain assumption is that the mind can control pain in labor based from medication the gate theory of pain. This theory refers to gate control s mechanisms in the brain that are capable of halting an impulse at the level of the spinal cord so the impulse is never perceived at the brain level as pain. The techniques that assist in gating mechanisms are cutaneous stimulation (e.g. effleurage), distraction (e.g. breathing, vocalization, focusing), and reduction of anxiety (e.g. teaching a woman what to expect during labor). (Pilliterri) d. vocalizatio n

was being resuscitated. You allowed the students to observe for a while. After the patient was successfully revived, one of your students asked you the why did the doctor ordered Atropine sulfate. You know that the primary action of this drug is: Choose one answer. a. It is an antidysrhythmic drug that allows the heart to maintain the regular beat of the heart

b. It reverses respiratory arrest brought about by the cessation of the function of the heart

- It increases cardiac output and heart rate which is important since heart ceases to function during an arrest. Atropine sulfate is c. It increases cardiac output an anticholinergic-parasympatholytic drug and heart rate which is that increases cardiac output and heart rate important since heart ceases by blocking the vagal stimulation of the to function during an arrest heart. Lidocaine HCl is the antidysrhythmic drug that affect the action potential of the cardiac cells, altering their automaticity, conductivity, or both. An anti-arrythmic also decreases the an increased heart rate that may cause an arrythmia. Naloxone is the resuscitative drug that reverses respiratory arrest. (Wong) and (Karch) d. It decreases heart rate which abnormally increases during a cardiac arrest

Incorrect

Marks for this submission: 0/1. Question 29 Marks: 0/1

Many factors enter the categorization of high risk pregnancy. Most health care agencies use some tool to determine high risk, but no tool is perfect because the concept of high risk is very individualized. In which of the following situations show the psychosocial factor that is least suggestive of high risk pregnancy? Choose one answer. a. Via, 4 weeks pregnant because her ex-boyfriend raped her

b. Gia, 4 weeks pregnant and has hyperthyroidism

c. Mia, 4 weeks pregnant, living in a shanty under the bridge

I
- Gia, 4 weeks pregnant and has hyperthyroidism. All the other choices involve psychosocial factors affecting the women: inadequate and poor housing (Mia), adolescent pregnancy (Ria), and lack of acceptance of pregnancy (Via). Hyperthyroidism on the other hand, is part of the medical history. (Pilliterri)

Incorrect

Marks for this submission: 0/1. Question 30 Marks: 0/1

One of the most distressing side effects of several drugs is gastrointestinal mucosal cell damage, which results in ulcers anywhere along the alimentary tract. Which of the following interventions for oral ulcers in a toddler undergoing chemotherapy is the most acceptable? Choose one answer.

I
d. Ria, 17 years old and is 4 weeks pregnant

a. To reduce oral pain, massage - To reduce oral pain, massage the back of the back of both hands with an both hands with an ice cube. A strategy ice cube that may be helpful in reducing oral pain is massaging the area on the back of both hands between the thumb and index finger with an ice cube for 5 to 7 minutes until the area becomes numb. A satisfactory method of cleaning the gums is to wrap a piece of gauze around a finger; soak it in a saline or plain water; and swab the gums, palate and inner cheek surfaces with the finger. Mouth rinses are best accomplished with plain water or saline because the toddler cannot yet gargle or spit out excess fluid. Agents such as lemon glycerin, hydrogen peroxide are avoided because of the drying effects

on the mucosa and lemon may irritate eroded tissue and can decay the teeth. Mouth care is done routinely before and after feeding and as often as every 2 to 4 hours to rid mucosal surfaces of debris, which becomes an excellent medium for bacterial and fungal growth. Drinking can usually be encouraged if a straw is used to bypass the ulcerated oral mucosa. (Wong) b. To clean the gums, use lemon glycerin to swab on the gums, palate and inner cheek surfaces

c. Using a straw when drinking is usually not advisable because the straw can increase likelihood of damage to the area

d. Mouth care should be done two to thrice a day to reduce further injury from the activity

Incorrect Marks for this submission: 0/1. Question 31 Marks: 0/1

According to Erik Erikson's psychosocial stage of development, a sense of industry, or a stage of accomplishment is achieved somewhere between age 6 years and adolescence. This stage is described by the following except: Choose one answer. a. This stage is the time when children learn the value of doing things with others and the benefits derived from division of labor in the accomplishment of goals.

b. The danger in this period is the occurence of situations that might result in a sense of role confusion.

- The danger in this period is the occurrence of situations that might result in a sense of role confusion. All of the following choices refer to this stage of industry vs. inferiority in the school-age. The goal of this stage of development is to achieve a sense of personal and interpersonal competence through the acquisition of technologic and social skills. Failure to develop a sense of accomplishment may result in a sense of inferiority or lack of self-worth. Identity vs. role confusion is for the adolescent stage. (Wong)

c. In this stage, interests expand and with a growing sense of independence, children want to engage in tasks that can be carried through completion.

d. Peer approval is a strong

motivating power.

Incorrect

Marks for this submission: 0/1. Question 32 Marks: 0/1

Cancers in children are often difficult to recognize. Therefore, being alert to the persistence of unusual symptoms is essential. All but one of the following is a cardinal symptom of cancer in children: Choose one answer. a. excessive weight gain

b. unexplain ed pallor

c. prolonged fever

I
- excessive weight gain. All are cardinal symptoms of cancer except for excessive weight gain. Weight loss together with pallor, lethargy generalized malaise is common in the child with malignant invasion of the bone marrow. Fever is most often caused by infection secondary to malignant process. Sudden eye and vision changes may also occur especially when the cancer cells affect the facial area. The presence of a white reflection as opposed to the normal pupillary reflex in the pupil of a child's eye is the classic sign of retinoblastoma. Squinting, strabismus, or swelling can indicate other solid tumors of the eye. (Wong)

Incorrect

Marks for this submission: 0/1. Question 33 Marks: 0/1

A post term infant is one born after the 42nd week of pregnancy. Which of the following assessment findings is least likely seen in a post term infant? Choose one answer. a. hypoglyc emia

In
d. sudden eye changes b. polycythe mia

c. meconiu m aspiration

d. decrease d hematocr it

- decreased hematocrit. In a post term infant, meconium aspiration is more likely to occur as fetal intestinal contents are more likely to reach the rectum. Hypoglycemia may develop because the fetus had to use stores of glycogen for nourishment in the last weeks of intrauterine life, since the placenta functions only up to 40-42nd weeks, also, polycythemia may have developed from decreased oxygenation. The hematocrit therefore is elevated because of the polycythemia and dehydration, which lowers the circulating plasma level. (Pilitteri)

Incorrect Marks for this submission: 0/1. Question 34 Marks: 0/1 The umbilical cord is formed from the fetal membranes and provides a circulatory pathway that connects the embryo to the chorionic villi of the placenta. Its function is to transport oxygen and nutrients to the fetus from the placenta. In the initial assessment of a newborn female child, you found that the umbilical cord has one vein and one artery. You will suspect that this child has a high risk of developing which condition? Choose one answer. a. undescended testes

b. congenital heart anomaly

- congenital heart anomaly. The number of vein (normally 1) and arteries (normally 2) in the cord is always assessed and recorded at birth. This is because 1% of all infants are born with a cord that contains only a single vein and artery. About 15% of these infants are found to have accompanying congenital anomalies, particularly of the kidney and heart.(Wong) Undescended testes will never occur since in the first place, the patient is female.

c. none of these since this finding is normal

d. congenital hearing and visual loss

Incorrect Marks for this submission: 0/1. Question 35 Marks: 0/1 Children as young as the early teens need guidelines for safer sex because of increasing trends of adolescents who are already sexually active. You are evaluating a group of 15 year old adolescents in a school where you are invited to teach about safe sex practice. You know that further teaching is needed of the student who will verbalize: Choose one answer. a. "Voiding immediately after sex is helpful in washing away

contaminants."

Incorrect

Marks for this submission: 0/1. Question 36 Marks: 0/1

I
b. "Anal intercourse carries increased risk for developing AIDS than vaginal sex." d. "The use of condom is the best protection against infection."

- "Condoms should be inflated before use to test if they are intact and is safe to use." Condoms c. "Condoms should be are the best protection against infection brought inflated before use to test by sexual intercourse. However, it should not be if they are intact and is inflated to test for intactness because it weakens safe to use." the rubber in which it is made of and may become prone to destruction. Voiding instantly after sexual relations may aid in washing away contaminants on the vulva or in the urinary tract. Anal intercourse, more prone to bleeding than vaginal, pose greater risk for the partners to contract AIDS. (Pilliterri)

Often a nurse is the one who first identifies a need for counseling and referral by identifying the presence of an inherited disorder in a family history or by noting physical, mental, or behavioral abnormalities when performing a nursing assessment. All of the following findings in the family history of your pediatric patient is considered to be an assessment clue to genetic abnormalities except: Choose one answer. - heart palpitation. The clues to genetic disorders include: (1) major or minor birth defects (anomalies) and dysmorphic features, e.g. cardiac defect, wide-set eyes, low-set ears; (2) growth abnormalities, e.g. short stature, overgrowth; (3) a. heart palpitation skeletal abnormalities, e.g. scoliosis, finger abnormalities; (4) visual or hearing problems, e.g. hearing loss, congenital cataracts; (5) metabolic disorders, e.g. unusual breath and stool odors; (6) sexual development disorder, e.g. micropenis, large testicles; (7) skin disorders, e.g. skin tumors, dry and scaly skin; (8) recurrent infection or immunodeficiency, e.g. ear infection, pneumonia; (9) development and speech delays or loss of developmental milestone; (10) cognitive delays such as mental retardation; (11) behavioral disorders, e.g. attention deficit, aggressive behavior. (Wong) b. attention deficit disorder

c. hearing loss

d. scoliosis

Incorrect Marks for this submission: 0/1. Question 37 Marks: 0/1 Menopause is the cessation of menstrual cycles. Perimenopausal is a term used to denote the period wherein menopausal changes are occurring. An obese 42-year old mother, asks you why she experienced menopause at this early age when her friend who is now 50 years old, nullipara, still has regular menstruation. You will ask her which of the following to confirm if the age of her menopause is normal: Choose one answer. a. age of menarche - age of menarche. The age of menopause, like the age of menarche (age of first menstruation), tend to be familial so if menopause occurred early in the mother, it may begin early in her daughter. The earlier the age of menarche, the earlier the menopause occurs. The number of children, age of first sexual activity and age of thelarche (breast development) do not correlate with the age of menopause. (Pilliterri)

b. the number of children

c. age of first sexual activity

d. age of thelarche

Incorrect

Marks for this submission: 0/1. Question 38 Marks: 0/1

Because genetic disorders often occur in varying degrees of expression, a careful physical assessment of any family member with a disorder, that child's siblings, and the couple seeking counseling is needed. A child with the following characteristics: slanted eyes, with epicanthal folds and large tongue, and is cognitively challenge is most probably manifesting which genetic disorder? Choose one answer. a. turner syndrome

In
b. klinefelter syndrome c. cri-duchat syndrome

Incorrect

Marks for this submission: 0/1. Question 39 Marks: 0/1

Assessing for the reflexes is an important element in the Pediatric Physical Assessment, especially that of an infant. It is a measure of an intact neurologic system. A student nurse caring for a sleeping 6month old patient needs further reading when she noted in the chart that this reflex is positive: Choose one answer. a. Babins ki

In
- Down's syndrome Other manifestations of Down syndrome, having extra chromosome 21, are abnormal color of the iris and d. Down's late closure of the fontanelles. syndrome Klinefelter syndrome, having an extra X chromosome, is manifested by small testes, gynecomastia, and infertility. In turner syndrome, with only one X chromosome present, the patient has short stature, streak gonads, infertility and webbed neck. Patients with cri-du-chat (deletion of short arm of chromosome 5) also cognitively challenged, with facial structure anomalies and peculiar cat-like cry. (Pilliterri) b. Rootin g

Incorrect

Marks for this submission: 0/1. Question 40 Marks: 0/1

Radiography is used frequently in diagnostic evaluation in children. Although no definitive information exists on the effects of low-dose radiation, measures are carried out to protect vulnerable areas from possible damage. In caring for a child that is suspected of having brain tumor, the nurse would anticipate which radiologic examination to be done to the patient? Choose one answer. a. brush biopsy

In
c. Suckin g - Palmar grasp. Palmar grasp is elicited by putting your finger in d. infant's palm from ulnar side then the infant closes fingers around Palmar your finger. This is present at birth up to the 4th month. Rooting grasp reflex present from birth to 4 months and up to 12 months during sleeping and is elicited by stroking the infant's cheek and corner of the mouth. It is positive when the infant's head turns in that direction. The sucking reflex is present from birth to 4th month and up to 7 months during sleep. Present at birth to 2 years of age is the babinski that is elicited by stroking the bottom foot then the toes will fan. (Mandleco)

Incorrect

Marks for this submission: 0/1. Question 41 Marks: 0/1

Although the parent and child are separate and distinct individuals,

In
b. bronch ograph y - MRI. Magnetic resonance imaging (MRI) uses a large magnet and radio waves to produce 2- 3- dimensional image of organs such as the brain and heart. It clearly identifies soft tissues. Bronchography c. MRI involves instillation of contrast medium directly into the bronchial tree through opaque catheter inserted via orotracheal tube. Brush biopsy and capnography are not radiologic examinations. In a brush biopsy, the material for biopsy is obtained with a nylon brush on end of wire and passed through tube placed via nose, pharynx, trachea and airways. to involved lung segment. Capnography measures CO2 during inhalation and exhalation cycle and produces graph of CO2 concentration over time. (Wong) d. capnog raphy

the nurse's relationship with the child is frequently mediated via the parent, particularly in the case of younger children. Which of the following interventions facilitates effective communication with parents of pediatric patients the least? Choose one answer. a. active listening

b. using silence

c. directing the focus of the conversation

d. being sympathetic

- being sympathetic. Sympathy is having feelings or emotions in common with another person, rather than understanding those feelings. Sympathy is not therapeutic in the helping relationship because it leads to feeling emotionally over involved and potentially to professional burnout. Rather than being sympathetic, the nurse should be empathic, which is the ability to put oneself in another's shoes. Using silence permits the parents to sort out thoughts and feelings and search for responses to questions. Active listening is truly aimed at understanding the patient, it is an active process that requires concentration and attention to all aspects of the conversation. The nurse should have the ability to direct the focus of the interview while allowing for maximum freedom of expression.

Incorrect Marks for this submission: 0/1. Question 42 Marks: 0/1 Menses is actually the end of an arbitrarily defined menstrual cycle. Which of the following statements best describe menses? Choose one answer. a. Both B and C

b. The last day of the menstrual flow marks the beginning of a new menstrual cycle.

c. The iron loss is so small that the body is still able to adequately replace the iron loss.

d. It contains only little - It contains only little amount of blood flow, amount of blood flow, approximately 30 to 80 ml of blood. Contrary to approximately 30 to 80 ml common belief, a menstrual flow contains only of blood. approximately 30 to 80 ml of blood. If it seems like more, it is because of the accompanying mucus and endometrial shreds. The iron loss in

a typical menstrual flow is approximately 11 mg and is enough loss that many women need to take a daily iron supplement to prevent iron depletion during the menstruating years. Menses is the only external marker of the menstruation cycle so the first day of the menstrual flow is used to mark the beginning day of the new menstrual cycle. (Pilliterri) Incorrect Marks for this submission: 0/1. Question 43 Marks: 0/1 Amniocentesis is a procedure done in a physician's office or in ambulatory clinic and is typically scheduled between the 14th and 16th weeks of pregnancy to allow for a generous amount of amniotic fluid to be present and also at near term. Which of the following information is provided by this test? Choose one answer. a. grading of the placenta

b. volume of the amniotic fluid

c. biparietal diameter of the fetal head

Incorrect

Marks for this submission: 0/1. Question 44 Marks: 0/1

The overall management of infertility involves treating the underlying causes such as chronic disease, inadequate hormone production, endometriosis, or infection. You would advise a patient undergoing hormone therapy with Clomiphene citrate of the increased possibility of occurence of which condition: Choose one answer. a. early menopau se

b. myoma

In
d. fetal lung maturity - fetal lung maturity. Amniocentesis involves the aspiration of amniotic fluid from the pregnant uterus for examination. After amniocentesis, the lecithin/sphingomyelin (L/S), the protein components of the lung enzyme surfactant that the alveoli begin to form at about the 22nd to 24th weeks of pregnancy, ratio may be determined. An L/S ratio of 2:1 is traditionally accepted as lung maturity. It is the ultrasound which can measure the biparietal diameter of the fetus, grading of the placenta and the volume of the amniotic fluid. (Pilliterri)

Incorrect

Marks for this submission: 0/1. Question 45 Marks: 0/1

Cardiac arrest in a pregnant woman is most often related to events at the time of birth such as amniotic fluid embolism, eclampsia, stroke, hemorrhage, and acute respiratory failure. In which aspect is the cardiopulmonary resuscitation different between a pregnant patient and an adult non-pregnant one? Choose one answer.

In
c. ovarian atrophy - multiple pregnancy. Clomiphene citrate (Clomid) is an estrogen d. agonist used to stimulate the ovary. The drug binds to estrogen multiple receptors, decreasing the number of available estrogen receptors pregnanc and falsely signaling the hypothalamus to increase FSH and LH y secretion. It may overstimulate an ovary, causing multiple ova to come to maturity and possibly resulting in multiple births. Other possible adverse effects are ovarian enlargement , visual disturbances and nausea and vomiting. Development of myoma or ovarian atrophy is not associated with this drug (Pilliterri). a. the site where the pulse is palpated to establish circulation

Incorrect

Marks for this submission: 0/1. Question 46

I
b. the maneuver used to open the airway d. the ratio of compression to breathing to be given by one rescuer.

- the position of the patient. The protocol set by the American Heart Association in 2000 recommends standard CPR with the uterus displaced laterally, fluid resuscitation and defibrillation is indicated. To c. the position of the prevent supine hypotension, the pregnant woman is placed on a flat, firm surface with the uterus patient displaced laterally either manually or with a wedge or rolled blanket or towel under her right hip. If defibrillation is needed, the paddles need to be placed one rib interspace higher than usual because the heart is displaced slightly by the enlarged uterus. The maneuver to open airway (head tilt-chin lift), the pulse to palpate (carotid), and the ratio of compression (15) to breathing (2) for one-rescuer remains the same for both pregnant and nonpregnant adult patients. (Lowdermilk)

Marks: 0/1 Simply stated, acidosis (acidemia) results from either accumulation of acid or loss of base, and alkalosis (alkalemia) results from either accumulation of base or loss of acid. A newborn patient having the following ABG results: pH=7.49, pCO2= 46 mmHg and HCO3= 21mEq/ml, would alert you because this is a manifestation of: Choose one answer. a. metabolic acidosis

b. respirator y acidosis

c. respirator y alkalosis - respiratory alkalosis. The normal ABG values for a newborn infant are as follows: pH = 7.11-7.36, PCO2= 27-40 mmHg, HCO3= 21-28 mEq/ml. The pH is high (alkalosis), the pCO2 is also high (respiratory alkalosis). The HCO3 is normal.

d. metabolic alkalosis

Incorrect

Marks for this submission: 0/1. Question 47 Marks: 0/1 Many women enter pregnancy with a deficiency of iron stores resulting from a diet low in iron, heavy menstrual periods, or unwise weight-reducing programs. A patient who is in 24th week of pregnancy presents with the following signs and symptoms: generalized pallor, pale conjunctivae and nail beds, with cool clammy skin. Her complete blood count shows decreased hematocrit and hemoglobin levels. She was later diagnosed with iron-deficiency anemia. The most appropriate nursing diagnosis for this patient is: Choose one answer. - Ineffective tissue perfusion related to decreased circulating hemoglobin. Hemoglobin is the oxygencarrying component of the blood. Therefore, an inadequate amount of it (anemia), caused by a deficiency in iron, can leave the tissues with less oxygen as manifested by pallor of the mucous membranes and cool skin temperature. A decrease in hematocrit (percentage of redblood cells in the blood) is not a manifestation of deficient fluid volume, rather the increase of it. Deficient fluid volume is usually not seen in iron-deficiency anemia. Activity intolerance may be manifested by a pregnant woman with this condition but it is not experienced by the patient in the situation. This nursing diagnosis is for patients with a reduction in one's physiologic capacity to endure activities to the degree desired or required. Lastly, impaired cellular circulation is not included in the NANDA Approved Nursing diagnoses. (Pilliterri and Carpenito)

a. Ineffective tissue perfusion related to decreased circulating hemoglobin

b. Activity intolerance related to maternal anemia during pregnancy

c. Deficient fluid volume related to decreased hematocrit concentration

d. Impaired cellular circulation related to maternal anemia during pregnancy

Incorrect Marks for this submission: 0/1. Question 48 Marks: 0/1 Although most women are pleased to be pregnant, the symptoms of early pregnancy tend to cause discomfort to a woman rather than provide evidence that she is carrying a child. In a woman with nursing diagnosis of constipation related to reduced peristalsis in pregnancy, which of the following interventions is the most appropriate? Choose one answer. a. Promote use of mineral oil in relieving constipation so that vitamins will not be excreted.

b. Inform her that over-the- - Inform her that over-the-counter laxatives are counter laxatives are contraindicated. Over-the-counter laxatives are

contraindicated because they are non-essential drugs during pregnancy unless specifically prescribed or sanctioned by the physician. contraindicated. Evacuation of the bowels should be done regularly, according to the normal schedule of the woman, not specifically every other day. BRAT diet do not increase roughage in the bowel, rather it is recommended for patients experiencing diarrhea. Mineral oil should not be advised to relieve constipation because it can absorb fat-soluble vitamins (A,D,E,K), which are necessary for both good fetal and maternal health, and flush them out of the body. (Pilliterri)

c. Encourage her to evacuate her bowels every other day.

d. Advise her to take BRAT (banana, rice, apple and tea) diet to increase roughage in her diet.

Incorrect Marks for this submission: 0/1. Question 49 Marks: 0/1 When palpating for the testes during a pediatric physical examination, avoid stimulating the cremasteric reflex which pulls the testes higher into the pelvic cavity. A student nurse is having difficulty in her assessment asks you of an effective way of how to avoid eliciting this

reflex. You teach her that this is best done by: Choose one answer. a. positioning the patient in dorsal recumbent

b. placing the thumb and index finger over the lower part of the scrotal sac

c. positioning the patient in lithotomy

d. warming her hands

- warming her hands. The cremasteric reflex is stimulated by cold, touch, emotional excitement, or exercise. Therefore, it is best to warm the hands before doing the palpation. Other effective ways to avoid eliciting this reflex is by positioning an older child in a tailor or "Indian sit" position to stretch the muscles preventing its contraction and by placing the thumb and index finger over the upper part of the scrotal sac along the inguinal area to block the normal pathway of ascent of the testes. (Wong)

Incorrect Marks for this submission: 0/1. Question 50

Marks: 0/1 Kidney transplantation is the preferred means of renal replacement therapy in the pediatric age-group. Which of the following manifestations when exhibited by a patient who underwent kidney transplant 5 months ago is most suggestive of transplant rejection? Choose one answer. a. hypotensi on

- hypertension. Rejection is evidenced by both biochemical and b. hypertens clinical abnormalities. The most common finding is fever, which is usually accompanied by swelling and tenderness over the graft, ion hypertension, and diminished urinary output. A severe reaction may even cause oliguria. Increases in serum blood urea nitrogen and creatinine levels are laboratory evidence of decreased transplant function. Vomiting does not suggest transplant rejection. (Wong) c. decrease d BUN levels

d. vomiting

Incorrect

You might also like